Está en la página 1de 109

BANCO DE PREGUNTAS DE EMBRIOLOGÍA

FACULTAD DE MEDICINA

UNIVERSIDAD
NACIONAL DE
COLOMBIA
Sede Bogotá
Contenido

Primer mes de gestación...........................................................................................2


Segundo mes de gestación.....................................................................................17
Aparato cardiovascular............................................................................................20
Aparato digestivo.....................................................................................................38
Aparato faríngeo......................................................................................................48
Aparato genitourinario.............................................................................................61
Aparato locomotor....................................................................................................78
Aparato respiratorio.................................................................................................79
Aparato tegumentario..............................................................................................84
Órganos de los sentidos..........................................................................................85
Sistema nervioso.....................................................................................................88
PRIMER MES DE GESTACIÓN

1) Las células que forman las tres capas germinales primitivas se derivan de
¿Cuál de las siguientes?
a) Citotrofoblasto
b) Epiblasto
c) Sincitiotrofoblasto
d) Hipoblasto
e) saco vitelino

La respuesta es b. Las células de la masa celular interna (embrioblasto) del


blastocisto se diferencian en el epiblasto e hipoblasto. Las células del epiblasto
migran hacia la línea primitiva en la segunda semana y llegan a ser internalizadas,
la formación de las capas germinales del mesodermo y endodermo. Las células
restantes del epiblasto se convierten en la capa germinal ectodérmica. Las células
del hipoblasto (respuesta d) contribuirán al saco vitelino. Las células de la masa
celular externa del blastocisto se diferenciarán en el citotrofoblasto y
sincitiotrofoblasto (respuestas A y C), lo que contribuirá a la formación de la
placenta. El saco vitelino (respuesta e) se incorpora en el embrión como el
intestino primitivo durante el plegamiento embrionario. (Sin referenciar).

2) ¿El ectodermo se deriva directamente de cuál de las siguientes?


a) Endodermo
b) Células del epiblasto que se someten a la gastrulación
c) Mesodermo
d) Endodermo
e) Epiblasto No gastrulado

La respuesta es e. Las células no gastruladas del epiblasto forman el ectodermo


(epidermis, anexos epidérmicos, y el sistema nervioso). Durante la segunda
semana de desarrollo, el embrioblasto da lugar a dos capas germinales primitivas,
el epiblasto y el hipoblasto subyacente (respuesta a). Al comienzo de la tercera
semana, las células del epiblasto (respuesta b) migran hacia la línea media (línea
primitiva) y se mueven hacia el interior (la gastrulación). Las células del epiblasto
migran, desplazan a las células del hipoblasto a la periferia para formar el
revestimiento endodérmico (respuesta d) del tracto digestivo y formar una capa
intermedia de mesodermo (respuesta c) que dará lugar al músculo, hueso y
estructuras cartilaginosas. (Sin referenciar).

3) La placa neural se forma directamente de ¿cuál de las siguientes?


a) El ectodermo
b) Endodermo
c) Mesodermo de la Somatopleura
d) Esplacnopleura del mesodermo
e) Hipoblasto

La respuesta es a. La primera etapa de la formación del tubo neural es la


inducción por la notocorda y la placa precordal del mesodermo de la placa neural
que se forma del ectodermo. Esto se conoce como inducción primaria y se
acompaña de cambios moleculares en las moléculas de adhesión celular
[restricción a la molécula de adhesión celular neural (N-CAM)]. Esta primera etapa
de desarrollo del tubo neural es seguida por una fase de remodelación,
neurulación, y el cierre del tubo neural. Endodermo (la respuesta b) es
responsable de la formación del tubo digestivo y el sistema respiratorio. El
mesodermo somatopleural (respuesta c) hace importantes contribuciones a la piel
(dermis) y porciones no musculares de las extremidades. El mesodermo
esplacnopleural (respuesta d) es el corazón y los músculos del tracto
gastrointestinal y el sistema urinario. El hipoblasto (respuesta e) es la capa
delgada de células ventral al epiblasto; se desplaza por las células del epiblasto,
que forman endodermo. (Sin referenciar).

4) Los gemelos monocigóticos surgen por medio de los cuales de los


siguientes?
a) Fusión de los blastómeros embrionarios de dos cigotos
b) Fecundación de dos óvulos por dos espermatozoides
c) Fertilización de un ovocito por dos espermatozoides
d) División de la masa celular interna (embrioblasto) en dos primordios
embrionarios
e) Divisiones de escisión adicionales del cigoto inducidos por la presencia de
un doble corion

La respuesta es D. Los gemelos monocigóticos surgen de las divisiones del


embrioblasto para formar dos embriones. Gemelos monocigóticos también se
forman de la separación temprana de los blastómeros. Básicamente, los gemelos
monocigóticos pueden surgir en cualquier fase desde las dos células
(blastómeros) a la fase de mórula. Gemelos monocigoticos se hayan con
placentas fusionadas o separadas, sacos bicoriales separados o condensados o
un saco coriónico y sacos biamnióticos. Gemelos dicigóticos surgen de la
fecundación de dos óvulos por dos espermatozoides (respuesta b) y no son más
que "compañeros vientre." Se diferencian en el genotipo y, por lo tanto, puede ser
diferente sexo. La fertilización de un ovocito por dos espermatozoides (respuesta
c) no puede ocurrir debido a la Ca2 + dependiente del bloqueo de la poliespermia
(ver pregunta 6). Esa reacción cortical luevo afecta a la zona pelúcida de dos
maneras: (1) la hidrólisis de hidratos de carbono impide la unión de esperma y (2)
la actividad proteolítica se endurece. (Sin referenciar).

5) Una mujer de 19 años de edad, que se encuentra en la semana 20 de


gestación de su primer embarazo, asiste al médico presentando
hemorragia vaginal y ausencia de movimientos fetales. No tiene
antecedentes de ninguna enfermedad. El examen físico muestra un útero
más grande de lo esperado para el periodo de la gestación. Durante una
ecografía se observan pequeñas vesículas translúcidas, de paredes
delgadas, de diferentes tamaños; y un poco de líquido. ¿Cuál de las
siguientes es la complicación más probable de la condición de este
paciente?
a) Coriocarcinoma
b) Tumor de células de la granulosa
c) Aumento del riesgo de carcinoma de mama
d) Mola invasora
e) Ovarios poliquísticos

La respuesta correcta es D. Esta paciente tiene muy probablemente una mola


hidatiforme completa. Esta forma de enfermedad trofoblástica gestacional, se
considera que es el resultado de la fecundación de un óvulo que ha perdido sus
cromosomas, por un solo espermatozoide portador de X con la duplicación del
núcleo, o por dos espermatozoides. La complicación más común de la mola
hidatiforme es la mola invasora (10 a 15 %); y con menor frecuencia (2-3%),
puede ocurrir el coriocarcinoma (Opción A). Las molas invasoras pueden
inmiscuirse lo suficiente en la pared del útero para que la hemorragia sea una
complicación potencial. Los tumores de las células de la granulosa (Opción B) son
un tumor tipo estroma y de los cordones sexuales. Estos tumores están
compuestos de proporciones variables de la granulosa y/o células de la teca.
Pueden presentar sangrado uterino anormal en mujeres postmenopáusicas y
menometrorragia en mujeres en edad reproductiva. La mola hidatiforme no
aumenta el riesgo de desarrollar cáncer de mama (Opción C). El síndrome de
ovario poliquístico (síndrome de Stein-Leventhal) se asocia con infertilidad,
menstruaciones irregulares, hirsutismo y ovarios poliquísticos (Opción E). También
puede ir acompañada de síndrome metabólico con obesidad central y resistencia a
la insulina. (Sin referenciar).

6) Cuando se está examinando histológicamente una sección de ovario


normal, el histotecnólogo observa un oocito rodeado de varias capas de
células foliculares. Hay un pequeño atrio presente. ¿Cuál de las
siguientes opciones es correcta para la estructura completa, compuesta
por el oocito, las células foliculares y el atrio?
a) Cuerpo lúteo.
b) Folículo de Graaf.
c) Folículo primario.
d) Folículo primordial.
e) Folículo secundario.

La respuesta es E. Los folículos en los diferentes estadios de maduración tienen


apariencias diferentes. D es incorrecto los folículos más primitivos, los folículos
primordiales, son folículos inactivos en reserva que contienen ovocitos primarios
(atrapados en la profase de la primera división meiotica) rodeados por una sola
capa de células foliculares aplanadas. C es incorrecta los folículos primarios, el
siguiente estadio, son ligeramente más grandes y contienen un ovocito central
rodeado por una o varias capas de células foliculares cubicas y se forma la zona
pelucida las células de la granulosa y la teca. Cuando varios espacios pequeños
con líquido folicular en la masa folicular se fusionan forman un atrio (cavidad
folicular), el folículo es denominado ahora folículo secundario por tanto E es
incorrecta. Los folículos secundarios continúan creciendo, y desarrollan una
estructura más compleja que incluye un cúmulo ooforo que luego de la ovulación
se denomina corona radiada. B es incorrecta el folículo de Graaf es una forma
madura del folículo, que se extiende a través de todo el córtex y sobresale en la
superficie ovárica. A es incorrecta luego de su ruptura y la liberación del ovulo, el
cuerpo lúteo se desarrolla de las células del folículo y las células agrandadas de la
teca interna, convirtiéndose en células luteinicasepiteloides, y secretando
estrógeno. Las células granulosas luteinicas que contienen un pigmento amarillo
propio de contenido lipídico y secretan estrógenos y progesterona. Si el embarazo
no ocurre, el cuerpo lúteo, eventualmente, se degenera a cuerpo blanco; si ocurre
el embarazo, se mantiene a través de la gestación. (Sin referenciar).

7) Una mujer de 35 años de edad se somete a una histerectomía total porque


ella ya tiene hijos y tiene múltiples leiomiomas que le están causando
dolor pélvico crónico. Secciones histológicas a través del ovario
demuestran múltiples folículos primarios, cada uno conteniendo un
oocito. Estos oocitos se encuentran detenidos en ¿Cuál de las siguientes
etapas de la meiosis?
a) Anafase
b) Interfase
c) Metafase
d) Profase
e) Telofase

La respuesta es D. Una de las características inusuales de la meiosis en mujeres


es que los oocitos están detenidos, por gran parte de la vida de la mujer, en
profase de la primera división meiotica. Ellos terminan la segunda división meiotica
solamente después de que el oocito ha sido liberado de un folículo maduro y ha
sido penetrado por un espermatocito. Esto parcialmente explica porque la edad
materna tiene un mayor impacto sobre el riesgo de trisomías (como el síndrome
de Down) a comparación con la edad paterna, dado que más cosas le pueden
ocurrir durante un periodo de 40 años a los cromosomas que ya están pasando
por meiosis a comparación de aquellos que están relativamente protegidos en
interfase. (Sin referenciar).
 Interfase es la etapa durante la cual las células no se están activamente
dividiendo.
 Metafase es la etapa durante la cual los cromosomas están alineados a lo
largo de las placas ecuatoriales.
 Anafase es la etapa durante la cual los cromosomas empiezan a alejarse
de la placa ecuatorial.
 Telofase es la etapa durante la cual los cromosomas han sido separados
a lados opuestos de los husos mitóticos, y la división citoplasmática está
cerca a ocurrir.

8) Una mujer de 28 años que no tuvo cuidados prenatales da a luz a las 42


semanas de gestación. El neonato tiene algunas vísceras abdominales
sobresalientes a través de un defecto en la pared abdominal. ¿Cuál de las
siguientes es la causa más probable de defecto?
a) Fallo del bucle intestinal para retraerse desde el cordón umbilical.
b) Fallo en la degeneración del saco vitelino.
c) Fallo en la fusión peritoneal.
d) Fusión incompleta de los pliegues laterales corporales.
e) Hernia Umbilical.

La respuesta es D. Durante la cuarta semana del desarrollo, los pliegues


laterales corporales se mueven ventralmente y se fusionan en la línea media
desde la pared corporal anterior. La fusión incompleta resulta en un defecto que
permite a las vísceras abdominales sobresalir de la cavidad abdominal sin estar
cubiertas por peritoneo, una condición conocida como gastrosquisis. Durante el
desarrollo el intestino medio se hernia en el cordón umbilical y subsecuentemente
se retrae dentro de la cavidad abdominal. A es incorrecta en fallo en la retracción
del bucle intestinal desde el cordón umbilical resulta en onfalocele. B es incorrecta
el fallo en la degeneración del saco vitelino resulta en un divertículo íleal (de
Meckel) o una fistula o seno vitelino. En el embrión temprano, el intestino está
conectado al saco vitelino por una conexión estrecha denominada tallo vitelino.
Normalmente esta conexión se degenera. Durante el desarrollo, algunos órganos
peritoneales se fusionan con la pared abdominal posterior para convertirse en
retroperitoneales secundarios. C es incorrecta el fallo de esta fusión peritoneal
causaría que ciertos órganos que están fijos usualmente puedan moverse (ej.
Ciego móvil). E es incorrecta una hernia umbilical resulta de la protrusión de
vísceras abdominales a través de una debilidad en la pared abdominal luego del
desarrollo, tales protrusiones están cubiertas por fascia subcutánea y piel, lo cual
la distingue de la gastrosquisis. (Sin referenciar).

9) Una pareja llega a su médico por consejo acerca de anticoncepción y


planificación familiar. El médico les da consejos acerca de varios
métodos de planificación. Mientras discuten la opción de usar un
dispositivo intrauterino, una discusión del proceso de implantación
aparece. ¿A qué etapa del desarrollo embrionario normalmente se
implanta un embrión en el endometrio?
a) Blastocisto.
b) Estadio de cuatro células.
c) Mórula.
d) Embrión trilaminar.
e) Cigoto.

La respuesta es A. Luego de la fertilización, el óvulo fertilizado se empieza a


dividir mientras migra a través de la trompa uterina. Alcanza la etapa de
blastocisto (aproximadamente 110 células) hacia el día 5, y entra en el útero hacia
el día 6. La implantación normal empieza en el día 6 con el sincitiotrofoblasto del
polo embrionario del blastocisto que erosiona el endometrio. E es incorrecta luego
de la fertilización, inician las divisiones del cigoto. Éstas son divisiones mitóticas
que resultan en el aumento de número de las células pero no en un aumento en el
número total de masa citoplasmática. B es incorrecta el cigoto se divide en un
embrión de dos células, y de éstas células luego para formar un embrión de cuatro
células. El embrión sigue estando en el límite superior de la trompa uterina en esta
etapa de desarrollo, y está rodeado por la zona pelúcida, que previene que se
implante. C es incorrecta las divisiones continúan para eventualmente resultar en
una bola sólida de células llamada mórula hacia el día 4. La mórula sigue estando
en la trompa uterina pero está cerca a entrar en el útero. D es incorrecta el
embrión trilaminar es formado durante la tercera semana. Éste resulta de la
gastrulación, el proceso en que las células del epiblasto migran a través del saco
primitivo para las capas germinativas y la notocorda. Esto ocurre tiempo después
de que la implantación ha ocurrido. El Cigoto es un embrión unicelular que es
creado en la fertilización luego de la unión del pronúcleo masculino y femenino.
Éste es considerado el día 1 de desarrollo y típicamente ocurre en la ampolla de la
trompa uterina. (Sin referenciar).

10)Durante el desarrollo embrionario, la hematopoyesis ocurre en diferentes


órganos en diferentes momentos. ¿Cuál de los siguientes son los
órganos correctos, en el orden correcto, en el que la hematopoyesis
ocurre embriológicamente?
a) Amnios, saco vitelino, médula ósea.
b) Placenta, hígado y bazo, saco vitelino, médula ósea.
c) Placenta, bazo y órganos linfáticos, médula ósea.
d) Saco vitelino, médula ósea, hígado y bazo.
e) Saco vitelino, hígado, bazo, médula ósea.

La respuesta es E. Para la tercera semana del desarrollo, la hematopoyesis


comienza en las islas sanguíneas del saco vitelino. Comenzando el primer mes de
edad y continuando hasta el 5 mes, los elementos sanguíneos son formados en el
hígado y el bazo más que formación promueve destrucción de elementos formes
de la sangre. La hematopoyesis luego ocurre en la medula ósea hasta el parto y
de allí en adelante es el principal órgano hematopoyético, en el adulto puede
ocurrir en otros órganos linfáticos. A, B, C, son incorrectas. D lista los órganos
correctos en el orden incorrecto. (Sin referenciar).

11)Una mujer de 25 años de edad se presenta en obstetricia luego de


hacerse un test casero de embarazo, con un resultado positivo. Afirma
que en su familia ha habido gemelos y que le gustaría un ultrasonido para
saber si ella tiene un embarazo de gemelos. Los estudios radiográficos
confirman que el embrión se ha dividido en la etapa de blastocisto. La
división del embrión en la etapa del blastocisto resulta en cuál de las
siguientes opciones:
a) Siameses.
b) Gemelos dicigoticos.
c) Gemelos fraternos.
d) Gemelos monocigóticos.
e) Una sola gestación.

La respuesta es D. Gemelos monocigóticos, o gemelos idénticos, desarrollados


de un solo huevo fertilizado dividido subsecuentemente, durante cualquiera de los
dos estadios, blastómero o blastocisto. Es más común en el estadio del blastocisto
hacia el final de la primera semana. Ello resulta en dos masas celulares internas
dentro de la misma cavidad. Ellos usualmente desarrollan una placenta y una
cavidad corionica comunes, pero cavidades amnióticas separadas. La división
durante la segunda semana usualmente se acompaña de cavidades amnióticas
comunes también. A es incorrecta los siameses resultan de la división incompleta
del blastocisto. B y C son incorrectos los gemelos dicigoticos y gemelos fraternos
son lo mismo, y son el tipo de gemelos más común. Ellos comparten la misma
relación génica que tienen dos hermanos de diferentes gestaciones. Este tipo de
gemelos se dan cuando simultáneamente ocurre una doble ovulación y una
fertilización doble de dos espermatozoides. Cada uno de ellos desarrolla su propia
placenta y membranas. E es incorrecta una sola gestación, y un solo nacimiento,
es el resultado de una sola fertilización sin ninguna división del embrión. (Sin
referenciar).

12)Un recién nacido de 4 días de vida experimenta una corrección quirúrgica


de una masa espinal que fue diagnosticada en el útero. Una masa bien
definida situada en la cara caudal de la médula espinal es completamente
eliminada. El examen del tejido muestra tejido adiposo, tejido neural,
tejido pancreático, piel, y músculo esquelético. Los tejidos están
dispuestos de una manera desordenada. ¿Cuál de los siguientes es el
origen más probable de esta masa?
a) Ectodermo
b) Endodermo
c) Mesodermo
d) Restos notocorda
e) Restos línea primitiva

La respuesta correcta es E. Este bebé tenía un teratoma sacro coccígeo. Los


teratomas se presentan en muchos lugares y se presentan como masas. La región
sacro coccígea es el sitio más frecuente. Estos tumores se presentan más
comúnmente en niños pequeños y pueden ser diagnosticados en el útero o al
nacimiento. La mayoría de los afectados son mujeres. La frecuencia de malignidad
aumenta con la edad de presentación. Los teratomas contienen a menudo
derivados de las tres capas germinales: ectodermo, mesodermo y endodermo.
Debido a que las tres capas germinales están presentes, se cree que los
teratomas sacro coccígeos surgen de la línea primitiva (una etapa anterior a la
separación ectodermo, mesodermo y endodermo). (Sin referenciar).

13)Los siguientes son procesos que se llevan a cabo en la tercera semana


de vida intrauterina:
a) Establecimiento de la bilateralidad del embrión.
b) Formación de la línea primitiva por células mesodérmicas de diferente
polaridad.
c) Invaginación de todas las células del mesodermo por la línea primitiva.
d) a y b son ciertas.
e) a, b y c son ciertas

Respuesta: E. Establecimiento de la bilateralidad del embrión, formación de la


línea primitiva por células mesodérmicas de diferente polaridad e invaginación de
todas las células del mesodermo por la línea primitiva. Durante la tercera semana
se da principalmente el proceso de la gastrulación, iniciando con la aparición de la
línea primitiva desde el extremo cefálico, estableciendo así la bilateralidad del
embrión, y la invaginación en la fosita primitiva de células prenotocordales hasta el
establecimiento de la placa notocordal (Embriología de Langman, págs. 65 y 66).

14)Las células de determinación mesodérmica del nudo embrionario se


activan solamente hacia:
a) Final de la primera semana
b) Final de la segunda semana
c) Final de la tercera semana
d) Final de la cuarta semana
e) Final de la quinta semana

La respuesta es c. La tercera semana del desarrollo se caracteriza por la


formacion en la superficie del ectoblasto que mira a la cavidad amniotica de la
linea primitiva. El extremo craneal de esta línea se denomina nudo embrionario (de
Hansen) y aparece como una depresión ligeramente elevada. (Sin referenciar).

15)Una mujer de 23 años de edad, con un ciclo menstrual natural está


llegando a la ovulación. El ovocito de un folículo maduro se induce a
someterse a la primera división meiótica como resultado de ¿cuál de los
siguientes estímulos hormonales?
a) El cese de la secreción de progesterona
b) La elevación gradual de la hormona folículo-estimulante (FSH)
c) Los títulos bajos de estrógeno asociados con la maduración del folículo
d) La elevación lenta de la progesterona producida por las células lúteas
e) El aumento de la hormona luteinizante (LH) iniciado por títulos altos de
estrógeno

La respuesta es E. Los ovocitos primarios se han desarrollado para el momento


del nacimiento. Desde la pubertad hasta la menopausia, estas células germinales
permanecen suspendidas en la profase meiótica I. Una oleada de LH a mitad del
ciclo provoca la reanudación de la meiosis y hace que la FSH provoque que el
folículo se rompa y descargue el óvulo. Bajo la influencia de la LH, el folículo roto
se transforma en un cuerpo lúteo, que produce progesterona. FSH y LH de la
adenohipófisis producen el crecimiento y la maduración del folículo ovárico. Bajo la
estimulación de la FSH, las células de la teca proliferan, se hipertrofian, y
comienzan a producir estrógenos. (Sin referenciar).

16)La determinación de la línea media dorsal del cuerpo del embrión se da a


partir de la formación de:
a) Línea primitiva
b) Mesodermo intra-embrionario
c) Notocorda
d) Nudo primitivo
e) Membrana buco-faríngea

Respuesta C. (Langman 11° edición capítulo 5) La línea primitiva es el primer


signo de la gastrulación. Se trata de una zona densa situada en la región central
posterior del disco embrionario, su formación se debe a la migración y proliferación
de células del epiblasto hacia el plano medial del disco embrionario. Durante el
proceso de gastrulación la notocorda se origina de las células mesodérmicas
cilíndricas, extendiéndose a lo largo de la línea media. La notocorda induce a que
las células del ectodermo que las recubre se diferencien en células neuronales
precursoras, las cuales se organizan en una estructura llamada placa neural. El
proceso que da origen a esta placa se llama neurulación.

17)La principal función del factor de crecimiento nodal es:


a) Diferenciación celular de la notocorda
b) Mediar la formación de la línea primitiva
c) Mantener la polaridad neutra del nudo primitivo
d) Permitir la migración de células hipoblásticas
e) Potenciar la secreción de proteínas como la bmp-4 en las células del nudo
primitivo

Respuesta C. (Langman 11° edición capítulo 5) El eje anteroposterior lo


determinan las células de la parte anterior del disco embrionario. Esta área,
denominada endodermo visceral anterior (AVE), expresa genes esenciales para la
formación de la cabeza, incluidos los factores secretados cerberus y lefty, que
inhiben la actividad nodal en el extremo craneal del embrión. Estos genes
establecen el extremo craneal del embrión antes de la gastrulación. La misma
línea primitiva se empieza a formar y se mantiene gracias a la expresión de nodal,
un miembro de la familia del factor de transformación del crecimiento. Una vez
formada la línea primitiva, nodal regula el número de genes responsables de la
formación del mesodermo ventral y dorsal, y de las estructuras de la cola y la
cabeza.

18)El establecimiento del eje céfalo-caudal del cuerpo del embrión se da


principalmente por acción de la proteína:
a) Bmp-4
b) Goosecoid
c) Activina
d) Cordina
e) Sonic hedgehog

Respuesta C, (Langman 11° edición capítulo 5). La línea primitiva se forma en la


región medial del disco; las células de ambos lados se unen y forman la estría
primitiva. Este crecimiento es inducido por la activina (De la familia del TGF- b);
el Nodal de la familia TGF- b está involucrado en la formación y desarrollo de la
línea primitiva. La estría se alarga por adición de células hacia el extremo cefálico
hasta formar el nodo primitivo; al mismo tiempo se forma el surco primitivo con una
depresión llamada la Fóvea primitiva, resultado de la invaginación de células del
epiblasto. Con el origen de la línea primitiva se define el eje cráneo-caudal.

19)La activina es una proteína perteneciente a la familia TGF-b que se


produce principalmente a nivel de:
a) Zona cefal del pre-embrión
b) Zona marginal posterior del pre-embrión
c) Zona caudal del pre-embrión
d) Zona marginal anterior del pre-embrión
e) Todas las anteriores son ciertas

Respuesta B, (Langman 11° edición capítulo 5) La formación del epiblasto e


hipoblasto define el dorso-ventral, también indicado por las células que se
encuentran en el margen posterior del disco embrionario que ocupan un área
llamada zona marginal posterior (PMZ) primitiva, en estas células es donde se
secreta principalmente la molécula "activina", la cual a su vez induce la formación
de la línea que induce la formación de la línea primitiva. Así queda establecido el
eje caudal del embrión.

20)El establecimiento del eje corporal izquierdo-derecho en el embrión se da


a partir de la acción del factor:
a) Brachyury
b) Sonic hedgehog
c) Bmp-4
d) FGF
e) Goosecoid

Respuesta B. (Langman 11° edición capítulo 5) La lateralidad derecha izquierda,


que también se establece en las primeras etapas del desarrollo, viene
determinada por una cascada de genes. Cuando aparece la línea primitiva, las
células del nódulo primitivo y de la misma línea primitiva secretan el factor de
crecimiento fibroblastico que induce la expresión de nodal, pero solo en el lado
izquierdo del embrión. Más tarde cuando se induce la placa neural, FGF-8
mantiene la expresión de nodal en el mesodermo lateral, así como la de LEFTY-2.
Estos dos genes regulan la lateralidad. Sonic hedgehog, también desempeña esta
función y, además, evita que los genes del lado izquierdo se expresen en el lado
derecho.

21)De la primera diferenciación mesodérmica se deriva:


a) Mesodermo pluripotencial, mesodermo intermedio, mesodermo paraxil
b) Mesodermo pluripotencial, mesodermo cordal, mesodermo cardiogénico
c) Mesodermo pluripotencial, mesodermo esplácnico, mesodermo somítico
d) Mesodermo pluripotencial, mesodermo cordal, mesodermo paraxial
e) Mesodermo pluripotencial, mesodermo cardiogénico, mesodermo
esplácnico

Respuesta B, (Langman 11° edición capítulo 6) El logro de la migración e


invaginación del mesodermo hacia su posición definitiva depende de la formación
de tres estructuras: la línea primitiva, el nodo de primitivo y la placa precordal. Las
tres tienen, a través de sus productos moleculares, una función regulatoria sobre
la invaginación del mesodermo y su diferenciación inicial en: mesodermo cordal o
notocorda, mesodermo cardiogénico y mesodermo lateral pluripotencial. Este
último se subdivide en mesodermo paraxial, mesodermo lateral definitivo y
mesodermo intermedio.

22)El ectodermo epitelial origina principalmente:


a) Tubo neural
b) Dermis de la cabeza
c) Epidermis
d) Dermis del cuerpo
e) Crestas neurales

Respuesta C. (Langman 11° edición capítulo 6) Las células que cubren


el embrión tras la neurulación, la formación del sistema nervioso central. En la
mayoría de los vertebrados, esta estructura monocapa original se transforma
rápidamente en un tejido compuesto por dos capas: una capa exterior temporal,
denominada peridermo, que degenera una vez la capa basal interior (stratum
germinativum) se ha formado.

23)El mesodermo lateral definitivo somático interactúa principalmente con:


a) Ectodermo neural
b) Ectodermo epitelial
c) Endodermo intra-embrionario
d) Mesodermo lateral definitivo esplácnico
e) Ectodermo lateral

Respuesta B, (Langman 11° edición capítulo 6) El ectodermo epitelial se


diferencia a partir del ectodermo neural por el proceso de inducción primaria
regulado por la notocorda y por el ambiente molecular que genera la interacción
con el mesodermo lateral pluripotencial. Al igual que la diferenciación del
ectodermo neural, el ectodermo epitelial interacciona con células mesodérmicas
en forma unidireccional meso - ectodermo a través de productos celulares
difusibles que se ligan a receptores membranales para activar procesos de
transducción de señales que regulan finalmente la activación de los genes que
codifican para proteínas de citoesqueleto y membrana de las células epidérmicas.

24)Son estructuras que se derivan del endodermo intraembrionario, excepto:


a) Epitelio de revestimiento del tubo digestivo
b) Epitelio de revestimiento de vías respiratorias
c) Musculatura lisa del tracto digestivo
d) Glándulas anexas del sistema respiratorio
e) Revestimiento del intestino primitivo

Respuesta C, (Langman 11° edición capítulo 14) El endodermo forma el


revestimiento epitelial del tubo digestivo y origina las células específicas (el
parénquima) de las glándulas, como los hepatocitos y las células exocrinas y
endocrinas del páncreas. El estroma (tejido conjuntivo) de las glándulas deriva del
mesodermo visceral. El musculo liso, el tejido conjuntivo y los componentes
perioneales de la pared del intestino también derivan del mesodermo visceral.
25) Son estructuras derivadas del ectodermo, excepto:
a) Piel
b) Epitelio sensorial del oído
c) Glándula hipófisis posterior
d) Bazo

Respuesta D, (Langman 11° edición capítulo 6) La capa germinal ectodérmica


origina los órganos y las estructuras que establecen contacto con el mundo
exterior: El sistema nervioso central y periférico, el epitelio sensitivo de la oreja
nariz y los ojos, la piel incluida uñas y el pelo, la hipófisis, la glándula mamaria y
sudorípara, y el esmalte de los dientes.

26)Son estructuras derivadas del mesodermo, excepto:


a) Porción cortical de la glándula adrenal
b) Riñones
c) Gónadas
d) Hígado
e) Músculo liso y estriado

Respuesta D, (Langman 11° edición capítulo 6) Del mesodermo paraxial se


originan los somitomeros, que originan el mesénquima de la cabeza y se
organizan en somitas en los segmentos caudales y occipitales. Los somitas
originan el miotoma (tejido muscular), esclerotoma (cartílago y hueso), y el
dermatoma (tejido subcutáneo de la piel). La parte dorsal del somita se convierte
en dermis bajo la influencia de neurotrofina 3; Así mismo el mesodermo origina el
sistema vascular( es decir, el corazón, las arterias, las venas, los vasos linfáticos y
todas las células sanguíneas y linfáticas); y por ultimo también derivan el bazo y la
corteza de las glándulas suprarrenales. Por otra parte el hígado hace parte de los
derivados del endodermo.

27) La formación de la epidermis se da principalmente por acción de:


a) Cadherinas E
b) Cadherinas N
c) Cadherinas P
d) Cadherinas CAM
e) Caspasas

Respuesta A, (Langman 11° edición capítulo 6) Mediante la neurulación primaria


se divide el ectodermo en tres grupos celulares correspondientes a la epidermis en
su parte más externa, el tubo neural internamente y a las células de la cresta
neural, que se forman en la región ubicada entre las dos anteriores. El primer paso
hacia la neurulación es el engrosamiento de dos tercios dorsales del ectodermo,
pasando de ser una región de epitelio cuboide a uno cilíndrico más alto, lo que es
denominado placa neural, proceso controlado e inducido desde más abajo por un
tubo de mesodermo propio de los vertebrados, la notocorda, que
segrega cordina , noggina y folistatina (en anfibios), las cuales inhiben las demás
proteínas del ectodermo BMP's (formadoras de piel y hueso), de modo que el
ectodermo de esa región deja de ser futura piel o cartílago para convertirse
en tejido neural. El tamaño de la región y sus plegamientos futuros también son
controlados por ella, mediante la proteína Sonic hedgehog. Posterior a la
formación de la placa neural se da el engrosamiento y la migración hacia arriba de
los bordes de la placa neural; esta migración forma los pliegues neurales con el
aparecimiento del surco neural en forma de U en el centro de la placa. Los
pliegues neurales posteriormente migrarán hacia el centro del embrión y
fusionandose para formar el tubo neural bajo el ectodemo que lo recubre; las
células de la porción dorsal del tubo neural formaran la cresta neural. El cierre del
tubo neural está regulado por Cadherinas; las células que llegaran a ser tubo
neural, inicialmente, expresan Cadherina-E, al momento de culminarse la
formación del tubo neural se detiene la expresión de Cadherina-E y se comienza a
expresar Cadherina-N y N-CAM las cuales hacen que no se adhieran los tejidos
del ectodermo superficial y el tubo neural.

28)Son estructuras derivadas del endodermo, excepto:


a) Tiroides
b) Paratiroides
c) Páncreas
d) Glándula mamaria
e) Timo

Respuesta D, (Langman 11° edición capítulo 6) La capa germinal endodérmica


proporciona el revestimiento epitelial al tubo gastrointestinal, el aparato respiratorio
y la vejiga urinaria. También forma el parénquima de las glándulas tiroidea y
paratiroides, el hígado y el páncreas; Por último el revestimiento epitelial de la
cavidad timpánica y el conducto auditivo también se forman de esta capa
germinal.

29)Son objetivos básicos del desarrollo intrauterino de la segunda semana,


excepto:
a) Formación del disco bilaminar con un componente endodérmico y otro ecto-
mesodérmico
b) Formación de la cavidad amniótica a partir de una proliferación de las
células del mesodermo extra-embrionario
c) Completar el proceso de implantación a partir de una regulación de
impronta de género en las células del trofoblasto polar
d) Formación del saco vitelino a partir de una proliferación de células
endodérmicas
e) Iniciar el proceso de nidación a partir de la formación de proteínas
desiduales

La respuesta es B. Segunda semana, finalización de la implantación y


continuación con el desarrollo embrionario. El trofoblasto: sincitiotrofoblasto y
citotrofoblasto invaden el tejido conjuntivo endometrial hasta formar las
vellosidades coriónicas. Mientras el embrioblasto se orfaniza en dos capas el
epiblasto y el hipoblasto formando la cavidad amniótica y el saco vitelino. El
mesodermo extraembrionario formará la cavidad coriónica que contiene al
embrión. Keith L. Moore & T.V.N. Persaud. (2007). Embriología clínica: El
desarrollo del ser humano: Segunda semana. (7°.). (pp. 44-57). Barcelona,
España: Elsevier.

30)La región dorso-lateral del cuerpo se forma principalmente por derivados


de la interacción:
a) Mesodermo lateral esplácnico-mesodermo lateral somático
b) Ectodermo epitelial-mesodermo lateral esplácnico
c) Ectodermo epitelial-mesodermo lateral somático
d) Ectodermo epitelial-dermatomas de somitas dorsales
e) Ectodermo epitelial-mesodermo lateral pluripotencial

Respuesta: D. En la neurulación, cuándo el embrión es llamado Néurula, se


produce la fusión de los pliegues neurales al final de la tercera semana, que forma
la placa neural, tubo neural y las crestas neurales. Al separarse el tubo neural del
ectodermo superficial este último se hace continuo a lo largo de la espalda del
embrión y se diferencia en epidermis. El mesodermo intraembronario prolifera a
cada lado de la notocorda y el tubo neural, forma el mesodermo paraxial, luego
intermedio hasta constituir el mesodermo lateral. El mesodermo paraxial se
diferencia en somitas en dirección cráneo caudal, que dan lugar a la mayor parte
del esqueleto axial y musculatura asociada, así como a la dermis adyacente a la
piel. Keith L. Moore & T.V.N. Persaud. (2007). Embriología clínica: El desarrollo
del ser humano. (7°.). (pp. 67-70). Barcelona, España: Elsevier.

SEGUNDO MES DE GESTACIÓN


1. En el desarrollo del embrión/feto humano, la mayoría de los órganos
internos comienzan a formarse ¿en qué mes?
a. Primero
b. Segundo
c. Cuarto
d. Sexto
e. Noveno

La respuesta es b. La formación de la mayoría de los órganos internos se


produce durante el segundo mes, el período de organogénesis. El primer mes
(opción a) del desarrollo embrionario en general se refiere a la escisión, la
formación de las capas germinales, y el establecimiento del cuerpo embrionario. El
período de la novena semana hasta el final de la vida intrauterina (respuestas c, d
y e), conocido como el período fetal, se caracteriza por la maduración de los
tejidos y el crecimiento rápido del cuerpo fetal. (Sin referenciar).

2. Un obstetra practica una ecografía a una mujer de 25 años que quedó


en embarazo luego de un aborto involuntario. El médico detecta
movimiento fetal. ¿A cuál de las siguientes edades ocurre el primer
movimiento fetal?
a. Primer mes.
b. Segundo mes.
c. Cuarto mes.
d. Sexto mes.
e. Séptimo mes.

La respuesta es B. El desarrollo neuromuscular es suficiente para permitir


movimiento fetal en la octava semana del desarrollo. Otras características de la
octava semana incluyen primera aparición de una piel delgada, la cabeza del
mismo largo del resto del cuerpo, formación de lo que serán los ojos, aparición de
los dedos en manos y pies, aparición de testículos y ovarios (pero sin genitales
externos distinguibles), y una longitud cráneo caudal de 30mm. Para el final de la
octava semana, casi todas las estructuras adultas han al menos empezado a
desarrollarse. (Sin referenciar).

3. Una mujer de 27 años de edad da a luz a una bebe a término. El


patólogo examina la placenta luego del nacimiento. ¿Cuál de los
siguientes componentes placentarios es derivado de la madre más
que del feto?
a. Corion.
b. Citotrofoblasto.
c. Red lacunar.
d. Vellosidades primarias.
e. Sincitrotrofoblasto.

La respuesta es C. La placenta se deriva de ambos tejidos, tanto materno como


fetal. La madre produce el lecho capilar que forma la red lacunar de la placenta. A,
B, E son incorrectas el feto produce el citotrofoblasto y el sincitotrofoblasto, que
juntos forman el corión. Las proyecciones tempranas de ello se denominan
Vellosidades primarias, D es incorrecta. (Sin referenciar).

4. ¿Cuál de los siguientes términos es el que mejor define una


combinación de un defecto primario, como agenesia bilateral renal,
con su cambio estructural secundario?
a. Asociación
b. Deformación
c. Disrupción
d. Secuencia
e. Síndrome

La respuesta es D. (Cotran, pp 464–466. Jorde, pp 319–331.) Hay varios


términos clínicos similares que se usan para describir diferentes tipos de
anormalidades del desarrollo físico. Una secuencia es un patrón reconocido como
resultado de una única anormalidad preexistente (ejemplo: secuencia de
oligohidroamnios) (Opciones A, B, C y E). Una asociación es un patrón no
aleatorio de anomalías con un mecanismo desconocido por ejemplo la asociación
VATER. Una deformación es la alteración de una parte del cuerpo, normalmente
formada por fuerzas mecánicas. Una disrupción es un defecto como resultado de
la interferencia en un proceso normal de desarrollo. La disrafia se refiere a
defectos causados por la falla en la fusión de estructuras adosadas (ejemplo:
espina bífida). La distopía es la retención de un órgano en un sitio durante el
desarrollo (Por ejemplo, los testículos distópicos que permanecen en el canal
inguinal se denominan criptorquidia). Una malformación es un defecto causado por
un proceso de desarrollo intrínsecamente anormal (como el labio fisurado o la
polidactilia). Síndrome se refiere a anormalidades múltiples con un patrón
reconocido y patogénesis identificada (Síndrome de Down).

5. Una higienista dental está preocupada por los efectos de la radiación


en el desarrollo en el útero de su bebé. ¿Durante cuál de los
siguientes periodos es el embrión más susceptible a las influencias
ambientales que pueden inducir la formación de malformaciones
congénitas letales?
a. Fertilización y 1 semana de vida fetal
b. La segunda semana de vida fetal
c. De la tercera a la octava semanas de vida fetal
d. El tercer mes de vida fetal
e. El tercer trimestre de la vida fetal

La respuesta es c. La exposición del embrión a factores nocivos ambientales


(teratógenos), tales como productos químicos, virus, y / o radiación, puede ocurrir
en cualquier momento. Durante la tercera a octava semanas de vida embrionaria,
aparatos orgánicos se desarrollan y son más susceptibles a los teratógenos.
Durante ese tiempo, cada sistema de órganos tiene su propio período específico
de la susceptibilidad de pico. La exposición del embrión a teratógenos durante las
primeras 2 semanas de vida fetal (respuestas A y B) induce generalmente aborto
espontáneo y es, por lo tanto, letal. Después de la octava semana del desarrollo
intrauterino (respuestas D y E), la exposición teratogénico generalmente resulta en
retraso de crecimiento de los órganos y no en nuevos cambios estructurales o
funcionales. (Sin referenciar).

APARATO CARDIOVASCULAR

1) En la circulación fetal la sangre con mayor saturación de oxígeno se


encuentra en:
a) Aorta abdominal.
b) Arteria umbilical.
c) Tronco pulmonar.
d) Vena umbilical.
e) Ventrículo izquierdo.

La respuesta es D. La sangre oxigenada de la vena umbilical llega a la vena cava


inferior mezclándose con sangre proveniente de la vena porta, lo cual disminuye
aún más la saturación de oxígeno, en la aurícula derecha parte de la sangre que
proviene de la Vena cava inferior se mezcla nuevamente con sangre poco
oxigenada procedente de la vena cava superior, pero la mayoría pasa a la aurícula
izquierda a través del foramen oval. De allí es bombeada a la aorta, de modo que
la sangre más oxigenada que llega al corazón se distribuye a este y al cerebro. La
saturación de oxígeno máxima de la sangre fetal se encuentra alrededor del 80%.
(Sin referenciar).

2) Un hombre de 78 años se queja de respiraciones muy cortas y mareo. El


examen físico muestra distensión en las venas del cuello y edema
periférico. La auscultación cardíaca revela un soplo mesosistólico y
desdoblamiento prolongado del segundo ruido en el foco pulmonar. La
ecocardiografía revela una gran masa partiendo de la porción lisa del
atrio derecho. Esta región del atrio deriva de cuál de las siguientes
estructuras embrionarias:
a) BulbusCordis.
b) Atrio primitivo.
c) Ventrículo primitivo.
d) Seno venoso.
e) Tronco arterioso.

La respuesta es D. La porción lisa del atrio derecho (Sinus venarum) se deriva


del seno venoso. El seno coronario y la vena oblicua del atrio izquierdo también
derivan del seno venoso. El mixoma auricular derecho es menos común que los
tumores del lado izquierdo pero puede producir síntomas relacionados con la
obstrucción física del atrio derecho. A es incorrecto. El bulbuscordis da lugar a la
porción lisa del ventrículo derecho (cono arterioso) y la porción lisa del ventrículo
izquierdo (vestíbulo aórtico). B es incorrecta el atrio primitivo da lugar a la porción
trabeculada de los ventrículos izquierdo y derecho. C es incorrecta esta estructura
no deriva del ventrículo primitivo, E es incorrecta el tronco arterioso da lugar a la
porción proximal de la aorta y la porción proximal de la arteria pulmonar. (Sin
referenciar).

3) Una mujer de 25 años de edad da a luz a un infante en la semana 38. El


neonato tiene color azulado al nacer, que se resuelve lentamente durante
la siguiente hora. En la unidad de cuidados intensivos neonatales, las
enfermeras han notado que el infante presenta cianótico mientras está
llorando. Eventualmente, desarrollará falla cardiaca congestiva e
hipertensión pulmonar. ¿Cuál de los siguientes defectos congénitos
puede ser el causante de estos hallazgos?
a) Presencia de ductus arterioso.
b) Tronco arterioso persistente.
c) Defecto atrial septal del tipo primum.
d) Defecto atrial septal tipo secundum.
e) Defecto septal ventricular.
La respuesta es B. Un tronco arterioso persistente resulta de una falla en la
formación del septo aorticopulmonar. El septo aorticopulmonar divide el tronco
arterioso en aorta ascendente y el tronco pulmonar. Si hay un fallo en la formación
de éste septo, el mismo tronco arterioso recibirá la sangre tanto del ventrículo
derecho como del izquierdo, permitiendo que la sangre oxigenada se mezcle con
la desoxigenada. La circulación sistémica recibe por lo tanto sangre que no está
completamente oxigenada, causando cianosis. A es incorrecta. La presencia del
ductus arterioso resulta en el paso de la sangre de la aorta al tronco pulmonar
postnatalmente. Esta desviación de izquierda a derecha no causa cianosis. La
cianosis se causa por la desviación de derecha a izquierda. Prenatalmente el
ductus arterioso permite el paso del tronco pulmonar a la aorta. Luego del
nacimiento, cuando el gradiente de presión se revierte y la presión es mayor en el
lado izquierdo que en el derecho del corazón, el flujo en el ductus arterioso del
paciente se revierte generando una desviación de izquierda a derecha. C y D son
incorrectas. Los defectos atriales septales permiten la desviación de sangre de
izquierda a derecha postnatalmente ya que la presión es mayor en el atrio
izquierdo que en el atrio derecho. Solamente las desviaciones que fluyen de
derecha a izquierda causan cianosis. E es incorrecta defectos septales
ventriculares también producen una desviación en sentido izquierda a derecha a
razón del gradiente de presión de izquierda a derecha. Por lo tanto, estos son
defectos no cianóticos. (Sin referenciar).

4) Una mujer de 29 años de edad dio a luz a un bebé sano después de 38


semanas de gestación y el alumbramiento de la placenta intacta
espontáneamente. El embarazo se complicó con preeclampsia, pero el
monitoreo fetal y ecografía fueron normales durante toda la gestación.
Las estructuras predominantes que se muestran en la microfotografía de
acompañamiento de la placenta, ¿se derivan de cuál de las siguientes?
a) Una combinación de tejidos fetales y tejidos maternos
b) Glándulas endometriales
c) Estroma endometrial
d) Tejidos fetales
e) Vasos sanguíneos maternos

La respuesta es D. Las estructuras de la placenta que se muestran en la


microfotografía son vellosidades coriónicas que son derivadas de tejidos fetales.
La contribución de la madre de la placenta (respuestas a y e) es la sangre que
fluye más allá de las vellosidades coriónicas. Un óvulo fertilizado llega al útero
aproximadamente 4 días después de la fertilización. En ese momento, se ha
convertido en una esfera hueca multicelular llamada blastocisto. El blastocisto
pronto se adhiere al endometrio secretor y se diferencia en una masa celular
interna que va a desarrollar el embrión y una capa de trofoblasto primitivo. La
expansión del trofoblasto penetra el endometrio superficial (respuestas b y c) y
erosiona en los vasos sanguíneos maternos. Con el tiempo, desarrolla dos capas,
una interna citotrofoblástica, y una externa sincitiotrofoblastica. Cordones sólidos
de trofoblasto forman las vellosidades coriónicas, que son luego invadidas por
vasos sanguíneos fetales. (Sin referenciar).

5) La sangre fetal de la placenta esta aproximadamente 80% oxigenada. Sin


embargo, se mezcla con la sangre no oxigenada en varios puntos lo cual
reduce el contenido de oxígeno. ¿Cuál de los siguientes vasos fetales
contiene la sangre con el más alto contenido de oxígeno?
a) Aorta Abdominal
b) Arteria Carótida común
c) Ducto arterioso
d) Arteria pulmonar
e) Vena pulmonar

La respuesta es b. La sangre de la placenta en el cordón umbilical esta


aproximadamente 80% oxigenada. La mezcla de sangre no oxigenada de las
venas vitelinas y la vena cava inferior reduce algo el contenido de oxígeno. Sin
embargo, esta corriente con contenido relativamente alto de oxígeno es dirigida
por la válvula de la vena cava inferior directamente a través del foramen oval hacia
la aurícula derecha. Esto evita mezcla con la sangre sin oxígeno que entra en la
aurícula derecha de la vena cava superior. Por lo tanto, la sangre saturada de
oxígeno que entra en el ventrículo izquierdo y se bombea en el arco de la aorta,
las arterias subclavias y las arterias carótidas comunes tiene el más alto contenido
de oxígeno. La sangre sin oxígeno de la vena cava superior se dirige hacia el
ventrículo derecho y luego al tronco pulmonar. Aunque una pequeña parte de este
flujo pasa a través de los pulmones (donde cualquier oxígeno residual se extrae
por el tejido del pulmón que no respira), más se desvía en la aorta torácica a
través del ductus arterioso y por lo tanto reduce el contenido de oxígeno de dicho
buque. Esto se produce distal a los orígenes de las arterias carótidas y asegura
que el cerebro en desarrollo rápido tiene el mejor suministro de oxígeno. (Sin
referenciar).

6) Una niña nace aproximadamente 10 semanas antes de tiempo (a las 30


semanas) y pesa 1.710 g. Ella tiene el síndrome de dificultad respiratoria y
es tratada con surfactante exógeno. Ella es intubada por vía endotraqueal
con ventilación mecánica inmediatamente después del nacimiento.
Durante los primeros 4 días después del nacimiento se reducen la
presión del respirador y la fracción de oxígeno inspirado. Cuando inicia el
quinto día después del nacimiento, ella tiene breves desaturaciones que
se vuelven más persistentes. Ella necesita un aumento de la asistencia
respiratoria y oxígeno. Al séptimo día se torna cianótica. Un examen más
detallado, ecocardiograma y radiografías revelan dilatación de la aurícula
izquierda, la arteria pulmonar aumentada, aumento de la vasculatura
pulmonar, y un murmullo como una máquina continua. ¿Cuál de los
siguientes es el diagnóstico más probable?
a) La persistencia del foramen oval
b) La persistencia del Ductus arterioso
c) Defecto en la pared septal ventricular
d) Estenosis pulmonar
e) La coartación de la aorta

La respuesta es B. La presencia de un soplo podría ser indicativo de cualquiera


de las opciones de respuesta. Sin embargo, la presencia de un soplo como una
máquina continua es indicativo de un ductus arterioso permeable (PDA). Por lo
general, como en este caso, el bebé prematuro con PDA no presenta una cianosis
aguda, aunque pueden producirse breves desaturaciones que se vuelven más
persistentes. Los requisitos de ventilación se incrementan debido al aumento de la
concentración de CO2 (como los pulmones se convierten en "húmedo", el CO2
aumenta). La presión arterial diastólica generalmente cae y hay una presión de
pulso ensanchada (generalmente mayor que 20). El PDA siempre estuvo ahí, es
sólo que su resistencia vascular pulmonar se relajó lo suficiente como para
permitir que más de izquierda a derecha la derivación y un mayor flujo de sangre a
los pulmones (menos para el cuerpo). Un defecto septal auricular (ASD), como por
ejemplo un foramen oval persistente, podría ser eliminado del diagnóstico, ya que
el murmullo se escuchó como una división anormal del segundo sonido durante la
espiración (respuesta a). Un foramen oval permeable es común encontrarlo en los
bebés prematuros y por lo general no es objeto de seguimiento a menos que
aparezca notable al cardiólogo pediatra o hay un rumor persistente. Un foramen
oval permeable puede dar lugar a cianosis sólo mínima o intermitente durante el
llanto o hacer esfuerzos para defecar. Un murmullo causado por un defecto septal
ventricular (VSD, la respuesta c), se produce entre el primer y segundo ruidos
cardíacos (S1 y S2) y se describe como holosistólico (pansistólico) porque la
amplitud es alta en toda la sístole. La estenosis pulmonar se escucha como un
soplo de eyección sistólica severa (respuesta d). La coartación de la aorta (la
respuesta e) daría lugar a un soplo sistólico. PDA se refiere al mantenimiento del
conducto arterioso, una estructura normal del feto. En el feto, el conducto arterial
evita la circulación pulmonar, ya que los pulmones no están involucrados en el
intercambio de CO2 / O2 hasta después del nacimiento. La placenta realiza la
función de intercambio de gases durante el desarrollo fetal. Las derivaciones
conducto arterial persistente en el flujo de la arteria pulmonar izquierda a la aorta.
Los niveles altos de oxígeno después del nacimiento y la ausencia de
prostaglandinas de la placenta hacen que el conducto arterioso se cierre en la
mayoría de los casos dentro de 24 horas. Un PDA con mayor frecuencia se corrige
solo dentro de varios meses después del nacimiento, pero puede requerir la
infusión de indometacina (un inhibidor de prostaglandina) como tratamiento, la
inserción de tapones quirúrgicos durante el cateterismo, o ligadura quirúrgica real.
(Sin referenciar).

7) ¿Cuál de los siguientes procesos coloca el corazón en desarrollo en la


región torácica, craneal al septo transverso?
a) Gastrulación
b) Plegamiento lateral
c) Plegamiento craneal
d) Neurulación
e) Fusión de los tubos endocárdicos corazón

La respuesta es c. El plegamiento craneal es responsable de la colocación del


corazón en desarrollo en la región torácica de presunción del embrión.
Inicialmente, la porción craneal en desarrollo del tubo se encuentra neural dorsal y
caudal a la membrana orofaríngea. Sin embargo, el crecimiento excesivo del
cerebro anterior hace que se extienda más allá de la membrana orofaríngea y
sobresalga del área cardiogénica. El crecimiento posterior del cerebro anterior
empuja el corazón en desarrollo ventralmente y caudalmente a una posición en la
presunta región torácica caudal a la membrana orofaríngea y craneal al septo
transverso que formará el tendón central del diafragma. La gastrulación (respuesta
a) es el proceso por el cual las células del epiblasto migran hacia la línea primitiva
y se convierten en internas para formar las capas germinales mesodérmicas y
endodermo. Plegado lateral (respuesta b) del embrión se forma el tubo y
endodermo que rodea las capas concéntricas del mesodermo y ectodermo.
Neurulación se refiere a la formación del tubo neural del ectodermo de superficie
(respuesta d). La fusión de los dos tubos endocárdicos del corazón (respuesta e)
se produce como se produce el plegado lateral. El tubo fundido formará el
endocardio rodeado por el miocardio primordial derivado del mesodermo
esplácnico que formará el músculo cardíaco (miocardio). (Sin referenciar).

8) ¿Cuál de los siguientes está en contacto directo con la sangre materna en


las lagunas de la placenta?
a) Las células del citotrofoblasto
b) Mesodermo Extraembrionario
c) Los vasos sanguíneos fetales
d) Las células del sincitiotrofoblasto
e) Células amnióticas

La respuesta es d. En el feto en desarrollo, la sangre materna está en contacto


directo con el sincitiotrofoblasto. Durante la implantación, el sincitiotrofoblasto
invade el endometrio y erosiona los vasos sanguíneos maternos. La sangre
materna y secreciones glandulares de nutrientes llenan las lagunas y se bañan las
proyecciones de sincitiotrofoblasto. Las vellosidades primarias consisten en
sincitiotrofoblasto con un núcleo de células citotrofoblastos. En las vellosidades
secundarias, el núcleo citotrofoblastico es invadido por mesodermo y
posteriormente por los vasos sanguíneos umbilicales en vellosidades terciarias.
(Sin referenciar).

9) ¿Cuál de los siguientes procesos es responsable de la fusión de las dos


aortas dorsales?
a) Plegamiento lateral
b) Plegamiento craneocaudal
c) Enrollamiento del tubo del corazón
d) Neurulación
e) Gastrulación

La respuesta es a. La fusión de las aortas dorsales se produce a través del


plegado lateral. Fusión del tubo endocárdico del corazón y la incorporación del
saco vitelino en el intestino primitivo también se produce como resultado del
plegado lateral. Plegamiento craneocaudal (respuesta b) establece las regiones de
la cabeza y de la cola definitivos del embrión. La fusión es ya completa en el
momento que el enrollamiento del tubo cardíaco ocurre (la respuesta c). La
gastrulación (respuesta d) establece las tres capas germinales (disco trilaminar), y
neurulacion establece el surco neural con dos pliegues neurales. Neurulación es la
formación del tubo neural (respuesta e). (Sin referenciar).

10)Un hombre de 56 años de edad, desarrolla un aneurisma aórtico en el


origen de una arteria que emerge de la aorta a nivel de la vértebra T12.
¿Cuál de las siguientes estructuras no es un derivado del endodermo
pero recibe irrigación de esta arteria?
a) Ampolla duodenal.
b) Páncreas exocrino.
c) Vesícula biliar.
d) Hígado.
e) Bazo.
La respuesta es E. Los derivados del endodermo del intestino anterior, suplidos
por el tronco celiaco incluyen al esófago, estómago, la primera parte del duodeno
(la ampolla), el páncreas exocrino, la vesícula biliar y el hígado. El tronco celiaco,
que se ramifica a nivel de T12, da una rama esplénica que suple al bazo, el cual
no es un derivado del endodermo del intestino anterior puesto que se desarrolla a
partir de una masa de células mesenquimales (sin epitelio intestinal), localizadas
entre las capas del mesogastrio dorsal. A, B, C, D son incorrectas pues si son
derivados del endodermo. (Sin referenciar).

A continuación se presenta una tabla que muestra las estructuras del adulto
derivadas de cada una de las tres divisiones del tubo intestinal primitivo.

Intestino anterior Intestino medio Intestino posterior


(Tronco Celiaco) (arteria mesentérica (arteria mesentérica
superior) inferior)

Esófago Duodeno 2ª, 3ª y 4ª Cólon transverso


porción (tercio distal)

Estómago Yeyuno Cólon descendente

Duodeno 1ª y 2ª Íleon Cólonsigmoide.


porción

Hígado Ciego Recto

Páncreas Apéndice Canal Anal (parte


superior)

Aparato biliar Cólon ascendente

Vesícula biliar Cólon transverso (2/3


proximales

Bolsas faríngeas*

Pulmones*

Tiroides*
*Estos son derivados del intestino primitivo pero no forman parte del tracto
gastrointestinal per se.

11)Una mujer de 27 años, con un antecedente de 15 años de evolución de


Diabetes Mellitus tipo 1, se encuentra embarazada. Durante su embarazo,
el control de su glucemia fue difícil. Da a luz a un bebe que fallece
durante su primera semana de vida. ¿Cuál de los siguientes diagnóstico
es el más probable?
a) Comunicación interauricular
b) Coartación de la aorta
c) Síndrome de Eisenmenger
d) Tetralogía de Fallot
e) Trasposición de grandes vasos

La respuesta es E. La diabetes materna es mejor conocida por ser la mayor


causa de inmadurez en los bebes. También hay una relación específica entre la
diabetes materna y la transposición de los grandes vasos. La transposición es
causada, comúnmente, por la persistencia anormal del cono subaórtico y una
reabsorción del infundíbulo muscular, adaptándose la aorta al ventrículo derecho
durante el desarrollo. En la transposición de los grandes vasos, la aorta nace en la
parte anterior del ventrículo derecho y el tronco pulmonar nace en la parte
posterior del ventrículo izquierdo. Esto produce una completa separación de la
circulación pulmonar. Sin la corrección quirúrgica, la mayoría de los infantes
mueren en sus primeros meses de vida, aunque el conducto arterioso se
encuentre permeable. (Sin referenciar).

12)Un hombre de 45 años de edad se queja de disnea de esfuerzo y


palpitaciones. Él ha sido diagnosticado con un defecto septal atrial. ¿Cuál
es la causa más común de este tipo de malformación cardiaca congénita?
a) Fallo de la formación del septum primum.
b) Fallo de la formación de la septum secundum.
c) Adhesión incompleta entre el septum primum y septum secundum.
d) Malformación del tabique interventricular membranosa.
e) Malformación del tabique interventricular muscular.

La respuesta es C. La forma más común de defecto septal auricular se encuentra


cerca del foramen oval (que no debe confundirse con un foramen oval permeable,
que es de poca o ninguna significación hemodinámica). Son el resultado de la
adherencia incompleta entre el septum primum y el septum secundum durante el
desarrollo. A y B son incorrectas la comunicación interauricular con menos
frecuencia se presentan como resultado de la falla de formación del septum
primum y septum secundum. D y E son incorrectas malformaciones del tabique
interventricular causa defectos del tabique ventricular en lugar de defectos del
tabique auricular. (Sin referenciar).

13)¿De cuál de los siguientes vasos fetales surge la arteria umbilical?


a) Aorta.
b) Arterias carótidas.
c) Ductus arterioso.
d) Arterias iliacas internas.
e) Arterias pulmonares.

La respuesta es D. El par de arterias umbilicales surge de las arterias iliacas


internas. Ellas suministran sangre fetal desoxigenada a la placenta. La vena
umbilical toma la sangre proveniente de la placenta nueva, limpia y oxigenada y
la lleva a la circulación fetal, A, B, C y E, la aorta, las arterias carótidas y las
pulmonares no originan las arterias umbilicales y hacen parte de circulación fetal y
transportan sangre oxigenada con la salvedad de que la arteria pulmonar izquierda
se conecta con la aorta por el ducto arterioso. (Sin referenciar).

14)El examen físico de un niño de 6 años revela un soplo cardiaco. Un


electrocardiograma revela un tipo de defecto atrial septal, ostium primum.
Éste defecto resulta del fallo de:
a) El ostium primum para formarse a partir del septum primum.
b) El ostum secundum para formarse a partir del septum primum
c) El septum primum para fusionarse con las almohadillas endocardicas
d) El septum primum para fusionarse con el septum secundum
e) El septum secundum para fusionarse con las almohadillas endocardicas.

La respuesta es C. El septum primum (primer septo interatrial) se desarrolla por


el crecimiento desde el extremo craneal del atrio embrionario hacia las
almohadillas endocardicas. El vínculo que existe entre los dos atrios es el ostium
primum. A medida que el septum primum continúa su crecimiento, el ostium
primum se vuelve más pequeño. Cuando el septum primum concluye su
crecimiento y antes de una fusión completa con las almohadillas endocardicas
ocurre un proceso de muerte celular en septum primun y se forma el ostium
secundum. Un fallo en la fusión completa del septum primum con las almohadillas
endocardicas deja un ostium primum persistente, un tipo de defecto atrial septal. A
es incorrecta el ostium primum no se forma solo a partir del septum primum. El
ostium primum es la comunicación entre los dos atrios que existe durante la
formación del septum primum. Eso es, el ostium primum es el espacio entre los
atrios en desarrollo que aún no está ocupado por el septum primum. B es
incorrecta el ostium secundum normalmente se forma a partir del septum primum
antes de que el ostium primum se cierre por la fusión del septum primum con las
almohadillas endocardicas. Un fallo en la formación del ostium secundum
resultaría en la muerte del embrión, ya que no habría vía para el paso de la sangre
del atrio derecho al izquierdo cuando el ostium primum se cierre, deprivando así
de sangre oxigenada al embrión. La mayoría del septum primum normalmente
desaparece. El remanente forma la valva del foramen oval. D es incorrecta esta
parte del septum primum normalmente no se fusiona con el septum secundum
durante la vida prenatal. Luego del nacimiento, la valva del foramen oval es
empujada a través del septum secundum como resultado de la presión creciente
en el atrio izquierdo. Esto logra un cierre funcional del foramen oval. La fusión no
ocurre normalmente en este momento, pero usualmente ocurre luego en la vida de
la mayoría de las personas. En algunas personas, sin embargo, la fusión completa
nunca ocurre (sonda de permeabilidad). E es incorrecta el septum secundum no
se fusiona con las almohadillas endocardicas. (Sin referenciar).

15)Una mujer primigrávida de 36 años de edad con 16 semanas de


gestación, consulta al médico para un examen prenatal de rutina. La
amniocentesis se realiza para el screening genético de su feto. El análisis
cromosómico muestra una microdelección en una región que se cree está
asociada con la migración y el desarrollo de células de la cresta neural. Si
su microdelección produjera alteraciones del desarrollo en el feto. ¿Cuál
de las siguientes estructuras podría estar afectada?
a) Septo aórticopulmonar.
b) Fosa oval.
c) Ligamento arterioso.
d) Músculos papilares.
e) Atrio derecho.

La respuesta correcta es A. Las células de la cresta neural invaden crestas


troncales y bulbares del corazón en desarrollo. Estas crestas crecen en forma de
espiral, formando el tabique aórticopulmonar, que divide el tronco arterioso y el
cordón bulbar entre la aorta y la arteria pulmonar. C es incorrecta el ligamento
arterioso proviene del 6 arco aórtico izquierdo, B es incorrecta la fosa oval
proviene de la abertura entre el septum primun y septum secundum que forman el
tabique interauricular que es permeable en la gestación, D es incorrecta los
músculos papilares provienen de los ventrículos primitivos bien sea del ventrículo
o del bulbo arterial que originan el ventrículo derecho e izquierdo respectivamente,
E es incorrecta la aurícula derecha proviene de la formación del seno venoso,
junto con las estructuras vasculares que son las astas venosas; la izquierda se
degenera y formará el seno coronario y la vena auricular oblicua izquierda, el asta
derecha formara la parte lisa de aurícula derecha, la válvula de la vena cava,
válvula del seno coronario, creta terminal y parte del tabique interauricular. (Sin
referenciar).
16)Durante el desarrollo intrauterino, los siguientes entregan flujo sanguíneo
a la aurícula izquierda:
a) Venas pulmonares.
b) Foramen oval.
c) Vena cardiaca menor (venaecordisminimae).
d) Ductus arterioso.

La respuesta es A y B. Las venas pulmonares entregan flujo, también se entrega


flujo a la aurícula izquierda a través del foramen oval, esta sangre proviene de la
aurícula derecha y es guiada por la válvula de la vena cava inferior, las venas
cardíacas menores no entregan flujo a la aurícula izquierda pero si a la aurícula
derecha y a los 2 ventrículos. A través del ductus arterioso circula sangre desde el
ventrículo derecho hacia a la aorta descendente en su mayoría en donde se
mezcla con sangre de la aorta proximal, esta sangre no alcanza a atravesar las
arterias pulmonares a causa de la alta resistencia de los vasos pulmonares en la
vida intrauterina por lo cual la sangre cruza el ductus. (Sin referenciar).

17)Los cambios circulatorios que ocurren al nacimiento normalmente


incluyen:
a) Elevación de la presión de la aurícula izquierda.
b) Incremento del flujo sanguíneo a través de los pulmones.
c) Disminución de la presión de la aurícula derecha.
d) Cesación del flujo a través del foramen oval.
e) Todas las anteriores.

La respuesta es E. Al nacimiento, Los pulmones se expanden con el incremento


concomitante del flujo sanguíneo pulmonar, así se reduce la presión en la aurícula
derecha y se incrementa la presión en la aurícula izquierda. Esta presión reversa
(entre aurícula izquierda y derecha) causa que el agujero oval se cierre, evitando
un retorno sanguíneo a la aurícula derecha. (Sin referenciar).

18)¿Cuál de las siguientes afirmaciones sobre el ductus arterioso, es


incorrecta?
a) Se extiende desde la arteria pulmonar izquierda a la aorta, justamente distal
al origen de la arteria subclavia izquierda.
b) Se ha reportado que ocurra en el lado derecho.
c) Normalmente se cierra a las 16 horas de vida postnatal.
d) La persistencia de permeabilidad es más común en los hombres.

La respuesta es D. A corresponde a la correcta definición del ductus arterioso


clásico, B es correcto; la asociación de arco aórtico a la derecha con persistencia
del conducto arterioso ipsolateral es rara pero se presenta, C es correcto el cierre
funcional se produce horas después del nacimiento (habitualmente, a las 10-72
horas), y el anatómico, 2 o 3 semanas más tarde. D es incorrecto; el ductus
arterioso persistente es 2 – 3 veces más frecuente en mujeres. (Sin referenciar).

19)¿Cuál de las siguientes afirmaciones describen correctamente la


permeabilidad del ductus arterioso?
a) El ductus de un prematuro es más sensible a indometacina que el de un
niño a término.
b) El tratamiento quirúrgico siempre puede esperar.
c) Típicamente se encuentra un soplo continuo que se extiende a la clavícula.
d) La cianosis es distintiva de la entidad.

La respuesta es C. Se escucha un soplo continuo, que abarca todo el sístole y


gran parte del diástole (soplo en maquinaria). Es más intenso hacia el segundo
ruido y hasta lo puede ocultar. Se ausculta en el 2° espacio intercostal izquierdo,
debajo de la clavícula y puede acompañarse de frémito. A es incorrecto; El ductus
de un prematuro es igualmente sensible a indometacina que el de un niño a
término pero se usa como tratamiento profiláctico en prematuros para evitar las
complicaciones, B es incorrecto siempre se debe realizar tratamiento quirúrgico
una vez se han agotado las otras opciones de tratamiento, si la falla cardiaca es
muy grande, se requiere cirugía urgente, D es incorrecto la cianosis no es
distintiva de la entidad podría no presentarse. (Sin referenciar).

20)¿Cuál de las siguientes características anatómicas están incluidas en la


tetralogía de Fallot?
a) Estenosis pulmonar.
b) Hipertrofia ventricular izquierda.
c) Defecto septal ventricular.
d) Aorta Cabalgante que nace directamente por encima de un defecto septal.
e) Todas las anteriores.

La respuesta es E. Todas estas corresponden al cuadro anatómico de la


tetralogía de Fallot. (Sin referenciar).

21)Seleccione las afirmaciones CORRECTAS sobre la tetralogía de Fallot


a) Muchos niños se vuelven sintomáticos después de 3 o 6 meses de vida.
b) Se caracteriza por presencia de cianosis al nacimiento.
c) Se puede revelar en niños con disnea.
d) El shunt de Blalock-Taussig es el procedimiento correctivo definitivo.
La respuesta es A y C. Muchos niños se vuelven sintomáticos los primeros
meses de vida, B es incorrecto aunque se puede presentar lo habitual es que
aparezca semanas después o que no aparezca (Fallot acianótico), C es correcto
los niños con disnea por tetralogía de Fallot adoptan una posición de cuclillas
durante los episodios de cianosis, esta posición es característica y posiblemente
les diagnostique el cuadro, D es incorrecto El shunt de Blalock-Taussig (derivación
estándar de la arteria pulmonar con la subclavia) se realiza cuando el niño tiene
crisis cianóticas y atresia pulmonar significativa o arteria pulmonar hipoplásica
para evitar la muerte o la hemiplejía. A los tres meses postnatales se hace una
cirugía correctiva primaria. (Sin referenciar).

22)¿Cuál de los siguientes es el precursor embrionario de las fibras


remanentes que corren en una fisura sobre la superficie visceral del
hígado?
a) Ductus arterioso.
b) Ductus venoso.
c) Vena hepática portal.
d) Arteria esplácnica lateral.
e) Arteria umbilical izquierda.
f) Vena umbilical izquierda.
g) Ligamento teres.
h) Ligamento venoso.
i) Arteria vitelina.
j) Vena vitelina.

La respuesta es B. La estructura descrita es el ligamento venoso (Opción H)


el cual es derivado del ductus venoso. A es incorrecto el ductus arterioso es
derivado del sexto arco aórtico. Este se oblitera y forma el ligamento arterioso.
C es incorrecto la vena hepática portal es derivada de la vena vitelina
(opción J). D es incorrecto a arteria esplácnica lateral surge de cada costado
de la aorta dorsal. Ella abastece inmediatamente al mesodermo y da lugar a la
arteria renal, suprarrenal, frénica, testicular y ovárica. E es incorrecto la arteria
umbilical da lugar a la arteria iliaca interna proximalmente y al ligamento
umbilical distalmente. G es incorrecto el ligamento teres es derivado de la vena
umbilical izquierda. I es incorrecto la arteria vitelina va a formar la arteria
celiaca y suministra irrigación a la pared anterior, la arteria mesentérica
superior suple a la pared media y arteria mesentérica inferior suple la pared
posterior. (Sin referenciar).

23)La radiografía de tórax realizada en un infante de dos semanas, muestra


aumento del tamaño del ventrículo izquierdo y de la aurícula izquierda,
así como dilatación de la aorta. ¿Cuál de los siguientes diagnósticos, es
el más probable?
a) Comunicación interauricular
b) Conducto arterioso persistente
c) Estenosis pulmonar
d) Tetralogía de Fallot
e) Comunicación interventricular

La respuesta es B. El conducto arterioso persistente (CAP), es un desorden


cardiaco congénito; en el cual la sangre que viaja por la aorta es desviada a través
del ductus arterioso a las arterias pulmonares. En la radiografía; el ventrículo
izquierdo y la aurícula izquierda, deben encontrarse dilatadas, y se debe observar
signos de hipertensión pulmonar. El CAP se caracteriza por la presencia de un
soplo en la auscultación. Si el ductus es ampliamente permeable, se desarrolla
hipertensión pulmonar (típicamente en un periodo de varios años) y el cortocircuito
inicial de izquierda a derecha se revierte, enviando la sangre desoxigenada a
través de la aorta descendiente; por lo anterior, se pueden encontrar pies
cianóticos, pero los dedos generalmente no se encuentran cianóticos. En la
comunicación interauricular (Opción A) pasa la sangre de izquierda a derecha
causando un volumen sobrepasante del ventrículo izquierdo, incrementando la
presión de la válvula pulmonar. También se debe escuchar un soplo diastólico,
que refleja el incremento de presión de la aurícula derecha. La estenosis pulmonar
(Opción C) típicamente, produce una forma de desorden cardiaco congénito
cianótico; caracterizado por: comunicación interventricular, obstrucción del
ventrículo derecho, dilatación de la aorta e hipertrofia del ventrículo derecho. El
corazón usualmente se describe como que tiene “forma de bota” en la radiografía.
La comunicación interventricular (Opción E) permite el paso de sangre,
inicialmente de izquierda a derecha; caracterizado por un soplo holosistólico, y una
vasculatura aumentada en la radiografía. (Sin referenciar).

24)La principal función del factor GATA-4 es:


a) Fusión de tubos cardiacos primitivos
b) Tabicación cameral
c) Tabicación del seno aórtico
d) Formación de los grandes vasos
e) Ninguna de las anteriores

Respuesta es A. Fusión de tubos cardiacos primitivos. Varios miembros de la


familia GATA de factores de transcripción se expresan ya en las crestas
percardiacas y desempeñan un papel primordial en la especificación miocardiaca.
La expresión de GATA4, GATA5 y GATA6 se produce desde los estadios más
tempranos de la formación miocárdica, y al menos GATA4 es imprescindible en
los primeros estadios de gestación dado que su ausencia revierte en la formación
de cardia bífida. (Regulación de la expresión génica en el miocardio durante el
desarrollo cardíaco, Franco; Dominguez; Castro; Aránega. Revista Española de
cardiología).

25) El primer proceso que se lleva a cabo en la morfogénesis cardiaca es:


a) Tabicación cameral
b) Fusión de las placas cardiacas
c) Valvulogénesis
d) Formación de grandes vasos
e) Angiogénesis

Respuesta es B. Fusión de las placas cardiacas. En el mesodermo aparecen


unos islotes sanguíneos que formarán células sanguíneas y vasos mediante un
proceso de vasculogénesis. Con el tiempo dichos islotes o placas, se fusionan y
forman un tubo en forma de herradura revestido de endotelio y rodeado por
mioblastos, estableciendo el llamado campo cardiogénico (embriología de
Langman, pág 165).

26) De los arcos aórticos se puede decir que, excepto:


a) El primer arco aórtico forma la arteria maxilar
b) El segundo arco aórtico forma las arterias carótidas
c) El tercer arco aórtico tiene la misma disposición del lado derecho e
izquierdo
d) El cuarto arco aórtico origina una porción del cayado de la aorta y la
subclavia derecha
e) El sexto arco aórtico origina el conducto arterioso

Respuesta es B. El segundo arco aórtico forma las arterias carótidas. El segundo


arco aórtico forma las arterias hioidea y estapedia. Las arterias carótidas son
formadas por el tercer arco aórtico, la común y la primera parte de la interna,
siendo la carótida externa un brote del tercer arco (Embriología de Langman,
págs. 187-189).

27)La comunicación interventricular se debe principalmente a:


a) Falta de desarrollo de la almohadilla cardiogénica inferior y sus moléculas
reguladoras
b) Falta de desarrollo de la almohadilla cardiogénica superior y sus moléculas
reguladoras
c) Ausencia del desarrollo del mesodermo lateral pluripotencial
d) Ausencia de invaginación del mesodermo cardiogénico por el nudo de
hensen
e) Todas las anteriores

Respuesta: A Las paredes internas de los ventrículos en expansión se acercan y


poco a poco se fusionan, formando de tal manera el tabique interventricular
muscular. En ocasiones, la fusión entre las paredes es incompleta, lo cual se
manifiesta en una hendidura apical más o menos profunda entre los dos
ventrículos. El espacio que queda entre el borde libre del tabique interventricular
muscular y las almohadillas endocárdicas fusionadas permite la comunicación
entre los dos ventrículos. (Sin referenciar).

28)¿Cuál de los siguientes órganos no es un derivado del intestino anterior


pero es irrigado por estructuras vasculares del intestino anterior (tronco
celiaco)?
a) Hígado.
b) Pulmones.
c) Páncreas.
d) Vesícula biliar.
e) Bazo.

La respuesta es E. El bazo es una estructura derivada del mesénquima del


mesenterio dorsal específicamente mesogastrio dorsal y es irrigado por la arteria
esplénica rama del tronco celiaco. A, B, C, D son incorrectas la vesícula biliar, el
páncreas, el hígado y los pulmones son derivados del intestino anterior además el
pulmón tiene como particularidad no ser irrigado por ramas del tronco celiaco sino
por arterias bronquiales y la de la arteria pulmonar. (Sin referenciar).

29)La autopsia de un infante que murió con múltiples anormalidades


congénitas revela una malformación en el corazón. La aorta surge de un
ventrículo derecho morfológico, ubicado en el lado derecho del corazón.
La arteria pulmonar anula un defecto ventricular septal. ¿Cuál de los
siguientes términos describe mejor el corazón del infante?
a) “Patrón común” de transposición de las grandes arterias.
b) “Patrón corregido” de transposición de las grandes arterias.
c) Malformación de Taussig-Bing
d) Tetralogía de Fallot.
e) Tronco arterioso.

La respuesta es C. El corazón de este infante presenta las características de la


Malformación de Taussig-Bing, que es considerada una variante de la
transposición de las grandes arterias, porque la aorta surge del ventrículo derecho
morfológico. A es incorrecta en el “Patrón común” de transposición de las grandes
arterias, la aorta surge de un ventrículo derecho morfológico del lado derecho del
corazón, y la arteria pulmonar surge de un ventrículo izquierdo morfológico
ubicado en el lado izquierdo del corazón. B es incorrecta en el “Patrón corregido”
de transposición de grandes arterias, la aorta surge de un ventrículo derecho
morfológico en el lado izquierdo del corazón, y la arteria pulmonar surge de un
ventrículo izquierdo morfológico en el lado derecho del corazón. D es incorrecto en
la tetralogía de Fallot la aorta se superpone a un defecto septal y la arteria
pulmonar proximal es estenótica además hay hipertrofia ventricular derecha y
comunicación interventricular. E es incorrecta en el tronco arterioso un solo largo
vaso se superpone a un defecto septal. (Sin referenciar).

30)Un investigador estudia los orígenes evolutivos de las arterias que


suministran sangre al cerebro. La investigación consiste en inyectar
medios de contraste en los corazones de los fetos de cerdo y estudiar las
arterias que se desarrollan en los arcos faríngeos, porque los vasos
sanguíneos que se desarrollan de forma bilateral en varias de estas
estructuras suministran al polígono de Willis. ¿Cuál de los siguientes
pares de arcos aórticos da lugar a las arterias que contribuyen a los
vasos sanguíneos en el círculo de Willis?
a) 1 y 2 Arco aórtico.
b) 2 y 3 Arco aórtico.
c) 3 y 4 Arco aórtico.
d) 4 y 5 Arco aórtico.
e) 4 y 6 Arco aórtico.

La respuesta es C. El 3 arco da lugar a la arteria carótida externa e inicio de la


interna bilateralmente y el 4 arco da lugar a gran parte del arco de la aorta y sus
ramas (menos tronco braquiocefálico) y las arterias subclavias derecha e
izquierda. A es incorrecta el 1 arco aórtico da lugar a la arteria maxilar y el
segundo a la arteria estapedial y las hioideas. D es incorrecta el 5 arco aórtico
puede no formarse o ser incompleto de todas maneras este se degenera
tempranamente en el desarrollo fetal. E es incorrecta el 6 arco da lugar a las
arterias pulmonares y al ductus arterioso. (Sin referenciar).

31)Los derivados del intestino posterior están irrigados típicamente por la:
a) Arteria celiaca.
b) Arteria mesentérica superior.
c) Arteria mesentérica inferior.
d) Arteria iliaca interna.
e) Arteria umbilical.

La respuesta es C. El intestino medio es irrigado por la arteria mesentérica


superior; en el intestino anterior caudal el suministro lo da el tronco celiaco y la
arteria para el intestino posterior y sus derivados es la arteria mesentérica inferior.
(Sin referenciar).

32)Una compañía de biotecnología está desarrollando un monitor que puede


detectar el latido del corazón fetal. Su objetivo es desarrollar un producto
que pueda ayudar a detectar el latido de corazón de un feto lo más pronto
posible. ¿De los siguientes tiempos, cuál es la época más temprana en la
que este dispositivo podría, teóricamente, detectar el latido?
a) 2 semanas.
b) 3 semanas.
c) 4 semanas.
d) 6 semanas.
e) 8 semanas.

La respuesta es C. El embrión de 3 semanas es una placa trilaminar, en este


momento el corazón se empieza a formar y comienza a latir casi inmediatamente.
La hematopoyesis ocurre en el saco vitelino, y un sistema circulatorio primitivo
conecta el plexo capilar del saco vitelino con el corion del embrión. La partición de
la aurícula comienza en la cuarta semana. Durante la quinta semana, se forma el
septum cardiaco y se fusionan las almohadillas endocardiacas. Para la sexta
semana, el corazón está cerca a estar completamente formado. Esta formación
temprana del corazón explica por qué es tan difícil intentar prevenir la ocurrencia
de malformaciones congénitas del sistema cardiovascular, ya que la madre de un
feto de 6 semanas tan solo está a 8 semanas de su último periodo menstrual, y
muchas asumen que tan solo “se saltaron un periodo” (un fenómeno muy común)
por razones diferentes al embarazo. (Sin referenciar).

33)La circulación útero-placentaria primitiva se establece funcionalmente


durante ¿qué período de desarrollo embrionario / fetal?
a) Primera semana
b) Segunda semana
c) Tercera Semana
d) Fin del primer mes
e) segundo trimestre

La respuesta es D. Durante la segunda semana de desarrollo fetal, los espacios


lacunares se desarrollan entre las células del sincitiotrofoblasto, particularmente
en la región del polo embrionario, desde que el blastocisto invade el endometrio.
Capilares endometriales en esta región se dilatan y llenan de sangre para formar
sinusoides. Las células sincitiales, producen erosión directa del endotelio de los
capilares maternos, permitiendo que la sangre materna entre en las lagunas y
bañe las células sincitiales. Durante la segunda semana, las vellosidades
primarias consisten en proyecciones de las células sincitiales que rodean un
núcleo de células de citotrofoblasto (respuesta B). Durante la tercera semana
(respuesta c), el núcleo de las vellosidades es invadido por las células del
mesodermo para formar una vellosidad secundaria. Las células del núcleo
mesodérmico entonces se diferencian para formar capilares y células de la sangre
para el final de la tercera semana (vellosidad terciaria). Los buques se conecten a
la circulación fetal temprana en la cuarta semana, el establecimiento de la
circulación útero-placentaria funcional. (Sin referenciar).

34)El sitio más importante de producción de los elementos hematopoyéticos


en la etapa fetal es:
a) Medula ósea
b) Bazo
c) Timo
d) Hígado
e) Riñón

Respuesta: D [ CITATION Ros12 \l 9226 ]. Fase hepática: ocurre en los comienzos de


desarrollo fetal, focos o centros hematopoyéticos aparecen en el hígado.

35)Del desarrollo del sistema cardiovascular se puede decir que, excepto:


a) El tubo cardíaco se forma a partir de células del mesodermo lateral
definitivo esplácnico
b) Las almohadillas cardiacas están formadas por células de las crestas
neurales cardiacas
c) La tabicación del seno aórtico se da a partir de células del mesodermo
lateral definitivo esplácnico
d) Las células de la cresta neural cardiaca se diferencian en células
musculares lisas de los vasos
e) Los compartimentos cardiacos están formados por procesos de tabicación
regulada por el gen Pax3

Respuesta: C. El aparato cardiovascular es el primer sistema embrionario en


desarrollarse, con incremento de las demandas nutricionales y de oxígeno
consecuentes al crecimiento acelerado del embrión. El aparato cardiaco se origina
de: mesodermo esplácnico, forma primordio del corazón posteriormente los tubos
cardíacos endocárdicos y el miocardio primitivo; mesodermo paraxial y lateral, y
células de la cresta neural ubicadas entre las vesículas óticas y el tercer par de
somitas. Keith L. Moore & T.V.N. Persaud. (2007). Embriología clínica: El
desarrollo del ser humano: Aparato cardiovascular. (7°.). (pp. 330-340). Barcelona,
España: Elsevier.

36)El proceso de tabicación del seno aórtico está mediado principalmente


por:
a) GATA-4
b) PAX-3
c) Semaforina
d) Endotelina
e) FOXC-1

Respuesta B. El gen Pax3, se encarga de regular la tabicación de los


compartimientos cardiacos. El gen GATA-4 se encarga de la fusión de los tubos
cardiacos primitivos. La semaforina regula la formación de los nervios periféricos,
actuando como quemorrepelentes unidos a la superficie celular o a la matriz
extracelular. Las endotelinas son vasoconstrictores con acción paracrina y
endocrina. Los genes Fox participan en la diferenciación celular. Keith L. Moore &
T.V.N. Persaud. (2007). Embriología clínica: El desarrollo del ser humano. (7°.).
(pp. 329-378). Barcelona, España: Elsevier. Dr. Edgar Osuna S. & Dr. Alfredo
Rubiano C. Guía de neuroanatomía estructural y funcional. (pp. 6-7). Michael H.
Ross & Wojciech Pawlina. (2008). Histología: Texto y atlas color con Biología
Celular y Molecular. (5°.). (pp. 408). Buenos aires, Argentina: Médica
panamericana. S. A.

APARATO DIGESTIVO

1) ¿En cuál de las siguientes estructuras se convierte el Alantoides fetal en


el humano adulto?
a) Cloaca.
b) Ligamentos umbilicales medios.
c) Uraco.
d) Uréter.
e) Uretra.
La respuesta es C. El uraco es un remanente fibroso que se extiende desde el
ombligo a la vejiga urinaria en el adulto, también recibe el nombre de ligamento
umbilical medio de la pared abdominal anterior. A es incorrecto la cloaca es la
región endodérmica primitiva que recibe la porción terminal del intestino, es
subdividida en áreas urogenital y anal. B es incorrecto los ligamentos umbilicales
medios son una estructura pareada localizada profundo al peritoneo de la pared
abdominal anterior. Se forman a partir de las porciones distales de las arterias
umbilicales obliteradas. D es incorrecto el uréter se desarrolla a partir de un
crecimiento en el extremo caudal del conducto mesonéfrico. E es incorrecto el
epitelio de la uretra es de origen endodérmico, mientras que el tejido conectivo y
muscular que la rodea deriva de la hoja esplácnica del mesodermo. (Sin
referenciar).

2) Las siguientes aseveraciones con relación a la embriología del divertículo


de Meckel son correctas excepto:
a) El divertículo de Meckel por lo general se origina en el íleon a unos 60
cm de la válvula ileocecal.
b) El divertículo de Meckel se debe a falta de obliteración del conducto
vitelino.
c) La frecuencia del divertículo de Meckel en la población general es de
5%.
d) El divertículo de Meckel es un divertículo verdadero que tiene todas las
capas de la pared intestinal.
e) La mucosa gástrica es el tejido ectópico más común que se encuentra
en un divertículo de Meckel.

La respuesta es C. En los estudios de necropsia se ha estimado que la


frecuencia del divertículo de Meckel es de 1 a 2% y los hombres son afectados
con más frecuencia que las mujeres con una razón de 2:1. A, B, D, E, son
correctas. Se origina por lo general en el borde antimesentérico del íleon, a unos
45 a 60 cm de distancia de la válvula ileocecal. Es un vestigio del conducto
onfalomesentérico o vitelino, que casi siempre experimenta obliteración completa
durante la sexta semana se la gestación. El divertículo de Meckel es un verdadero
divertículo que contiene todas las capas de la pared intestinal. Hay mucosa
gástrica en 50% de todos los divertículos de Meckel, pero en más del 75% de los
individuos sintomáticos. (Sin referenciar).

3) Un recién nacido no expulsa meconio hasta 48 horas después de nacido.


Dos semanas después su madre reporta que él recién nacido no ha
defecado con regularidad. La manometría anorrectal revela una presión
elevada del esfínter anal en la distensión rectal con un globo. El estudio
radiográfico revela una dilatación masiva del colon próximo al recto.
¿Esto indica una anormalidad del desarrollo en cuál de los siguientes
tejidos embrionario?
a) Ectodermo.
b) Endodermo.
c) Cresta Neural.
d) Neuroectodermo.
e) Mesodermo esplácnico.

La respuesta es C. Este neonato presenta las características de la Enfermedad


de Hirschsprung, que es causada por la ausencia de células ganglionares en la
pared del colon. La cresta neural contribuye a la formación de muchas de estas
estructuras en el adulto. Entre estas encontramos las neuronas postganglionares
del sistema nervioso autónomo y neuronas sensoriales del sistema nervioso
periférico. A es incorrecto el ectodermo forma la epidermis de la piel y las células
parenquimatosas de las glándulas asociadas con la piel como las glándulas
sudoríparas, sebáceas y mamarias. B es incorrecto, el endodermo forma el
revestimiento epitelial del tubo intestinal y células parenquimales de las glándulas
asociadas al tubo intestinal como el hígado y el páncreas. D es incorrecto el
neuroectodermo forma el sistema nervioso central, las neuronas somáticas
motoras del sistema nervioso periférico, y las neuronas preganglionares del
sistema nervioso autónomo. E es incorrecta el mesodermo esplácnico da origen a
la pleura visceral. (Sin referenciar).

4) Una mujer de 32 años se presenta a su obstetra con malestar abdominal,


aumento de dolor de espalda, dificultad para respirar e hinchazón en los
pies y los tobillos. El ultrasonido revela un índice de líquido amniótico
(AFI) de 27 cm (normal 5 a 24 cm). La condición es causada por cuál de
las siguientes?
a) Atresia Duodenal o esofágica
b) Agenesia bilateral de los riñones
c) Desarrollo precoz del reflejo de deglución en el feto
d) Hipoplasia del pulmón
e) Uropatía obstructiva

La respuesta es a. Un índice de líquido amniótico de 27 cm es indicativo de


polihidramnios, atresia Duodenal y / o esofágica resulta en una incapacidad del
feto a tragar líquido amniótico (respuesta c). El resultado es que la recirculación
normal de líquido amniótico a través del embrión se reduce o elimina en gran
medida, provocando un exceso de líquido amniótico. El exceso de líquido
amniótico se define como mayor de 2000 ml en el tercer trimestre. Bajos
volúmenes de líquido amniótico (oligohidramnios) son causadas por la rotura de
las membranas fetales, agenesia bilateral de los riñones (respuesta B), o uropatía
obstructiva (respuesta e, bloqueo de los cálices o los uréteres), que evita que la
orina se añada al líquido amniótico. Hipoplasia de los pulmones (respuesta d) y la
compresión del cordón umbilical se asocia con oligohidramnios, pero no lo causan.
La presencia de líquido adecuado en los pulmones desinflados es esencial para la
maduración pulmonar, y factores de crecimiento en el líquido amniótico también
pueden ser importantes. Los bajos niveles de líquido amniótico inhiben
severamente el desarrollo pulmonar. La fórmula para la comprensión de la relación
entre la orina y el líquido amniótico es: Menos producción de orina = menos líquido
amniótico; menos deglución = más fluido amniótico. (Sin referenciar).

5) Un bebé de sexo masculino a término presenta vómito en proyectil 1h


después de tomar leche materna. No hay aumento de peso durante las
primeras 48 horas. El vómito no es teñido de bilis y no tiene dificultad
respiratoria evidente. El examen físico revela un abdomen no tenso ni
hinchado. ¿Cuál de las siguientes es la explicación más probable?
a) Estenosis pilórica hipertrófica congénita.
b) La ausencia congénita de un riñón.
c) Divertículo íleal (De Meckel).
d) Ano imperforado.
e) Fístula traqueoesofágica.

La respuesta es A. El Bloqueo del intestino proximal en el recién nacido produce


vómito en proyectil. La estenosis pilórica hipertrófica congénita, ocurre en el 0,5-
1,0% de los hombres y es raro en mujeres, involucra la hipertrofia de la capa
circular de músculo en el píloro. Esto usualmente no tiene regresión espontánea y
requiere intervención quirúrgica. Durante la quinta a sexta semana de desarrollo,
el lumen del duodeno está ocluido por el músculo en proliferación pero
normalmente se recanaliza durante la octava semana, la falla en la recanalización
resulta en la atresia duodenal. Como éste evento ocurre proximal a la ampolla
hepatopancreática, el vómito no se verá teñido con bilis. El páncreas anular,
raramente, genera un bloqueo completo del duodeno. B es incorrecta la ausencia
congénita de un riñón no se presenta con los síntomas descritos. D es incorrecta
el ano imperforado produce distensión abdominal, timpanismo intestinal e
impactación fecal. E es incorrecta un neonato con una fístula traqueosofágica
típica al alimentarse puede broncoaspirar. C es incorrecta en un neonato con un
divertículo íleal por persistencia de una pequeña porción del conducto
onfalomesentérico o conducto vitelino, se forma una evaginación del íleon por lo
general asintomático aunque a veces en presencia de mucosa gástrica o tejido
pancreático heterotopico se puede presentar ulcera, hemorragia e incluso
perforación, en otro tipo de variante el conducto onfalomesentérico conserva la
permeabilidad en toda su longitud se presenta una conexión entre el tracto
gastrointestinal y el ombligo esta anomalía se denomina fistula umbilical o vitelina
y puede haber expulsión de heces por el ombligo. (Sin referenciar).

6) Un recién nacido tiene vómitos en proyectil en un periodo corto de


tiempo luego de cada comida. Se ha determinado que tiene una
obstrucción en el tracto digestivo como resultado de páncreas anular. El
páncreas anular es el resultado de una anomalía en cuál de los siguientes
procesos:
a) Rotación del brote pancreático dorsal alrededor de la primera parte del
duodeno.
b) Rotación del brote pancreático dorsal alrededor de la segunda parte del
duodeno.
c) Rotación del brote pancreático dorsal alrededor de la tercera parte del
duodeno.
d) Rotación del brote pancreático ventral alrededor de la primera parte del
duodeno.
e) Rotación del brote pancreático ventral alrededor de la segunda parte del
duodeno.

La respuesta es E. El brote pancreático ventral normalmente rota alrededor del


duodeno para fusionarse con el brote pancreático dorsal. Ambos brotes
pancreáticos se forman de evaginaciones desde la segunda porción del duodeno.
La rotación normal es alrededor del lado derecho del duodeno embrionario. El
páncreas anular resulta del brote pancreático ventral que se divide y rota alrededor
tanto del lado derecho como del izquierdo de la segunda porción del duodeno. A,
B, C son incorrectas el brote dorsal pancreático no rota alrededor del duodeno, y
por lo tanto no es la causa del páncreas anular. D es incorrecto el brote
pancreático ventral no se forma de la primera parte del duodeno y por lo tanto no
rota alrededor de esta parte del duodeno, el brote dorsal forma cuerpo y cola
mientras que el ventral la cabeza y el proceso uncinado. (Sin referenciar).

7) Mujer de 55 años de edad llega al médico quejándose de fatiga. Su


hematocrito es 32% a pesar del hecho de que ella pasó por la menopausia
hace cinco años. El médico sospecha hemorragia gastrointestinal y la
remite al gastroenterólogo. Una evaluación completa fracasa en revelar
alguna evidencia maligna, y el gastroenterólogo sospecha que la
hemorragia puede deberse a alguna anormalidad del desarrollo. ¿Cuál de
las siguientes anomalías del desarrollo podría explicar la pequeña
hemorragia gastrointestinal?
a) Heterotopía del sistema nervioso central.
b) Heterotopía gástrica.
c) Heterotopía pancreática.
d) Heterotopía paratiroidea.
e) Heterotopía tiroidea.

La respuesta es B. Los restos heterotópicos son pequeñas áreas de tejido


normal en sitios anormales. Usualmente son clínicamente insignificantes, a menos
que formen una masa notoria, tengan una hiperfunción o sean mal diagnosticados
por medio de biopsia (levantando sospechas de cáncer metastásico). En el
intestino delgado puede haber tejido heterotópico gástrico y producir ácido lo que
es suficiente para generar una úlcera péptica en la mucosa adyacente. Ésta úlcera
puede ser la fuente del sangrado gastrointestinal. A, D, E son incorrectas las
heterotopías en el sistema nervioso central, paratiroides y tiroides pueden ocurrir,
pero no serían de esperarse en el intestino delgado, y no es probable que causen
sangrado dado el caso de que se presenten. C es incorrecta la heterotopía
pancreática puede ocurrir en el intestino delgado, pero no causa sangrado. (Sin
referenciar).

8) Un neonato con bajo peso, que pasó meconio en el primer día de vida, es
iniciado en formula a los dos días de nacido. El infante desarrolla
distensión abdominal y dolor, acompañado de hallazgos que sugieren
sepsis, incluyendo hipertensión y neutrofilia. ¿Qué se observará más
probablemente en la cirugía de emergencia?
a) Un engrosamiento masivo del píloro.
b) Un colon masivamente dilatado.
c) Asas intestinales en la cavidad torácica.
d) Gangrena de la porción terminal del Íleon y el colon ascendente.
e) Tracto gastrointestinal de apariencia normal.

La respuesta es D. El infante tiene Enterocolitis necrozante, la cual puede afectar


a los infantes en cualquier momento en sus primeros meses de vida, pero es más
común que suceda en el momento en el que empiezan la ingesta por vía oral. Las
funciones inmunitarias del intestino neonatal dejan particularmente vulnerable a
una combinación de lesión isquémica y la colonización por organismos patógenos.
Los infantes alimentados con formula parecen ser más vulnerables que aquellos
alimentados con leche materna, posiblemente debido a la ausencia de anticuerpos
maternos u otros factores inmunoprotectores en la fórmula. Los prematuros y los
que tienen bajo peso al nacer desarrollarán más probablemente ésta condición. A
es incorrecta un engrosamiento del píloro es una característica de estenosis
pilórica hipertrófica, la cual se presenta típicamente con vómito y regurgitación en
el infante mayor. B es incorrecta un colon masivamente dilatado sugiere
enfermedad de Hirschsprung, que se puede presentar de una forma similar a la
enterocolitis necrozante. Ambas condiciones son usualmente distinguidas por el
paso previo de meconio del neonato, lo cual no sucede en la enfermedad de
Hirschsprung. C es incorrecta las asas intestinales en la cavidad torácica son
características de una hernia diafragmática congénita, la cual causa dificultad
respiratoria y a menudo la muerte en neonatos. E es incorrecta la aparición de un
tracto gastrointestinal normal no aparecería en un caso como este. (Sin
referenciar).

9) Seleccione la afirmación verdadera con respecto al páncreas anular.


a) La mitad de los casos son reportados en adultos.
b) En el neonato se presenta con pancreatitis.
c) Todos los pacientes con la lesión tienen enfermedad de la úlcera
péptica.
d) La resección Pancreática es el tratamiento de elección.

La respuesta es A. El cuarenta por ciento de los lactantes con páncreas anular


tiene estenosis o atresia duodenal concurrente. En casi un tercio de los casos
ocurren síntomas de obstrucción duodenal (distensión gástrica y vómitos) en la
primera semana de vida y casi en la mitad en el primer año. El resto son
asintomáticos hasta la vida adulta, cuando es posible que presenten dolor
abdominal, náuseas y vómito. Sin embargo es posible que algunos no originen
problemas y nunca se diagnostiquen, son incorrectas B, C, en el caso D, el
tratamiento casi siempre implica cirugía para realizar una derivación en la parte
bloqueada del duodeno. (Sin referenciar).

10)Un niño de 5 años es llevado a emergencias con un sangrado rectal


masivo indoloro. La colonoscopia no demuestra lesión alguna en el colon
o ano. La endoscopia digestiva alta falla en demostrar esofagitis, ulcera
gástrica, o ulcera duodenal. Una prueba de Tecnecio 99 demuestra una
anomalía en la parte inferior del abdomen. El fallo de un proceso normal
del desarrollo involucra cuál de las siguientes estructuras que es la más
probable causante del sangrado del niño:
a) Apéndice.
b) Ciego.
c) Duodeno.
d) Íleon.
e) Yeyuno.
La respuesta es D, Un divertículo de Meckel es causado por una falla en la
obliteración del ducto vitelo-intestinal. Está clásicamente localizado en el Íleon
distal dentro de los 45 a 60 cm de la válvula ileocecal, y la estructura es un
verdadero divertículo con mucosa, submucosa y muscularismucosae. Muchos
divertículos de Meckel contienen tejido pancreático ectópico o mucosa gástrica, y
la producción de ácido de la mucosa gástrica puede ser suficiente para producir
una pequeña úlcera péptica en la mucosa intestinal adyacente. Aquellas pequeñas
úlceras pépticas son fuentes ocasionales del misterioso dolor parecido a la
apendicitis, o del sangrado intestinal. El Tc99 se concentra en la mucosa gástrica
y el examen en este paciente demuestra una pequeña cantidad de mucosa
gástrica ectópica ubicada en el divertículo. A es incorrecta la apendicitis aguda es
usualmente muy dolorosa y no causa sangrado rectal típicamente. B es incorrecta
una lesión en el ciego habría sido revelada por la colonoscopia. C es incorrecta el
fracaso de la endoscopia digestiva alta en demostrar úlcera péptica del duodeno
hace improbable la enfermedad duodenal. E es incorrecta teóricamente, el yeyuno
puede haber sido la raíz del problema, pero el sangrado yeyunal es poco común y
el divertículo de Meckel es mucho más probable en población pediátrica respecto
al sangrado de origen yeyunal. (Sin referenciar).

11)El epiplón mayor es derivado de cuál de las siguientes estructuras


embrionarias:
a) Mesoduodeno dorsal.
b) Mesogastrio dorsal.
c) Canal pericardioperitoneal.
d) Membranas pleuropercardiales.
e) Mesenterio ventral.

La respuesta es B. Tanto la bolsa omental como el epiplón mayor se derivan del


mesogastrio dorsal, que es el mesenterio de la región estomacal. A es incorrecta
el mesoduodeno dorsal es el mesenterio del duodeno en desarrollo, que luego
desaparece de tal forma que el duodeno y el páncreas vienen a yacer
retroperitonealmente. C es incorrecto el canal pericardio peritoneal
embriológicamente conecta la cavidad pericárdica y peritoneal del celoma
intraembrionario este canal será el punto de desarrollo pulmonar y no existe el
espacio pleural puesto que no se han desarrollado los pulmones. D es incorrecta
las membranas pleuropericardicas separan el espacio pleural del pericárdico y
contribuyen a la formación del diafragma junto con la membrana pleuroperitoneal.
E es incorrecta el mesenterio ventral forma el ligamento falciforme, ligamento
redondo, y el epiplón menor. (Sin referenciar).

12)El ligamento umbilical medio es remanente embriológico de:


a) Arteria Umbilical.
b) Uraco.
c) Vena Umbilical.
d) Ductus Arterioso.
e) Septum Primum.

La respuesta es B. El ligamento umbilical medio es una banda fibrosa adherida al


ápex de la vejiga que la conecta con el ombligo, este es remanente embriológico
del uraco. A, C, D, E son incorrectas la arteria umbilical, el Ductus arterioso, el
Septum Primum, y la venas umbilical son todas estructuras embriológicas
asociadas con la circulación fetal. El remanente de la obliteración de las arterias
umbilicales en el adulto son los ligamentos umbilicales mediales. (Sin referenciar).

13)Un recién nacido arroja vómitos biliosos poco después de cada comida.
Una revisión de los registros de sus madres muestran la presencia de
polihidramnios durante el embarazo, pero un análisis del cariotipo fue
normal. ¿Cuál de las siguientes es la causa más probable de estos
resultados en el recién nacido?
a) Brote pancreático ventral bífido
b) Enfermedad de Hirschsprung
c) Divertículo ileal
d) Estenosis pilórica
e) Fístula traqueoesofágica

La respuesta correcta es A: El brote pancreático ventral normalmente gira


alrededor del duodeno para fusionarse con el brote pancreático dorsal. Ambas
yemas de páncreas se forman a partir de evaginaciones de la segunda parte del
duodeno, por lo tanto, la rotación es alrededor de la segunda parte del duodeno
distal a la ampolla de Vater. La rotación normal es alrededor de la parte derecha
del duodeno embrionario. Los resultados del páncreas anular de un brote
pancreático ventral bífido gira en torno a ambos lados derecho e izquierdo de la
segunda parte del duodeno, causando estenosis duodenal. Debido a la estenosis,
los pacientes presentan vómitos con bilis y los hallazgos radiológicos son los
mismos que se encuentran en la atresia duodenal: Mucho aire-líquido a nivel del
estómago y algo más en la primera porción del duodeno. (Sin referenciar).

14)De la formación del hígado se puede decir que, excepto:


a) El hígado se forma del piso del intestino anterior
b) Los hepatocitos derivan del endodermo
c) La vasculatura del hígado deriva del mesodermo
d) El hígado se forma por una evaginación del intestino primitivo sobre el
septo transverso
e) El conducto hepático comunica el hígado con el intestino anterior

La respuesta: D. El numeral d es falso porque el hígado se forma a partir de


evaginaciones del epitelio endodermico de la porción superior del duodeno. (Sin
referenciar).

15)Del desarrollo del intestino medio se puede decir que, excepto:


a) Su eje longitudinal es la arteria mesentérica superior
b) Su crecimiento longitudinal hace una rotación en sentido anti-horario en dos
tiempos diferentes
c) El asa cefálica permanece en la vida postnatal con posición cefálica y el asa
caudal permanece en la vida postnatal con posición caudal
d) Del asa cefálica se derivan la mayor parte del intestino delgado
e) Del asa caudal se origina el intestino grueso derecho

La respuesta es c. La rama cefálica del asa se convierte en la porción distal del


duodeno, el yeyuno y parte del íleon. La rama caudal se convierte en la parte
inferior del íleon, el ciego y el apéndice, el colon ascendente y los dos tercios
proximales del colon transverso. (Sin referenciar).

16)Del desarrollo del intestino posterior se puede decir que, excepto:


a) De él se deriva el colon sigmoide y el 1/3 izquierdo del colon transverso
b) Las células mesenquimatosas forman el tabique urorrectal
c) La cloaca se divide por el tabique urorrectal en una porción anterior anal y
posterior uro-genital
d) La porción superior del ano es de componentes endodérmico
e) La porción inferior del ano es el componente ectodérmico de la membrana
cloacal

La respuesta es c. El tabique urorrectal separa la región entre la alantoides y el


intestino posterior. El extremo de este tabique se aproxima hasta localizarse cerca
a la membrana cloacal, aunque las dos estructuras nunca toman contacto. Al final
de la séptima semana, la membrana cloacal se rompe y aparece la abertura anal
para el intestino posterior y un orificio ventral para el seno urogenital. (Sin
referenciar).

17)Son estructuras que derivan del esbozo dorsal del páncreas


principalmente, excepto:
a) Apófisis uncinada
b) Cabeza del páncreas
c) Cuerpo del páncreas
d) Cuello del páncreas
e) Cola del páncreas

La respuesta es a. La apófisis uncinada del etmoides se forma a partir del


mesodermo paraaxial. (Sin referenciar).

18)¿Durante qué semana del crecimiento fetal el intestino medio retorna a la


cavidad abdominal del cordón umbilical?
a) 3º Semana.
b) 4º Semana.
c) 10º Semana.
d) 15º Semana.

La respuesta es C. El intestino medio retorna a la cavidad abdominal alrededor


de la 10° semana de vida embrionaria. (Sin referenciar).

19)El intestino anterior va desde:


a) Membrana buco-faríngea a esbozo respiratorio
b) Esbozo respiratorio a vestíbulo intestinal anterior
c) Vestíbulo intestinal anterior a vestíbulo intestinal posterior
d) Vestíbulo intestinal posterior a membrana cloacal
e) Membrana bucofaríngea a membrana cloacal

Respuesta B, (Langman 11° edición capítulo 6). El intestino anterior está


delimitado temporalmente por una membrana ectoendodermica denominada
membrana bucofaríngea, esta membrana separa el estomodeo (cavidad bucal
primitiva que deriva del ectodermo) de la faringe (una parte del intestino anterior
que deriva del endodermo).

20)Del desarrollo del intestino anterior se puede decir que


a) Los rebordes tráqueo-esofágicos se fusionan y forman el tabique
traqueoesofágico
b) La parte dorsal corresponde al esófago
c) La parte ventral corresponde a la tráquea y a los esbozos pulmonares
d) El orificio laríngeo comunica la primordia respiratoria con la faringe
e) Todas las anteriores son ciertas

La respuesta es E. La FTE fístula tráqueo esofágica se produce por la división


incompleta de la parte craneal del intestino anterior en las porciones respiratorias y
esofágica durante la cuarta semana, lo que produce un tabique traqueoesofágico
defectuosos; la mayor parte de los recién nacidos afectados son los varones. De
sus cuatro tipos la más común es la AE-Atresia esofágica, que se produce en la
parte superior del esófago. Los neonatos con esta anomalía regurgitan
frecuentemente, tienen alto riesgo de neumonía y ahogamiento, el polihidramnios
es una característica de AE. Las opciones B, C y D, son características de los
sistemas respiratorios y digestivos que son características embrionarios que se
conservan en la adultez. Keith L. Moore & T.V.N. Persaud. (2007). Embriología
clínica: El desarrollo del ser humano. (7°.). (pp. 242-245). Barcelona, España:
Elsevier.

APARATO FARÍNGEO

21)La mamá de un niño de 7 años de edad le expresa al pediatra que su hijo


ha venido desarrollando una irritación en un lado de su cuello. El pediatra
realiza una palpación que revela un pequeño quiste secretor de mucosa a
lo largo del borde anterior del músculo esternocleidomastoideo. El seno
maxilar branquial, o fístula cervical externa, es el resultado de una
obliteración incompleta. ¿Cuál de las siguientes estructuras es la
responsable de la obliteración incompleta?
a) Primer arco faríngeo.
b) Primera hendidura faríngea.
c) Primera bolsa faríngea.
d) Segundo arco faríngeo.
e) Segunda hendidura faríngea.
f) Segunda bolsa faríngea.

La respuesta es E. En el desarrollo normal del cuello, la segunda hendidura


faríngea crece inferiormente para fusionarse con el reborde cardíaco. El seno
cervical formado por la fusión de estas masas de tejido es normalmente una
estructura transitoria, pero puede persistir como un quiste, o como una fístula
externa o interna. A es incorrecto, el primer arco faríngeo da lugar a diferentes
estructuras óseas, musculares y ligamentosas: el martillo y el yunque, los
músculos de la masticación, más el tensor del tímpano, tensor del velo palatino,
músculo hioideo y digástrico anterior, y el ligamento maleolar esfeno mandibular
anterior. B es incorrecto, la primera hendidura faríngea se mantiene como el
meato auditivo externo y el canal. C es incorrecto, la primera bolsa faríngea da
lugar a la cavidad timpánica y trompa de Eustaquio. D es incorrecto el segundo
arco faríngeo igual que el primero, da lugar a huesos, músculos y ligamentos: el
estribo y la parte superior del cuerpo del hueso hioides, los músculos de la
expresión facial, el estilo hioideo, digástrico posterior y el ligamento estilohioideo.
F es incorrecto la Segunda bolsa faríngea se convierte en el lecho amigdalar. (Sin
referenciar).

22)Son derivados mesodérmicos de las bolsas viscerales, excepto:


a) Huesos del cuello
b) Cartílagos laríngeos
c) Músculos laríngeos
d) Epidermis del cuello
e) Dermis del cuello

Respuesta: D. Epidermis del cuello. La capa germinal mesodérmica da origen a


los somitas que originan el miotoma (tejido muscular), el esclerotoma (cartílago y
hueso) y el dermatoma (tejidos subcutáneos de la piel), que son todos tejidos de
sostén del cuerpo. La epidermis es una estructura derivada del ectodermo
(Embriología de Langman, págs. 88,89).

23)Son derivados del mesodermo del primer arco faríngeo, excepto:


a) Músculo del martillo
b) Músculo del estribo
c) Yunque
d) Primer arco aórtico
e) Vientre anterior del digástrico
Respuesta: B. El músculo del estribo. El primer arco faríngeo forma el cartílago
de Meckel, del cual se conservan porciones dorsales para formar el yunque y el
martillo. Además del mesénquima de la apófisis maxilar superior e inferior. Entre
las musculatura está el músculo de la masticación, vientre anterior del digátrico,
milohiodieo, tensor del tímpano y tensor del velo del paladar. El músculo del
estribo hace parte de un derivado del segundo arco faríngeo (Embriología de
Langman, págs. 268 y 269).}

2) De las bolsas viscerales se puede decir que, excepto:


a) Tienen solamente un componente mesodérmico y endodérmico
b) Se diferencia en sentido céfalo-caudal
c) Se comienzan a formar inicialmente al final de la sexta semana
d) Sufre una regresión al final de la quinta semana
e) La cuarta y sexta se fusionan para formar una sola bolsa definitiva

La respuesta: A. Con el desarrollo de los arcos y las hendiduras se forman una


serie de evaginaciones, las bolsas viscerales (faríngeas) que penetran en el
mesénquima circundante y tienen un revestimiento endodérmico epitelial que
origina una serie de órganos importantes. Son los arcos faríngeos los que
aparecen en una secuencia céfalo-caudal (b). Para la 4ta semana se puede ver en
el embrión, con el rompimiento de la membrana bucofaríngea, la boca primitiva, el
estomodeo, y también, el primer par de arcos faríngeos (c). Los componentes
cartilaginosos del cuarto y sexo arcos faríngeos se fusionan para formar los
cartílagos tiroides, cricoides, aritenoides, corniculados y cuneiforme de la laringe
(e). (Sin referenciar).

3) Son derivados del endodermo de las bolsas viscerales, excepto:


a) Epitelio de revestimiento de la lengua
b) Epitelio de revestimiento de la cavidad oral
c) Músculos intrínsecos de la lengua
d) Receso tubotimpánico
e) Amígdala palatina

La respuesta es c. Los dos tercios anteriores de la lengua resultan de la


proliferación del mesénquima en el primer par de arcos faríngeos. El tercio
posterior se desarrolla a partir de estructuras procedentes del mesénquima del
segundo, tercero y cuarto pares de arcos faríngeos. El epitelio de revestimiento de
la lengua se origina a partir del endodermo del primer y segundo arcos faríngeos
(a).La primera bolsa faríngea forma un divertículo, el seno tubotimpanico, cuya
porción distal se ensancha en una estructura de forma similar a un saco, la
cavidad del oído medio y la parte proximal se mantiene estrecha, formando el
receso tubotimpánico (d). El revestimiento epitelial de la segunda bolsa faríngea
prolifera y forma unas yemas que penetran en el mesénquima circundante; a
continuación, las yemas son invadidas por tejido mesodérmico de manera que
forman el primordio de las amígdalas faríngeas (e). (Sin referenciar).

4) De la primera bolsa branquial se puede decir que, excepto:


a) El ectodermo epitelial origina el conducto auditivo externo
b) El mesodermo origina los músculos de la masticación
c) El arco endodérmico origina el receso tubotimpánico
d) El arco endodérmico origina el epitelio oral
e) La interacción de todos sus campos morfogenéticos originan la membrana
timpánica

La respuesta es c. La primera bolsa faríngea forma un divertículo, el seno


tubotimpanico, cuya porción distal se ensancha en una estructura de forma similar
a un saco, la cavidad del oído medio y la parte proximal se mantiene estrecha,
formando el receso tubotimpánico. La parte dorsal de la primera hendidura penetra
en el mesénquima inferior y origina el conducto auditivo externo (a).El mesodermo
del primer arco faríngeo originan a los músculos de la masticación (b). En el
séptimo mes, el revestimiento epitelial del suelo del conducto auditivo externo
participa en la formación del tímpano definitivo (e). (Sin referenciar).

5) Es una estructura que se deriva del endodermo de la primera bolsa


faríngea:
a) Músculo del martillo
b) Epitelio de revestimiento de la yema lingual
c) Cavidad del oído medio
d) Epitelio de revestimiento laríngeo
e) Epidermis del conducto auditivo externo

La respuesta: E. El endodermo de la primera bolsa se prolonga lateralmente, a


través de la trompa de eustaquio y forma el oído medio y el antro mastoideo. El
músculo del martillo se deriva del primer arco branquial (a). El epitelio de
revestimiento de la yema lingual deriva del primer arco faríngeo (b). El endodermo
que recubre la hendidura laringotraqueal origina el epitelio de revestimiento
laríngeo (d) y el conducto auditivo externo se origina en el primer surco faríngeo.
(Sin referenciar).

6) De la tercera bolsa visceral se puede decir que, excepto:


a) El ectodermo origina la epidermis de la región media del cuello
b) El mesodermo proviene de los somitas cervicales
c) El asta mayor y la parte media del hioides tienen origen endodérmico
d) El músculo estilofaríngeo y el tercer arco aórtico se originan del mesodermo
e) La hendidura endodérmica tiene una porción ventral y otra dorsal

La respuesta es c. El cartílago del tercer arco faríngeo produce la parte inferior


del cuerpo y el asta mayor del hueso hioides. El ectodermo superficial de los arcos
faríngeos origina la epidermis de la región del cuello. (a). Los somitas se forman a
partir del mesodermo paraxial presomitico no segmentado caudalmente y se
segmentan en las regiones situadas más cranealmente (b). El músculo
estilofaríngeo (séptima somitomera) y el tercer arco aórtico (mesodermo faríngeo)
se originan del tercer arco faríngeo (d).Las bolsas faríngeas da origen a un
divertículo dorsal y otro ventral (e). (Sin referenciar).

7) La hendidura endodérmica de la tercera bolsa visceral origina:


a) Timo
b) Paratiroides inferior y superior
c) Paratiroides superior
d) Glándula mamaria
e) Laringe

La respuesta es A. En la quinta semana de gestación, el ala dorsal de la tercera


bolsa origina la región ventral del timo (a) Las glándulas paratiroides inferiores
derivan del tercer arco faríngeo (b). Las glándulas paratiroides superiores derivan
del cuarto arco faríngeo (c). La glándula mamaria deriva de un engrosamiento
epitelial ectodérmico (brote mamario) (d) y la Laringe deriva del surco
laringotraqueal en el endodermo de la cara (e). (Sin referenciar).

8) De la cuarta y sexta bolsa visceral se puede decir que, excepto:


a) Del ectodermo se origina la epidermis de la región inferior del cuello
b) El mesodermo proviene del mesodermo lateral esplácnico definitivo
c) Los cartílagos laríngeos se derivan del mesodermo
d) El arco endodérmico origina la epiglotis
e) La hendidura endodérmica origina en su parte ventral las células c de la
tiroides

Respuesta: B El mesénquima original deriva del mesodermo durante la tercera


semana, a lo lar de la cuarta, casi todo el mesénquima proviene de las células de
las células de la cresta neural que migran hacia los arcos faríngeos. Moore
Persaud, Embriología clínica, Pág. 202.

9) De la inervación de las bolsas viscerales se puede decir que, excepto:


a) La primera bolsa recibe inervación del V par
b) La segunda bolsa recibe inervación del VII par
c) La tercera bolsa recibe inervación del IX par
d) La cuarta bolsa recibe inervación del X par
e) La quinta bolsa visceral recibe inervación del X par

La respuesta: E. Solo se consideran 5 arcos faríngeos porque el quinto arco


faríngeo es transitorio o rudimentario en los seres humanos. (Sin referenciar).

10)Embriológicamente, el timo comparte origen con:


a) Paratiroides superior
b) Paratiroides inferior
c) Glándula tiroides
d) Conducto arterioso
e) Tráquea

La respuesta: B. En la quinta semana, el epitelio del ala dorsal de la tercera bolsa


se diferencia en la glándula paratiroides inferior, mientras que la de la porción
ventral forma el timo. El epitelio del ala dorsal de la cuarta bolsa faríngea forma la
glándula paratiroides superior (a). La glándula tiroides aparece como una
proliferacion epitelial en el suelo de la faringe, entre el tubérculo impar y la cópula,
en un punto que mas adelante indicara el agujero ciego. La porción dorsal de la
arteria del sexto arco desaparece en el lado derecho pero perdura en el izquierdo
convertida en el conducto arterioso que de este modo conecta la arteria pulmonar
izquierda con el cayado de la aorta (d). La tráquea, al igual que la laringe, los
bronquios y los pulmones derivan del tubo laringotraqueal (e). (Sin referenciar).

11)Son derivados de la primera bolsa faríngea, excepto:


a) Hueso mandibular
b) Yunque
c) Estribo
d) Hueso cigomático
e) Hueso temporal

La respuesta es C, El cartílago del segundo arco faríngeo o arco hioideo da


origen al estribo, la apófisis estiloides del hueso temporal, el ligamento
estilohioideo y, ventralmente, el asta menor y la porción superior del cuerpo del
hueso hioides. (Sin referenciar).
12)Un neonato tiene malformaciones en la mandíbula, oídos, y paladar. Él es
diagnosticado con disostosis mandibulofacial. Este síndrome se debe al
desarrollo anormal de ¿Cuál de las siguientes estructuras?
a) Primer arco faríngeo.
b) Primera hendidura faríngea.
c) Segundo arco faríngeo.
d) Segunda hendidura faríngea.
e) Tercera hendidura faríngea.
f) Tercera bolsa faríngea.

La respuesta es A. Los arcos faríngeos son bolsas de tejido visible que


sobresalen en el cuello externo del embrión. Estos están separados por
hendiduras faríngeas (cada una caudal a su correspondiente arco). Las bolsas
faríngeas son las bolsas que sobresalen de la faringe visible en el interior del
embrión que corresponden a los arcos. El defecto descrito se debe al desarrollo
anormal de los derivados del primer arco faríngeo. Se piensa que el defecto inicial
puede ser la migración insuficiente de células de la cresta neural. (Sin referenciar).

13)Un bebe recién nacido presenta labio leporino unilateral izquierdo. No hay
anomalías en el paladar del bebe. ¿Cuál de los siguientes defectos del
desarrollo explica esta observación?
a) Fallo en la fusión del proceso palatino lateral con el proceso palatino
medial.
b) Fallo en la unión de la prominencia maxilar con la prominencia nasal medial
izquierda.
c) Fallo en la fusión del primer paladar con el segundo paladar.
d) Fallo en la fusión de las prominencias nasales mediales derecha e
izquierda.
e) Fallo del proceso palatino derecho para fusionarse con el proceso palatino
izquierdo.

La respuesta es B. En la formación del labio superior, las prominencias nasales


mediales derechas e izquierdas se fusionan por compresión originada por la
prominencia maxilar superior. La prominencia maxilar superior entonces se fusiona
con la fusión de las prominencias mediales nasales. La ocurrencia de un fallo en
esta fusión resulta en labio leporino unilateral. A y C son incorrectas el fallo en la
fusión del proceso palatino lateral con el proceso palatino medial, que es un
sinónimo junto con el fallo en la fusión del primer paladar con el segundo paladar,
resulta en labio paladar hendido primario. Los paladares hendidos primarios se
encuentran anteriores al foramen incisivo del paladar. Los paladares hendidos
primarios pueden ocurrir junto con los labios leporinos pero son el resultado de
diferentes defectos en el desarrollo. D es incorrecta el fallo en la fusión de las
prominencias nasales mediales derecha e izquierda resulta en labio leporino
medial. Esta es una anomalía rara. Normalmente, Las prominencias nasales
mediales derecha e izquierda se funden en una sola prominencia medial. E es
incorrecta el fallo del proceso palatino derecho para fusionarse con el proceso
palatino izquierdo resulta en un paladar hendido secundario. Los paladares
hendidos secundarios se encuentran posteriores al foramen incisivo.
Normalmente, los procesos palatinos derecho e izquierdo se fusionan juntos y se
fusionan con el paladar primario. (Sin referenciar).

14)La evaluación de un infante con una variedad de anormalidades


congénitas revela hipocalcemia debido a la carencia de hormona
paratiroidea. En rayos X, la sombra tímica está ausente. Un fallo en el
desarrollo y la diferenciación de cuál de las siguientes estructuras
embrionarias puede más probablemente ser el responsable de la
presentación observada:
a) Segundo arco faríngeo.
b) Segunda hendidura faríngea.
c) Segunda bolsa faríngea.
d) Tercer arco faríngeo.
e) Tercera bolsa faríngea.

La respuesta es E. la tercera bolsa faríngea normalmente da lugar a las


glándulas paratiroides inferiores y el timo. Las células de estas bolsas migran
caudalmente en el embrión para la eventual localización de éstos órganos. Las
glándulas paratiroides superiores se derivan de la cuarta bolsa faríngea. La
ausencia de estos órganos está presente en el síndrome de Di George, que se
presenta típicamente con inmunodeficiencia e hipoparatiroidismo. A es incorrecta
el segundo arco faríngeo da lugar a varias estructuras esqueléticas y musculares.
Ellas incluyen el estribo, procesos estiloides, el ligamento estilohioideo, porciones
del hueso hioides, el músculo estapedio, el músculo estilohioideo, el músculo
digástrico posterior, y todos los músculos de la expresión facial inervados por
facial 7 par craneano. B es incorrecta la segunda hendidura faríngea normalmente
desaparece, la persistencia de la hendidura puede resultar en un quiste lateral
cervical, sinusoide o fístula. C es incorrecta la segunda bolsa faríngea da lugar a la
fosa amigdalina y las amígdalas faríngeas. D es incorrecta el tercer arco faríngeo
da lugar a la mayor parte del hueso hioides y al musculo estilofaríngeo inervado
por glosofaríngeo. El tercer arco aórtico, que pasa a través del tercer arco
faríngeo, da lugar a la arteria carótida común y a la arteria carótida interna. (Sin
referenciar).
15)Una mujer de 28 años, presenta trabajo de parto y llega a la sala con
contracciones regulares y fuertes. Ella no tuvo cuidados prenatales, pero
el parto se da sin incidentes, y ella da a luz a un varón de 7 lb y 4 oz. Una
fotografía del recién nacido revela una malformación. Este tipo de defecto
al nacer resulta del fallo en la fusión de las masas mesenquimales de cuál
de las siguientes estructuras embrionarias:

a) Procesos nasales laterales y mediales.


b) Procesos maxilares.
c) Procesos maxilar y nasolateral.
d) Procesos maxilar y nasomedial.
e) Proceso nasomedial.

La respuesta es D. A es incorrecta: durante el desarrollo de la cara, el proceso


nasal lateral forma la porción lateral de la nariz y ayuda a la formación del
conducto nasolagrimal. E es incorrecta los procesos nasales mediales adyacentes
se fusionan hacia la línea media y junto con la prominencia maxilar forman el
segmento intermaxilar. El segmento da origen a el surco subnasal (parte media
deprimida) del labio superior, la porción maxilar que soporta los cuatro incisivos
superiores, y el paladar primario. Bajo condiciones normales, los pares de
procesos maxilares se fusionan con los aspectos laterales y en prominencia nasal
medial de la fusión de los anteriores surge el labio superior continuo y la porción
maxilar que soporta los cuatro incisivos superiores, y el paladar primario, parte de
la nariz y conducto nasolagrial. El fallo en la fusión de un solo lado es más común
que el fallo bilateral que ilustra ésta imagen. El labio leporino puede estar
acompañado de paladar hendido (como en éste caso), lo que es causado por la
falla en la fusión de la parte anterior del paladar primario con los procesos
palatinos laterales que forman el paladar secundario, palatinos del maxilar y el
septo nasal. B es incorrecto los procesos maxilares forman las porciones
superiores de las mejillas y los aspectos laterales del labio superior. C es
incorrecta los procesos maxilares y nasolaterales están separados por los surcos
nasolagrimales, depresiones superficiales que indican el curso de los ductos
nasolagrimales. (Sin referenciar).

(Medline Plus, 2015; Labio leporino, También disponible en:


http://www.nlm.nih.gov/medlineplus/spanish/cleftlipandpalate.html)

16)Un paciente de 7 años de edad presenta una masa en la línea media


anterior del cuello, ligeramente por encima de la laringe. La masa es
móvil y se eleva por sobre la protrusión de la lengua. Esta masa es
más probablemente un quiste que se desarrolló de cuál de las
siguientes estructuras embrionarias:
a) Primera hendidura faríngea.
b) Primera bolsa faríngea.
c) Segunda hendidura faríngea.
d) Segunda bolsa faríngea.
e) Ducto tirogloso.

La respuesta es E. El ducto tirogloso se desarrolla como una evaginación del piso


de la faringe en la región en donde se desarrolla la lengua, el agujero ciego adulto
de la lengua marca el lugar de esta evaginación. El extremo distal de este ducto
normalmente forma la glándula tiroides; la parte proximal del ducto normalmente
se degenera. El fallo en la parte del ducto degenerada puede estar relacionado
con un quiste en el ducto tirogloso o un quiste cervical medio, como se observa en
este paciente. A es incorrecta la primera hendidura faríngea forma el canal
auditivo externo y tímpano. B es incorrecto la primera bolsa faríngea forma
conducto auditivo externo, porción proximal de la trompa de Eustaquio, tímpano y
porción proximal del oído medio. C es incorrecta la segunda hendidura faríngea
normalmente no permanece abierta y se relaciona con 3 y 4 hendidura que al
unirse el 2 y 3 arco forman el seno cervical que tiende a desaparecer. Si el seno
cervical permanece abierto podría formar un quiste cervical lateral, que se observa
en la parte lateral del cuello por encima del borde anterior del músculo
esternocleidomastoideo. D es incorrecta la segunda bolsa faríngea forma la fosa
tonsilar de la faringe. La mucosa faríngea en esta área se origina del endodermo
de esta bolsa. El crecimiento interno de las células mesodérmicas y migración de
tejido linfático resulta en la formación de las amígdalas palatinas. (Sin referenciar).

17)Un hombre de 17 años de edad es examinado por un médico, quien


señala una masa en la parte posterior de la lengua del joven. El
médico hace una biopsia de la masa, y el informe de patología regresa
con un diagnóstico de tejido tiroideo normal. ¿La presencia ocasional
de dicho tejido en la parte posterior de la lengua está relacionada con
el origen embrionario de la tiroides, cerca de cuál de las siguientes
estructuras?
a) Primera bolsa faríngea.
b) Agujero ciego.
c) Conducto nasolagrimal.
d) Segundo arco faríngeo.
e) Tercera bolsa faríngea.

La respuesta es B. El primordio tiroideo aparece en el embrión como una


depresión del piso faríngeo, ubicada entre la 1 er y 2da bolsas faríngeas,
constituido por una proliferación endodérmica. Esta depresión se reconoce en
el adulto como el agujero ciego (foramen caecum) de la base de la lengua
ubicada en el vértice de la V lingual. Durante el desarrollo, la tiroides desciende
frente a la faringe, manteniendo una conexión con la lengua a través del ducto
tirogloso. Usualmente el ducto tirogloso desaparece. Infrecuentemente, tejido
ectópico tiroideo residual puede ser dejado en cualquier localización del
camino, incluyendo la parte posterior de la lengua. En casos extraños, todo el
tejido tiroideo permanece en este lugar, formando una masa. A es incorrecta la
primera bolsa faríngea desarrolla el oído medio y las trompas de Eustaquio. C
es incorrecta el conducto nasolagrimal conecta los ojos con la boca. D es
incorrecta del segundo arco faríngeo se desarrollan músculos de la expresión
facial, el proceso estiloides del hueso temporal, estribo, ligamento estilo
hioideo, asta menor del hioides. E es incorrecta de la tercera bolsa faríngea se
desarrollan el timo y las glándulas paratiroides inferiores. (Sin referenciar).

18)¿Cuál de los siguientes ocurre como resultado de una anormalidad


embriológica en el desarrollo tiroideo?
a) Lóbulo piramidal
b) Quiste del conducto tirogloso
c) Tiroides lingual
d) Todas las anteriores
La respuesta es D. La glándula tiroides se origina en la base de la lengua en el
orificio del agujero ciego y migra caudalmente para localizarse en la parte inferior
del cuello. Las anormalidades de este desarrollo son el lóbulo piramidal, el quiste
del ducto tirogloso y la tiroides lingual. (Sin referenciar).

19)Es cierto respecto a las glándulas paratiroides:


a) Se desarrollan a partir del segundo y el tercero sacos faríngeos, junto con la
amígdala palatina y el timo.
b) Emigran en sentido caudal en el cuello durante el desarrollo normal pero
pueden encontrarse en cualquier parte de la mucosa faríngea hasta el
mediastino profundo.
c) Secretan hormona paratiroidea y calcitonina para realizar la homeostasis
del calcio.
d) Por lo general son cuatro, pero a menudo sólo hay dos o tres.
e) Contienen enzimas que catalizan la conversión de 25(OH) vitamina D3 en
1,25(OH)2vitamina D3.

La respuesta es B. A es incorrecta las glándulas paratiroides se desarrollan a


partir del tercero y cuarto sacos faríngeos, junto con el timo y el cuerpo último
bronquial tiroideo. C es incorrecta la calcitonina es secretada por las células C de
la glándula tiroides. D es incorrecto la mayoría de las personas tiene cuatro
glándulas paratiroideas aunque pueden tener 5 o más. E es incorrecto estas
enzimas se encuentran en el hígado. (Sin referenciar).

20)Una niña de 2 meses es llevada al hospital por infecciones recurrentes


desde el nacimiento. El examen físico muestra orejas bajas, un filtrum
corto, mandíbula pequeña, membrana timpánica inflamada, y temperatura
corporal elevada. Estudios de laboratorio muestran niveles de calcio
reducidos por la presencia de más células secretoras de calcitonina que
de células secretoras de paratiroides. Las células secretoras de
calcitonina se desarrollan de cuál de las siguientes opciones.
a) Endodermo del foramen cecum.
b) Endodermo de la tercera bolsa faríngea.
c) Células de la cresta neural que migran hacia el tercer arco faríngeo
d) Ectodermo oral.
e) Cuerpo ultimobranquial.

La respuesta es E. Las células secretoras de calcitonina se desarrollan del


cuerpo ultimobranquial, remanente de la quinta bolsa faríngea, luego migran hacia
la línea media en un lugar parafolicular en la glándula tiroides. El endodermo del
foramen cecum (opción A) da lugar a las células foliculares de la tiroides. El
endodermo de la tercera bolsa faríngea (opción B) da lugar al timo y la glándula
parótida inferior. Células de la cresta neural que migran al tercer arco faríngeo
(opción C) forman un molde cartilaginoso para el hasta mayor y cuerpo inferior del
hueso hioides. Ectodermo oral (opción D) forma la cubierta externa de las
hendiduras faríngeas que separan cada arco faríngeo. El meato auditivo externo
es el remanente posnatal de la primera hendidura faríngea, y no es lugar para el
desarrollo de células linfáticas o glandulares. (Sin referenciar).

21)¿Cuál de los siguientes tejidos u órganos hematopoyéticos se desarrolla


a partir del endodermo?
a) Timo
b) Amígdalas
c) La médula ósea
d) Bazo
e) Islas de sangre

La respuesta es a. El parénquima del timo (células epiteliales) se desarrollan a


partir del endodermo del tercer arco faríngeo (branquiales). El rudimento del timo
es invadido por células óseas derivadas de la médula precursores de linfocitos al
inicio del tercer mes de desarrollo. Las amígdalas (respuesta b) desarrollan
nódulos linfáticos como parcialmente encapsulados. Su marco del parénquima se
deriva de mesodermo faríngeo. Huesos, por supuesto, si formado por la
osificación intramembranosa o endocondral, se derivan del mesodermo. Sus
cavidades de la médula están pobladas por células madre hematopoyéticas
(respuesta c) a partir del segundo mes de vida fetal. La cápsula de tejido conectivo
y el marco esquelético del bazo se desarrollan a partir de la placa esplácnica
lateral del mesodermo durante la quinta semana y son invadidos rápidamente por
las células hematopoyéticas del linaje mieloide (respuesta d). Sigue siendo un
órgano hematopoyético activo al menos hasta el séptimo mes en el útero. Islas de
sangre se desarrollan por la diferenciación de las células mesodérmicas en el
mesodermo extraembrionario que recubre el saco vitelino durante la tercera
semana del desarrollo fetal (respuesta e). Ellas dan lugar a los vasos vitelinos y
son el principal sitio de formación de glóbulos rojos en el embrión temprano. (Sin
referenciar).

22)Un hombre de 45 años visita al médico por cefalea y dolor mandibular


frecuente. Él dice que escucha un click en su mandíbula cuando mastica.
Examen físico muestra dolor en los músculos maseteros. ¿Cuál de las
siguientes estructuras embrionarias son las responsables de la
formación de estos músculos?
a) Primer arco faringeo
b) Segundo arco faringeo
c) Tercer arco faringeo
d) Cuarto arco faringeo
e) Quinto arco faringeo

La respuesta correcta es A. El primer arco faríngeo da camino a la formación de


los músculos de masticación (los maseteros, pterigoideo lateral, pterigoideo
medial, y temporal), el musculo milohioideo, el cuerpo anterior del musculo
digástrico, el musculo tensor del velo del paladar, y el musculo tensor del tímpano.
Estos son los músculos inervados por las ramas maxilares y mandibulares del
nervio trigémino (v2 y v3). Este paciente muy probablemente tiene irritación de la
articulación temporomandibular (TMJ – Temporomandibular joint dysfunction),
caracterizado por sensación de click durante masticación. (Sin referenciar).

23)¿En cuál de las siguientes bolsas faríngeas se desarrolla la tonsila


palatina?
a) Primera.
b) Segunda.
c) Tercera.
d) Cuarta.
e) Quinta.

La respuesta es B. El recubrimiento epitelial de la segunda bolsa faríngea se


origina dentro del mesénquima para formar la tonsila palatina. Parte de la bolsa
persiste en el adulto como la fosa tonsilar. A es incorrecto la primera bolsa
faríngea da lugar a la cavidad primitiva del oído medio y la trompa de Eustaquio. C
es incorrecto la Tercera bolsa faríngea da lugar al timo y las glándulas
paratiroideas inferiores. D es incorrecto la cuarta bolsa faríngea da origen a las
glándulas paratiroides superiores. Es importante recalcar que el desarrollo
anormal de la tercera y cuarta bolsa faríngea da lugar al síndrome de Digeorge,
que consiste en hipocalcemia y una alteración de la inmunidad celular con la
consecuente susceptibilidad a enfermedades virales y micóticas. E es incorrecto la
quinta bolsa faríngea origina las células C de la glándula tiroides del cuerpo
ultimobranquial, las cuales secretan calcitonina, hormona que disminuye el calcio
sérico, aunque esta bolsa sea considerada más parte de la cuarta bolsa. (Sin
referenciar).

24)Son afirmaciones correctas sobre las bolsas viscerales, excepto:


a) El mesodermo que las conforma proviene del mesodermo lateral definitivo
esplácnico
b) Todas derivan principalmente del intestino anterior
c) Las células con determinación ecto-endodérmica forman evaginaciones a
manera de hendiduras
d) Todas tienen un componente mesodérmico proveniente de crestas neurales
y somitas occipitales
e) El ecto-endodermo hace invaginaciones a manera de arcos

La respuesta es D. Las bolsas branquiales, faríngeas o viscerales; hacen parte


del aparato faríngeo o branquial, participan en la formación de cabeza, el cuello e
inician su desarrollo en la cuarta semana. Los arcos faríngeos están compuestos
por endodermo, mesénquima y ectodermo, durante la cuarta semana las células
de la cresta neural migran hacia el primer arco faríngeo produciendo las
prominencias maxilares y mandibulares. La musculatura esquelética y el endotelio
vascular se derivan del mesénquima original de los arcos faríngeos. Keith L.
Moore & T.V.N. Persaud. (2007). Embriología clínica: El desarrollo del ser
humano. (7°.). (pp. 202). Barcelona, España: Elsevier.

APARATO GENITOURINARIO

1) Una niña de 9 años de edad, con buen estado de salud, es sometida a un


examen médico de rutina. El médico nota que la niña no ha entrado
todavía en la etapa de la pubertad. ¿En qué etapa meiótica entonces
deberían estar la mayoría de los ovocitos en el ovario de la niña?
a) Anafase de la segunda división meiótica.
b) Metafase de la primera división meiótica.
c) Metafase de la segunda división meiótica.
d) Profase de la primera división meiótica.
e) Telofase de la primera división meiótica.

La respuesta es D. La primera división meiótica es la división meiótica de


“reducción”, en la cual el complemento diploide de ADN se reduce a un
complemento haploide. Las mayores partes de los ovocitos en mujeres pre
menopáusicas, niñas y bebés son arrestados en la Profase de la primera división
meiótica. Las mujeres posmenopáusicas tienen pocos ovocitos viables. (Sin
referenciar).

2) Se observó que un niño varón recién nacido tiene hipospadias. Al


realizarle una evaluación completa, se determina que el niño no tiene
más anomalías. Sin embargo si se realiza una reparación de la
hipospadias ¿Qué secuela puede ocurrir?
a) Extrofia vesical.
b) Hidrocele.
c) Fimosis.
d) Quistes de uraco.
e) Infección del tracto urinario.

La respuesta es E. La Hipospadias, que es el desplazamiento congénito de la


abertura de la uretra en la superficie ventral (inferior) del pene, se debe a una
malformación de la ranura de la uretra y del canal. La Hipospadias frecuentemente
acompaña otras anomalías genitourinarias, especialmente la criptorquidia. Cuando
se aísla la hipospadias es reparada porque la abertura anormal es a menudo
restringida, lo que conduce a la retención urinaria y las infecciones ascendentes
del tracto urinario. Otra consecuencia importante de la hipospadias es la
esterilidad, la cual ocurre cuando la abertura está demasiado cerca de la base del
pene para permitir la eyaculación normal. Aunque la hipospadias puede ocurrir de
forma aislada, está fuertemente asociada con otras anomalías urogenitales. A es
incorrecto la extrofia vesical es una malformación congénita sin ninguna relación,
en la que la pared abdominal anterior y la pared de la vejiga se forman de modo
incompleto, y la mucosa de la vejiga se expone hacia el medio ambiente. B es
incorrecto Hidrocele es una acumulación serosa en la túnica vaginal, a menudo
produciendo un masa. Hidrocele escrotal evidente se presenta generalmente sin
ninguna causa obvia. C es incorrecto Fimosis es la condición en la que el prepucio
no puede ser retraído sobre la cabeza del pene. Por lo general, ya sea una
malformación congénita o una consecuencia de la cicatrización. D es incorrecta
los Quistes de Uraco son producidos por las zonas de epitelio Uraco persistente,
que pueden estar presentes en cualquier lugar entre la vejiga y el ombligo. (Sin
referenciar).

3) La evaluación de los niveles de un antígeno prostático específico (PSA)


en un hombre de 54 años es parte de la rutina de salud. ¿Cuál de las
siguientes estructuras embriológicas da lugar al órgano que inicia el
carcinoma?
a) Tubérculo genital.
b) Proceso vaginal.
c) Rete testis.
d) Túnica albugínea.
e) Seno urogenital.

La respuesta es E. La próstata se deriva inmediatamente de la uretra prostática la


cual a su vez se deriva desde el seno urogenital, se recomienda que los niveles de
PSA (Antígeno Prostático Específico) sean medidos anualmente en hombres
mayores de 50 como tamizaje para carcinoma prostático (y registrar un nivel
básico). A es incorrecto el tubérculo genital da origen a las glans penis. B es
incorrecto el proceso vaginal es una extensión celómica dentro de la protuberancia
escrotal que forma el canal inguinal durante el descenso de los testículos. C es
incorrecto la rete testis conecta con varios túbulos mesonéfricos, los cuales se
transforman en los conductos deferentes de los testículos. D es incorrecto la
túnica albugínea se deriva del mesénquima y condensa para formar la cápsula de
tejido conectivo fibroso de la testis. (Sin referenciar) .

4) Una pareja casada, con diagnóstico de infertilidad de 6 años de duración,


se presenta a la clínica de fertilidad. Un espermocitograma, confirmado
por microscopía electrónica, revela que el marido produce todos los
espermatozoides con cabezas redondeadas, una condición conocida
como globozoospermia. La estructura de los espermatozoides que falta
se asocia con ¿cuál de las siguientes?
a) Pérdida de factor de capacitación
b) La retención de las espermátidas en desarrollo en células de Sertoli
c) La maduración de las enzimas líticas
d) La actividad mitótica
e) Divisiones meióticas

La respuesta es C. El hombre del escenario tiene espermatozoides que carecen


del acrosoma (globozoospermia) y por lo tanto las enzimas necesarias para la
penetración del óvulo faltan. La formación del acrosoma, un gránulo secretor
especializado, es uno de muchos eventos que ocurren durante la maduración de
la espermatogénesis (el proceso por el cual maduran los espermatozoides que se
forman a partir de las espermátidas). La formación del acrosoma implica la
maduración de enzimas líticas y se produce después de la división de
espermatocitos secundarios. No implica la actividad mitótica o meiótica
(respuestas D y E). El acrosoma se desarrolla a partir de vesículas de Golgi como
cualquier otro gránulo secretor. Contiene acrosina, una serina proteasa,
hialuronidasa, y la neuraminidasa, responsable de la capacidad de penetración de
los espermatozoides. Las células en desarrollo están en contacto con las células
de Sertoli para todas las etapas de la espermiogénesis. Al final de la
espermiogénesis, las espermátidas son liberadas por las células de Sertoli en un
proceso llamado espermiación (respuesta b). Los factores de capacitación no
están involucrados en la maduración acrosomal (respuesta a). (Sin referenciar) .

5) Una mujer de 38 años de edad, se casó recientemente y quedó


embarazada. El cigoto es el resultado de un óvulo fecundado por un
espermatozoide haploide de su marido. ¿Cuál de los siguientes es
necesario para la continuación de la segunda división meiótica para
producir el óvulo haploide?
a) La elevación de los títulos de progesterona
b) Expulsión del folículo maduro
c) El ambiente del oviducto y el útero
d) La fecundación por un espermatozoide
e) La presencia de gonadotropina coriónica humana (hCG)

La respuesta es D. El ovocito secundario entra en la segunda división meiótica


justo antes de la ovulación y se detiene en la metafase. La fecundación por un
espermatozoide proporciona la estimulación de la división de la cromatina al
número haploide. En el momento en que el óvulo fecundado llega al útero, la
progesterona (opción a) producida por el cuerpo lúteo ha iniciado la fase secretora
en el endometrio. Una vez se produce la implantación y el corion se desarrolla, la
gonadotropina coriónica humana (hCG) es sintetizada y el cuerpo lúteo se
mantiene (opción e). La expulsión del folículo (opción b) y el medio ambiente del
oviducto y el útero (la opción c) no provocan la segunda división meiótica. (Sin
referenciar).

6) Una pareja casada, con diagnóstico de infertilidad se presenta a la clínica


de fertilidad. El semen del marido sólo contiene 40 millones de esperma
total con un índice de motilidad progresiva ligeramente por debajo de lo
normal. Un ensayo de penetración de huevo de hámster se realizó en la
que se recogen los huevos de hámster y su zona pelúcida se elimina
enzimáticamente antes de la mezcla con el esperma. El esperma del
marido tienen una morfología normal, pero sus resultados del ensayo de
penetración son el 3,7% (normal, 10%). El ensayo de penetración en el
huevo de hámster, requiere cuál de las siguientes:
a) La formación de esperma, la maduración y la penetración
b) La adición de colesterol a la membrana plasmática del espermatozoide
c) Una disminución en la fluidez de la membrana plasmática del
espermatozoide
d) El secuestro de enzimas acrosomal
e) Capacitación, reacción del acrosoma y la penetración

La respuesta es e. Se requiere La capacitación, la reacción del acrosoma y la


penetración para el ensayo de penetración de los espermatozoides (SPA) en el
huevo de hámster. La capacitación prepara al esperma para la fertilización y
requiere un aumento en la fluidez de la membrana plasmática del espermatozoide.
El esperma debe residir en el tracto reproductivo femenino o bajo apropiadas
condiciones in vitro durante aproximadamente 1 hora para que se produzca la
capacitación. Durante la capacitación se produce una pérdida de factores de
capacitación que se han añadido a los espermatozoides del epidídimo por las
células y los órganos reproductores masculinos accesorios. Se elimina el
colesterol (no añadido, la respuesta b) de la membrana plasmática del
espermatozoide durante este período, lo que resulta en el aumento de la fluidez
(no una disminución, la respuesta c) de membrana que se requiere para la fusión
de la membrana acrosomal con la membrana del esperma. A continuación, hay
liberación de las enzimas acrosomales (no de secuestro, la respuesta d), que son
necesarias para la descomposición de la corona radiada y la zona pelúcida del
ovocito para facilitar la penetración del esperma. Formación y maduración del
esperma se producen en los testículos y el epidídimo (respuesta a) y por lo tanto
no se prueban directamente en un ensayo de penetración de huevo de hámster.
(Sin referenciar).

7) Una pareja está tratando de concebir un hijo. Después de la


relación sexual, ¿cuál de las siguientes es responsable de la
prevención de poliespermia?
a) Reanudación de la primera división meiótica
b) La reanudación de la segunda división meiótica
c) Capacitación
d) La zona de reacción
e) La liberación de enzimas del acrosoma de esperma.

La respuesta es D. En la fusión del primer espermatozoide con la membrana


celular de los ovocitos, el contenido de los gránulos secretores almacenados justo
debajo de la membrana del ovocito (gránulos corticales) se liberan (la reacción de
la zona). Las enzimas almacenadas en esos gránulos causan cambios
bioquímicos y eléctricos en la zona pelúcida y la membrana de los ovocitos que
impiden la unión de espermatozoides adicionales. Las células germinales
femeninas primitivas (ovogonias) entran en la primera división meiótica durante el
desarrollo fetal (respuesta a). Este proceso se detiene en la profase I hasta que
los ovocitos primarios individuales son hormonalmente inducidos a reanudar la
primera división meiótica durante la pubertad y la edad adulta temprana
(menarquia a la menopausia). La fusión de las membranas del espermatozoide y
los ovocitos inicia la reanudación de la segunda división meiótica, lo que resulta en
la formación de un pronúcleo haploide en el ovocito y la expresión del segundo
cuerpo polar (respuesta b). La capacitación (respuesta c) es un proceso mediante
el cual las secreciones enzimáticas del útero y los oviductos extraen glicoproteínas
de la membrana de la célula del espermatozoide. Esto es necesario para la
penetración de la capa de células que rodean el ovocito (corona radiata). La
liberación de enzimas (respuesta e) de la tapa acrosomal del espermatozoide (un
lisosoma ampliado) resulta en la digestión de la zona pelúcida que rodea al
ovocito, lo que permite la penetración del espermatozoide. (Sin referenciar) .

8) Un hombre de 26 años de edad, contrajo la gripe viral con una fiebre


continua de 39,5 ° C (103 ° F) durante 3 días. Debido a que la
espermatogénesis no puede ocurrir por encima de una temperatura
escrotal de 35,5 ° C (96 ° F), se quedó sin esperma viable después de su
recuperación. Aproximadamente, ¿cuánto tiempo se requiere para el
regreso de los espermatozoides viables al epidídimo?
a) 3 días
b) 1 semana
c) 5 semanas
d) 2 meses
e) 4 meses

La respuesta es D. En el hombre, el tiempo requerido para la progresión de


espermatogonias a espermatozoides móviles es de aproximadamente 2 meses
(61 a 64 días). La espermatogénesis, el proceso por el cual las espermatogonias
son sometidas a la división mitótica para producir espermatocitos primarios, se
produce a 1° C (2° F) por debajo de la temperatura normal del cuerpo. Las
Divisiones meióticas posteriores producen espermatocitos secundarios con un
número de cromosomas haploides bivalente y luego espermátidas con un número
de cromosomas haploides monovalente. Espermiogénesis, la maduración del
espermátide, da lugar a los espermatozoides. Los espermatozoides adultos se
mueven al epidídimo, donde se convierten en totalmente móviles. (Sin referenciar) .

9) Las células germinales primordiales que finalmente forman las oogonias


y espermatogonias se originan en cuál de las siguientes?
a) Mesenterio dorsal del intestino posterior
b) Cresta gonadal
c) Revestimiento endodérmico del saco vitelino
d) Cordones sexuales primarios de la gónada en desarrollo
e) Corion
La respuesta es c. Las células germinales primordiales se observan primero en el
revestimiento endodérmico de la pared del saco vitelino (derivado del hipoblasto)
al final de la tercera o comienzo de la cuarta semana en la región del alantoides.
Durante el plegamiento embrionario, la parte dorsal del saco vitelino se incorpora
en el embrión como el intestino primitivo. Las células germinales primordiales
migran posteriormente a lo largo del mesenterio dorsal del intestino posterior
(respuesta A) y en la cresta gonadal (genital) por la semana 6 (respuesta b). Los
cordones sexuales primarios crecen hacia el mesénquima subyacente a la cresta,
y las células germinales primordiales se incorporan a los cordones sexuales
primarios (respuesta d). El corion (respuesta e) es la membrana más externa del
feto y se compone de mesodermo extraembrionario somático, citotrofoblasto y
sincitiotrofoblasto. Se divide en el corion frondoso, donde forma vellosidades y
prolifera, y el corion liso, también conocido como el corion leve. El siguiente
diagrama muestra la disposición de las membranas fetales. (Sin referenciar) .

10)¿Cuáles de las siguientes estructuras están revestidas con epitelio


derivado del mesodermo de la yema uretérica?
a) Cápsula de Bowman.
b) Túbulo contorneado proximal.
c) Asa de Henle.
d) Túbulo contorneado distal.
e) Uréter.
La respuesta es E. El epitelio transicional que recubre el uréter, la pelvis renal y
los cálices mayores y menores derivan del mesodermo de la yema uretérica. De
forma similar el epitelio cuboidal de los túbulos colectores deriva del mesodermo
de la yema uretérica. A es incorrecto el epitelio escamoso simple que recubre la
cápsula de Bowman deriva del mesodermo de la vesícula metanéfrica. B es
incorrecto el epitelio cuboidal simple que limita el túbulo contorneado distal es
derivado del mesodermo de la vesícula metanéfrica. C es incorrecto el epitelio
escamoso simple que recubre el asa de Henle deriva del mesodermo de la
vesícula metanéfrica. D es incorrecto el túbulo contorneado proximal es un epitelio
columnar simple que deriva del mesodermo de la vesícula metanéfrica. (Sin
referenciar).

11)Un investigador emplea mutagénesis dirigida para producir mutaciones


en un gen que es importante para el desarrollo de la yema ureteral en un
embrión de rata. El embrión se desarrolla anormalmente, y se muere
veinte días después. ¿El examen del feto muerto podría revelar ausencia
de cuál de las siguientes estructuras renales?
a) Asa de Henle ascendente.
b) La cápsula de Bowman.
c) Sistema colector.
d) Asa de Henle descendente.
e) Penacho glomerular.

La respuesta es C. El sistema colector, incluyendo los ductos colectores, cálices


menores y cálices mayores además de la pelvis renal se derivan del brote uretral.
A, B, D, E son incorrectas estas estructuras listadas se derivan del mesodermo
metanefrico. (Sin referenciar).

12)El examen físico de un joven varón, revela descarga de orina desde el


ombligo. El médico concluye que el uraco ha fallado en fusionarse. ¿Cuál
de las siguientes estructuras es el remanente adulto normal del uraco
fusionado?
a) Pliegue umbilical lateral
b) Pliegue umbilical medial
c) Ligamento umbilical medial
d) Pliegue umbilical medio
e) Ligamento umbilical medio

La respuesta es E. El uraco es el derivado del ducto alantoico, que pasa desde


el seno urogenital al cordón umbilical. Normalmente, éste ducto se fusiona y deja
de ser evidente. El derivado adulto es el ligamento umbilical medio, que se
encuentra en la línea media a lo largo de la superficie de la pared abdominal. Pasa
desde el extremo superior de la vejiga hasta el ombligo. A es incorrecto el pliegue
umbilical lateral es el pliegue del peritoneo parietal que cubren la arteria y la vena
epigástrica inferior en el interior de la superficie de la pared del muro abdominal
anterior. B es incorrecto el pliegue umbilical medial es el pliegue del peritoneo
parietal que cubre el ligamento umbilical medial, en el adulto éste deriva de la
arteria umbilical. C es incorrecto el ligamento umbilical medial es el derivado
adulto de la porción distal de la arteria umbilical. La arteria umbilical se origina de
la Arteria iliaca interna. Pasa a lo largo de la vejiga luego por la pared abdominal
anterior para alcanzar el ombligo. Prenatalmente, esta arteria lleva la sangre fetal
hacia la placenta, donde gana oxígeno y nutrientes. Postnatalmente, la parte
proximal de la arteria umbilical queda abierta y provee de sangre a la superficie
superior de la vejiga. Distal a la vejiga, la arteria se vuelve fibrotica y es conocida
como el ligamento umbilical medial. D es incorrecto el pliegue umbilical medio es
el pliegue del peritoneo parietal que cubre el ligamento umbilical medio, que en el
adulto es el uraco. (Sin referenciar).

13)Un hombre recién nacido se encuentra con aberturas ureterales a lo largo


de la superficie ventral del pene. El médico explica a sus padres que las
estructuras bilaterales que deberían haberse fusionado en la línea media
no lo hicieron completamente, lo que resultó en éste defecto. Los padres
están muy preocupados, pero el médico les asegura que éste puede ser
corregido quirúrgicamente con facilidad. ¿Cuál de las siguientes
estructuras en una mujer normalmente se desarrollan de las mismas
estructuras que fallaron en fusionarse en éste niño?
a) Ligamentos redondos del útero.
b) Pilares del Clítoris (CrusClitoridis).
c) Labios mayores.
d) Labios menores.
e) Ligamentos ováricos.

La respuesta es D. Los pliegues uretrales en la mujer no se fusionan y


desarrollan los labios menores. El espacio entre los pliegues se convierte en el
vestíbulo de la vagina. En el hombre, los pliegues uretrales normalmente se
fusionan para convertirse en la superficie ventral del pene. Una falla en la fusión
normal de estos pliegues en el hombre resulta en hipospadias, la presencia de
aperturas de uretra sobre la superficie ventral del pene. A es incorrecta los
ligamentos redondos del útero son los remanentes adultos en la mujer de la
porción caudal de los ligamentos genitales caudales. Los ligamentos redondos del
útero en la mujer unen el útero a la fascia de los labios mayores. Los ligamentos
redondos pasan a través del canal inguinal en la mujer. B es incorrecta los pilares
del clítoris son 2 estructuras eréctiles que se fusionan en el cuerpo del clítoris
formando una V. C es incorrecto los labios mayores en la mujer se desarrollan de
evaginaciones genitales (pliegues labioescrotales). En el hombre, las mismas
estructuras se fusionan en la línea media para formar el escroto. E es incorrecta
los ligamentos ováricos en la mujer se desarrollan de la porción rostral del
ligamento genital caudal. El ligamento ovárico conecta el ovario al útero. En el
hombre, la misma estructura forma la porción rostral del gubernáculo testicular. En
la mujer, éste ligamento se convierte en dos estructuras: el ligamento ovárico y el
ligamento redondo del útero. (Sin referenciar) .

14)Un recién nacido a término de 8 libras nació por vía vaginal de una madre
de 36 años de edad. En el parto se evidenció un escroto anormalmente
grande. El ginecobstetra de turno palpó el escroto agrandado y determinó
que ambos testículos estaban presentes. Cuando el ginecobstetra
presiona suavemente sobre el abdomen del recién nacido el escroto se
hincha aún más. ¿Qué condición congénita anotó el ginecobstetra en la
historia?
a) Hernia abdominal.
b) Criptorquidia.
c) Varicocele.
d) Hidrocele.
e) Hernia femoral.

La respuesta es D. El cuadro descrito es compatible con hidrocele. Los testículos


se desarrollan en la pared abdominal posterior y son halados hacia abajo y afuera
de la cavidad abdominal por el gubernáculo. El descenso final a través del canal
inguinal generalmente no ocurre hasta el noveno mes in útero. Antes el testículo
persiste siendo un órgano retroperitoneal, el espacio se conecta a la cavidad
abdominal por el proceso vaginalis. B es incorrecta en éste caso los testículos han
migrado satisfactoriamente dentro del escroto en ambos lados por lo que no hay
criptorquidia, pero en su lugar hay una falla en el cierre del proceso vaginal con el
consecuente hidrocele, claro está que el proceso vaginal puede obliterarse al
nacimiento o normalmente se resuelve por sí solo luego de 2-3 meses sin
intervención. A es incorrecto una hernia es un defecto en la pared abdominal
anterior y no está presente en la clínica del caso presentado. C es incorrecta el
cuadro de varicocele consiste en dilataciones venosas del plexo pampiniforme y
provocan estasis de la sangre alrededor del testículo generalmente izquierdo y
usualmente se presenta como una masa escrotal azulada. E es incorrecta las
hernias femorales son defectos en las estructuras que conforman el anillo crural,
el ligamento inguinal (De Poupart), el ligamento de Cooper, cintilla ileopectínea, y
ligamento de Gimbernat.
15)El remanente de la cavidad abdominal presente en el Escroto adulto es el:
a) Gubernáculo. (Gubernaculumtestis)
b) Conducto deferente.
c) Cordón espermático.
d) Túnica Vaginal.
e) Ninguna de las anteriores.

La respuesta es A. El Gubernáculo. (Gubernaculum testis) es una condensación


mesenquimatosa rica en matrices extracelulares esta banda termina en la región
inguinal entre los músculos abdominales oblicuo interno y externo e inicia el
crecimiento de su porción extraabdominal apoyando el descenso del testículo
hacia la bolsa escrotal. B es incorrecto los conductos deferentes derivan de los
conductos mesofrenicos. C es incorrecta la túnica vaginal es la cubierta serosa del
testículo. Y D es incorrecta.

16)El sistema uro-genital proviene de la interacción:


a) Ectodermo epitelial-endodermo intra-embrionario
b) Mesodermo intermedio-mesodermo lateral definitivo esplácnico
c) Mesodermo intermedio-mesodermo lateral definitivo somático
d) Endodermo intra-embrionario-mesodermo lateral definitivo esplácnico
e) Endodermo intra-embrionario-mesodermo lateral definitivo somático

Respuesta: B. Mesodermo intermedio-mesodermo lateral definitivo esplácnico.


Tanto el sistema urinario como el genital se desarrollan a partir de una cresta
mesodérmica común, el mesodermo intermedio. Por otro lado el mesodermo
lateral de carácter esplácnico forma la membrana serosa alrededor de cada
órgano de dicho aparato (Embriología de Langman, págs. 235, 77-79).

17)El tubérculo genital en las mujeres forma:


a) Clítoris
b) Vagina
c) Trompas
d) Útero
e) Labios mayores

Respuesta: A. Clitoris. Los estrógenos estimulan el desarrollo de los genitales


externos del sexo femenino. De esta manera el tubérculo genital se alarga
ligeramente y forma el clítoris y los pliegues uretrales no se fusionan y se
convierten en los labios menores. Las protuberancias genitales se agrandan y
forman los labios mayores (Embriología de Langman, pág 258).
18)Un niño se presenta con rasgos faciales aplanados, orejas de
implantación baja y deformidades en los pies. Los pulmones se
encuentran en subdesarrollo. El embarazo fue complicado por la
presencia de oligohidramnios. La causa más probable de esta condición
es una malformación de:
a) Conductos paramesonéfricos.
b) Pronefros.
c) Yemas ureterales.
d) Seno urogenital.
e) Septum urorectal.

La respuesta es C. Este niño sufre de síndrome de Potter, causado por la


ausencia de ambos riñones. Esto a menudo se debe a un error del desarrollo de
las yemas ureterales. Las yemas uretrales forman los uréteres, la pelvis renal,
cálices, y túbulos colectores. A es incorrecto, los conductos paramesonéfricos
forman las trompas de Falopio, el útero y el tercio superior de la vagina. B es
incorrecto, el pronefros es una colección transitoria, que desaparece durante la
cuarta semana del desarrollo. No forma parte del riñón permanente. D es
incorrecto, el seno urogenital forma la vejiga (en ambos sexos). También forma la
uretra prostática, próstata, la uretra membranosa, glándulas bulbo-uretrales, la
uretra peneana (parte proximal) y las glándulas de Littré en el varón. En la mujer
forma la mayor parte de la uretra, las glándulas uretrales, las glándulas
parauretrales, el vestíbulo de la vagina y glándulas vestibulares mayores. E es
incorrecto, el Septum Urorectal divide la cloaca en el seno urogenital y el canal
anorectal que formara el recto y el canal anal superior. (Sin referenciar).

19)A un neonato con múltiples anomalías congénitas se le encuentra


ausencia completa de uno de los riñones, las otras estructuras
genitourinarias se encuentran intactas. Este es el resultado más probable
de cuál de las siguientes opciones:
a) Ausencia del brote uretral.
b) División completa del brote uretral.
c) Ectopia renal cruzada.
d) Doble brote uretral.
e) División incompleta del brote uretral.

La respuesta: A. La agenesia renal unilateral es una anomalía congénita común


(1 en 1000 nacimientos) que puede ser completamente asintomática, o puede,
como en este caso, ser identificada durante la evaluación de otras anomalías
congénitas. Una sola arteria umbilical, puede de hecho estar presente como una
pista clínica. La agenesia renal unilateral es usualmente el resultado de un fallo en
el desarrollo del brote uretral. B es incorrecta la división completa del brote uretral
produce un uréter bífido con doble riñón en uno de los lados. C es incorrecta la
ectopia renal cruzada produce riñones fusionados en uno de los lados. D es
incorrecta el doble brote uretral produce un riñón supernumerario. E es incorrecta
la división incompleta del brote uretral produce un uréter bífido con un riñón
parcialmente dividido. (Sin referenciar).

20)Un joven de 15 años es remitido a un endocrinólogo reproductivo por


desarrollo sexual anormal. El examen físico revela testículos pequeños e
inmaduros. Estudios adicionales revelan que los testículos no secretan
testosterona o factor de regresión mülleriana. Estudios cromosómicos
revelan un cariotipo 46 XY. ¿Cuál de las siguientes opciones describe
mejor el desarrollo del sistema reproductor in útero de éste individuo?
a) Tractos reproductivos internos tanto masculinos como femeninos y
genitales externos masculinos.
b) Tracto reproductivo interno femenino y genitales externos femeninos.
c) Tracto reproductivo interno femenino y genitales externos masculinos.
d) Tracto reproductivo interno masculino y genitales externos femeninos.
e) Tracto reproductivo interno masculino y genitales externos masculinos.

La respuesta es B. La descripción concuerda con una disgenesia gonadal, en la


ausencia de testosterona, los ductos Wolffianos harán regresión y fallarán en
diferenciarse en los tractos reproductivos internos masculinos normales. En la
ausencia del factor de regresión mülleriano, los ductos müllerianos se
diferenciarán automáticamente en oviductos y útero. La diferenciación de los
genitales externos masculinos es dependiente de la adecuada dihidrotestosterona
(a través de la acción de la 5α-reductasa en testosterona). En la ausencia de
testosterona, se desarrollarán genitales externos del tipo femenino. A es incorrecta
la disgenesia selectiva de las células de Sertolli podría producir la situación
descrita. Las células de Leydig normales podrían secretar la testosterona y
producir los tractos internos masculinos normales, y los genitales externos
masculinos. Sin embargo, la ausencia de factor de regresión mülleriana, que es
secretado por las células de Sertolli, permitiría la formación de un tracto interno
femenino también. C es incorrecta un tracto reproductivo femenino y genitales
externos de tipo masculino no es posible que sean formados bajo ninguna
circunstancia. D es incorrecta esta situación puede ocurrir con una deficiencia de
5α-reductasa. Los tractos masculinos internos normales podrían ser formados ya
que no hay requerimiento de dihidrotestosterona para ello. El factor de regresión
mülleriano prevendría la diferenciación de un tracto interno femenino, ya que la
diferenciación genital externa masculina depende de la dihidrotestosterona, la
deficiencia de 5α-reductasa estaría ligada a feminización. E es incorrecta la
situación expuesta es normal, y no ocurriría en el individuo descrito, quien tiene
disgenesia testicular. (Sin referenciar).

21)Un paciente que aparenta ser de sexo femenino, se encuentra en su


cariotipo 46 XY. La vagina del paciente es muy superficial, acabando en
una bolsa ciega, y hay masas palpables en los labios. Se hace el
diagnostico de síndrome de feminización testicular. ¿Cuál de las
siguientes opciones estuvo presente más probablemente durante el
desarrollo temprano de la vida fetal de éste individuo?
a) Vena ovárica.
b) Un útero.
c) Un oviducto.
d) Bajos niveles de testosterona.
e) MIF (Factor inhibidor mülleriano).

La respuesta es E. La feminización testicular es un desorden del receptor de


andrógenos. Fenotípicamente, el paciente es de sexo femenino, pero tiene una
vagina que acaba en ciego, y carece de útero u otros órganos reproductivos
femeninos internos. El paciente tiene un fenotipo XY. Dado que el gen para factor
determinante de los testículos (TDF) se encuentra en el cromosoma Y, TDF
provoca que la gónada indiferenciada se desarrolle en testículos conteniendo
células de Sertolli. Las células de Sertolli en éste estadio secretarán MIF, una
sustancia que suprime los ductos paramesonéfricos, previniendo la formación de
órganos reproductivos femeninos internos. A es incorrecta este paciente no tendrá
vena ovárica, un hallazgo propio del síndrome de Turner que está asociado a un
genotipo 45 XO. De hecho, el paciente tendrá testículos desde que el
complemento cromosómico contenga un cromosoma Y. Los testículos en
individuos con síndrome de feminización testicular a menudo no descendidos y
usualmente se remueven con cirugía. B, C son incorrectas el paciente no poseerá
útero o un oviducto ya que ambas estructuras son derivadas del ducto
paramesonefrico, el cual es suprimido por el MIF. D es incorrecto los niveles bajos
de Testosterona no se darían es éste paciente. De hecho, los individuos con
síndrome de feminización testicular tienen niveles de testosterona normales e
incluso ligeramente elevados. El desarrollo de genitales femeninos externos es el
resultado de un defecto en los receptores de andrógenos, no de bajos niveles de
testosterona. (Sin referenciar).

22)Un investigador está estudiando el desarrollo prenatal del riñón. En sus


estudios interrumpió selectivamente el desarrollo del riñón por medio de
la ablación microquirúrgica que típicamente induce el desarrollo del riñón
definitivo. ¿Cuál de las siguientes estructuras es la que lesionó más
probablemente?
a) Alantoides.
b) Ducto mesonefrico.
c) Mesonefros.
d) Metanefros.
e) Cresta urogenital.

La respuesta es D. El metanefros es un tejido derivado del mesodermo


metanefrico. Induce el desarrollo del riñón definitivo, que dará lugar a las unidades
excretoras. El metanefros, a su vez, forma los glomérulos, túbulos proximales, asa
de Henle y distales. A es incorrecta el alantoides, es un divertículo endodérmico
que se encarga de los desechos en etapa embrionaria con la formación de la
vejiga queda reducido y conectado con la vejiga por medio del uraco que se
oblitera postnatalmente y forma ligamento umbilical medio. Este se incorpora en
los senos urogenitales y contribuye a la formación de la vejiga urinaria. No está
involucrado en la formación del riñón. B es incorrecto el ducto mesonefrico o
conducto de Wolff es un ducto mesodérmico que se desarrolla en el conducto
deferente masculino, epidídimo, vesículas seminales, conducto eyaculador
también puede originar la yema ureteral encargada de la formación del sistema
urinario colector, túbulos colectores, cálices, pelvis, uréteres. C es incorrecto el
mesonefros es el riñón embrionario, que funciona en el embrión anterior al
desarrollo del metanefros, (o riñón definitivo). El mesonefros se origina del
mesodermo intermedio. E es incorrecto la cresta urogenital es una elevación
longitudinal del mesodermo intermedio formado por el mesonefros y la gónada.
(Sin referenciar).

23)¿Cuál o cuáles de las siguientes afirmaciones respecto de la embriología


del sistema reproductor en la mujer son correctas?
a) Los genitales externos se desarrollan a partir del tubérculo genital.
b) El sistema de Wolff experimenta regresión.
c) El sistema de Müller se fusiona en la línea media.
d) Los ovarios derivan del sistema mülleriano superior.
e) El testículo experimenta regresión a medida que se forma el sistema de
conductos femenino.

La respuesta es A, B, C. Los genitales externos se desarrollan a partir del


tubérculo genital, si no hay material genético para la formación de los testículos,
tampoco habrá andrógeno ni factor inhibitorio mülleriano producido por un
testículo. En estas circunstancias experimenta regresión del conducto de Wolff y el
sistema mülleriano crece y luego se fusiona en la línea media para formar la parte
superior de la vagina, el cuello uterino y el útero. Las estructuras intactas unitarias
que normalmente se ven sólo son posibles con la resorción del tabique de la línea
media. El ovario se forma tras la migración de las células germen primordial al
reborde genital y su: desarrollo subsiguiente en folículos primordiales. No hay
testículo ni experimenta regresión conforme se forma el sistema de conductos de
la mujer en la persona normal. (Sin referenciar).

24)Una niña recién nacida se trató clínicamente debido a varias anomalías


congénitas. Durante este estudio diagnóstico, se descubre que no se ha
producido un desarrollo normal de la vagina ni el útero en el infante. ¿El
desarrollo defectuoso del útero (agenesia) está directamente relacionado
con cuál estructura embrionaria?
a) Cresta urogenital
b) Conducto mesonéfrico
c) Conducto paramesonéfricos
d) Conducto metanéfrico
e) Epoóforo

La respuesta es la C: Los pares de conductos genitales consisten en los


conductos mesonéfricos (Wolff), que se extienden desde el mesonefros a la
cloaca, y a los conductos paramesonéfricos (Müllerianos), que discurren en
paralelo y laterales a los conductos de Wolff. Los conductos mesonéfricos en los
hombres si son estimulados por la testosterona secretada por las células de
Leydig se convierten en los conductos deferentes, el epidídimo y las vesículas
seminales. Por el contrario, porque las mujeres normales no segregan
testosterona, los conductos de Wolff involucionan y forman estructuras vestigiales.
Sin embargo, esto podría formar quistes mesonéfricos en el cuello uterino o en la
vulva o pueden formar quistes del conducto de Gartner en la vagina. El grupo
craneal de túbulos mesonéfricos (el epoóforo) permanecen como estructuras
vestigiales en el ligamento ancho por encima del ovario, mientras que el grupo
caudal de los túbulos mesonéfricos (el paraóforo) forma estructuras vestigiales en
el ligamento ancho al lado del ovario. El conducto paramesonéfrico (de Müller) en
la mujer forma las trompas de Falopio, el útero, la pared vaginal superior y la
hidátide de Morgagni. La parte inferior de la vagina y el vestíbulo se desarrollan a
partir del seno urogenital. Los hombres secretan factor de inhibición Mülleriano
(MIF) de las células de Sertoli en los testículos, la cual causa la regresión de los
conductos de Müller. Esto resulta en la formación del apéndice vestigial testicular.
El conducto metanéfrico en ambos sexos forma el uréter, pelvis renal, cálices, y el
conjunto de túbulos renales. Varias anomalías resultan de un desarrollo anormal
de los conductos de Müller. La agenesia uterina puede ser el resultado de un
desarrollo anormal o fusión de este par de conductos paramesonéfricos. Un
desarrollo incompleto en las porciones inferiores de los conductos de Müller
resulta en útero doble, mientras que una falla en la fusión de las porciones
superiores puede formar un útero bicorne. El retraso del crecimiento de uno de los
conductos paramesonéfricos junto con una fusión incompleta de los conductos
resulta en la formación de un útero bicorne con un cuerno rudimentario. (Sin
referenciar).

25)Los oogonios alcanzan su número máximo en ¿cuál de las siguientes


etapas del desarrollo humano?
a) Cinco meses de vida fetal
b) Nacimiento
c) La pubertad (12 a 14 años de edad)
d) La adolescencia (16 a 20 años de edad)
e) Edad adulta temprana (21 a 26 años de edad)

La respuesta es a. El número máximo de oogonias ocurre aproximadamente en


el quinto mes de desarrollo. Las células germinales primordiales llegan a la
gónada embrionaria de una hembra genética durante la 7° a las 12° semanas
donde se diferencian en oogonias. Después de someterse a una serie de
divisiones mitóticas, las células fetales forman grupos en la parte cortical del
ovario. Algunas de los oogonias se diferencian en los ovocitos primarios más
grandes (que no debe confundirse con folículos primarios). Los ovocitos primarios
comienzan la meiosis. Al mismo tiempo, el número de oogonias continúa
aumentando a aproximadamente 6.000.000 por el quinto mes. En este momento,
la mayoría de los oogonios sobreviven y algunos de los ovocitos se convierten en
atrésicos (respuestas b y c). Sin embargo, los ovocitos primarios que sobreviven
(400.000 a 1.000.000) se rodean por las células epiteliales y forman los folículos
primordiales por el séptimo mes. Durante la infancia (respuestas d y e) no se
continúa la atresia, de modo que en la pubertad sólo alrededor de 40.000 ovocitos
primarios permanecen. (Sin referenciar).

26)Un examen de ultrasonido de un feto de 7 meses, muestra deformidades


faciales y en extremidades. Se estima que la cantidad de fluido amniótico
es de 10 ml, cantidad considerablemente baja de lo normal. ¿Una falla en
cuál de los siguientes órganos puede iniciar el proceso de deformación
sufrido por éste feto?
a) Duodeno.
b) Esófago.
c) Corazón.
d) Riñones.
e) Pulmones.
La respuesta es D. La secuela es causada por el síndrome de Potter, en el que la
agenesia renal bilateral produce oligohidramnios (una cantidad anormalmente baja
de líquido amniótico) porque los riñones no producen cantidades normales de
orina. El oligohidramnios causa la compresión uterina del feto, que puede producir
deformidades en los miembros, posición de nalgas, e hipoplasia pulmonar. A y B
son incorrectas, la atresia duodenal y esofágica puede causar polihidramnios, y no
oligohidramnios, ya que el líquido amniótico no puede ser deglutido, pero continúa
siendo producido en los riñones. C es incorrecta, el corazón no está directamente
afectado en el síndrome de Potter, aunque serias malformaciones del corazón
podrían acompañar malformaciones de los riñones. E es incorrecta los pulmones
se vuelven hipoplasicos secundariamente como consecuencia de una cantidad
inadecuada de líquido amniótico, más que a partir de una malformación primaria.
(Sin referenciar).

27)El principal sistema del organismo que se deriva del mesodermo


intermedio del intestino posterior es:
a) Digestivo
b) Respiratorio
c) Cardiovascular
d) Urogenital
e) Nervioso

Respuesta D, (Langman 11° edición capítulo 6) El mesodermo intermedio, que


conecta temporalmente el mesodermo paraxial con la placa lateral, se diferencia
en las estructuras urogenitales. En la región torácica superior y en la cervical
forma grupos de células segmentarias (futuros nefrostomas o nefroporo), mientras
que caudalmente establece una masa no segmentada de tejido, el cordón
nefrógeno. Las estructuras del sistema urinario y las gónadas se desarrollan a
partir de este mesodermo intermedio que presenta zonas segmentadas y no
segmentadas.

APARATO LOCOMOTOR

1. ¿Qué forma el mesodermo paraxial?


a. Corteza adrenal
b. Médula adrenal
c. Húmero
d. Bíceps braquial
e. Masetero

La respuesta es D. Los músculos de las extremidades se forman a partir de los


somitas que se derivan del mesodermo paraxial; los huesos, tendones y tejido
conectivo de las extremidades se derivan del mesodermo somatopleural. El
mesodermo intermedio es el origen de los sistemas urogenitales y de la corteza
suprarrenal (respuesta a). La médula suprarrenal (respuesta b) forma la cresta
neural. El húmero (respuesta c) se forma del mesodermo somatopleural, pero los
músculos vinculados a éste son de origen de los somitas. El masetero (respuesta
e) es un músculo de la masticación formado a partir del primer arco branquial y
esta inervado por el sistema eferente visceral branquial (eferente visceral especial)
fibras del núcleo ambiguo en comparación con la inervación eferente somática
general de los bíceps y otros músculos, no del origen del arco branquial. (Sin
referenciar).

APARATO RESPIRATORIO

1) Un recién nacido de una mujer inmigrante de tercer mundo que no tuvo


atención prenatal tiene problemas respiratorios importantes y muere
minutos después de su nacimiento. La autopsia muestra intestinos
dentro de la cavidad torácica. ¿Cuál de las siguientes opciones es el
responsable de lo encontrado en la autopsia?
a) HeterotopÍa.
b) Estenosis pilórica hipertrófica.
c) Gran hernia deslizante.
d) Mal desarrollo de los pulmones.
e) Ausencia parcial del diafragma.

La respuesta es E. En algunos neonatos, hay debilidad o ausencia parcial o total


del diafragma (a menudo del lado izquierdo) que, si es lo suficientemente grande,
permitiría la herniación de los contenidos abdominales en el tórax. Pequeños
defectos son corregibles quirúrgicamente, pero los más grandes pueden causar
una falla respiratoria letal rápida en el recién nacido. A es incorrecta, Heterotopía
es la presencia de un tejido ectópico dentro de un órgano causada por la
detención de su migración desde el lugar de la matriz germinal hasta el sitio donde
debería quedar ubicado, B es incorrecta la estenosis pilórica hipertrófica congénita
puede causar vomito constante en la infancia, pero no causa herniación de los
intestinos en el tórax. C es incorrecta ya que la muerte infantil por la herniación
intestinal en el tórax no se relaciona con la hernia deslizante. D es incorrecta
debido a que el mal desarrollo de los pulmones es una consecuencia y no una
causa de los problemas de este neonato. (Sin referenciar).

2) Un neonato con múltiples anomalías congénitas presenta dificultad


respiratoria. Rayos X del tórax revelan hipoplasia severa de un pulmón.
¿Esta condición es muy probable que sea secundaria a cuál de los
siguientes problemas congénitos?
a) Defecto diafragmático
b) Defecto del cojinete endocárdico
c) Atresia esofágica
d) Gastrosquisis
e) Riñón en herradura

La respuesta es la A: Defectos significativos en el diafragma son por suerte muy


raros, pero pueden proveer un orificio lo suficientemente grande como para
permitir el paso del estómago o bucles del intestino hacia la cavidad torácica.
Cuando esto ocurre, el pulmón en desarrollo es comprimido, y puede no
desarrollarse correctamente. Defectos en los cojinetes endocárdicos (opción B)
pueden causar malformación cardíaca severa. La atresia esofágica (opción C)
puede estar relacionada con mal desarrollo de los bronquios, pero no causa
hipoplasia pulmonar. La gastrosquisis (opción D) es la extrusión del contenido
abdominal por una abertura en la pared abdominal anterior. El riñón en herradura
(opción E) usualmente es funcional, y no afecta el desarrollo pulmonar. (Sin
referenciar).

3) Del desarrollo del sistema respiratorio se puede decir que, excepto:


a) El esbozo pulmonar aparece en la cuarta semana de desarrollo.
b) El aparato respiratorio se forma por una evaginación de la pared ventral del
intestino anterior
c) El epitelio de la laringe y vías aéreas se forma del endodermo
intraembrionario
d) Los músculos y cartílagos laríngeos se originan del mesodermo lateral
definitivo esplácnico
e) El esbozo pulmonar está comunicado con el intestino anterior en su
expansión caudal

La respuesta es D. El revestimiento interno de la laringe es de origen


endodérmico, pero los cartílagos y los músculos provienen del mesénquima de los
arcos faríngeos cuarto y sexto. (Sin referenciar).
4) De la proliferación del árbol bronquial se puede decir que, excepto:
a) Se da a partir del día 26 de desarrollo intrauterino
b) Se da por una proliferación epitelial en el interior del intestino anterior
primitivo
c) El epitelio respiratorio protruye ventralmente en el mesénquima
esplacnopleural
d) Los bronquios se forman con un patrón de desarrollo simétrico bilateral
e) El bronquio izquierdo es más caudal y el derecho es más transversal

La respuesta E. Es al contrario, el bronquio izquierdo es más transversal y el


derecho es más caudal. (Sin referenciar).

5) De la formación de la laringe se puede decir que, excepto:


a) Se forma a partir del extremo craneal del esbozo respiratorio
b) Su epitelio se forma a partir del endodermo intra-embrionario
c) Sus músculos y cartílagos se forman a partir del mesénquima de los IV y VI
arcos faríngeos
d) El orificio laríngeo mantiene una posición sagital con forma de t
e) Sus músculos están inervados principalmente por el IX par

La respuesta: E. Los músculos de la laringe están inervados primordialmente por


el nervio vago (X). (Sin referenciar).

6) De la laringe se puede afirmar que:


a) Los músculos derivados del IV arco faríngeo están inervados por el nervio
laríngeo recurrente
b) Los músculos derivados del VI arco faríngeo están inervados por el nervio
laríngeo superior
c) Los límites del orificio laríngeo están dados por los cartílagos epiglotis y
aritenoides
d) Durante el crecimiento de los cartílagos laríngeos el epitelio prolifera
lentamente causando oclusión permanente del orificio laríngeo
e) Los ventrículos laríngeos se forman después de las cuerdas vocales

La respuesta C. A los lados del orificio laríngeo aparecen los engrosamientos


aritenoideos y en el extremo craneal de él se ubica el engrosamiento de la
epiglotis. Los músculos derivados del IV arco faríngeo, es decir, los músculos
constrictores de la faringe, el elevador del velo del paladar y cricotiroideo son
inervados por el nervio laríngeo superior (a). Los músculos intrínsecos de la
laringe, procedentes del sexto arco faríngeo, deben su inervación al nervio
laríngeo recurrente (b).Tal como ocurre en el intestino, el epitelio laríngeo prolifera
y obstruye temporalmente el lumen del conducto, recanalizandose hacia la décima
semana (d). Los pliegues mucosos que se insinúan hacia el lumen laríngeo forman
las cuerdas vocales falsas y verdaderas que delimitan a los ventrículos laríngeos
(e). (Sin referenciar).

7) Del desarrollo de la tráquea, bronquios y pulmones se puede decir que,


excepto:
a) El esbozo pulmonar forma la tráquea y dos evaginaciones laterales que son
los esbozos bronquiales
b) La pleura visceral deriva del mesodermo esplácnico y la pleura parietal se
origina del mesodermo definitivo somático
c) El desarrollo bronquial en el lado izquierdo forma 10 bronquios y en el
derecho 8 bronquios
d) La formación y ramificación bronquial se da por acción de la familia de FGF
e) El crecimiento de los bronquios y subsecuente diferenciación continua en el
período postnatal

La respuesta es C. En el pulmón derecho se forman 10 bronquios segmentarios,


mientras que en el pulmón izquierdo se forman 8 bronquios segmentarios. (Sin
referenciar).

8) De la maduración pulmonar se puede decir que, excepto:


a) En el estadío pseudo-glandular las vías aéreas comienzan a diferenciarse y
aparecen revestidas por un epitelio cilíndrico alto en las zonas distales y
cúbico en las proximales
b) Las glándulas mucosas se desarrollan en la semana 12
c) En el estadío canalicular se producen tres generaciones de bronquios
d) En el estadío canalicular aumenta la red capilar alrededor de los espacios
aéreos distales
e) En el estadío canalicular se forman los neumocitos tipo I

La respuesta A. Al final del desarrollo de vías aéreas aparecen las zonas distales
revestidas por un epitelio cúbico bajo mientras que las zonas proximales se
revisten de un epitelio cilíndrico alto. (Sin referenciar).

9) Son características del estadío canalicular del desarrollo bronquial,


excepto:
a) Se forman neumocitos tipo I
b) Se forman neumocitos tipo II
c) Hay disminución del tamaño de las células epiteliales respiratorias
d) Aparecen los sáculos terminales y conductos alveolares
e) Ninguna de las anteriores es cierta

La respuesta es D. El desarrollo de los sáculos terminales y conductos alveolares


se dan en la etapa post-natal, y no en la etapa canalicular. (Sin referenciar).

10)Del estadío alveolar del desarrollo bronquial se puede decir que, excepto:
a) Entre la semana 28 y 32 comienzan a formarse los alvéolos
b) Se establece una red capilar doble
c) Hay fusión de las redes capilares
d) Se comienza la producción de surfactante en los neumocitos tipo I
e) Hay expansión de los sacos y conductos alveolares

La respuesta: D. La generación de surfactante pulmonar está a cargo de los


neumocitos tipo II. Su producción comienza alrededor de la semana 24, lo que
corresponde al estadío canalicular. (Sin referenciar).

11)Un neonato con anomalías congénitas múltiples, presenta dificultad


respiratoria. Una radiografía torácica revela hipoplasia severa del
pulmón. ¿Esta es más probablemente la expresión secundaria a cuál de
los otros problemas congénitos?
a) Defecto diafragmático.
b) Defecto en las almohadillas endocardicas.
c) Atresia esofágica.
d) Gastrosquisis.
e) Riñón en herradura.

La respuesta es A. Los defectos significativos en el diafragma, son, por fortuna


raros, pero puede producir un agujero (hernia de Morgagni y de Bochdalek) lo
suficientemente grande que permita el paso del estómago o las asas del intestino
hacia la cavidad torácica. Cuando esto sucede, el pulmón en desarrollo se
comprime, y puede que no se desarrolle adecuadamente. B es incorrecto, los
defectos en las almohadillas endocardicas causan malformación cardiaca severa
porque están implicadas en separación de cavidades y formación valvular. C es
incorrecta la atresia esofágica puede estar relacionada con el mal desarrollo de la
tráquea, pero no causa hipoplasia pulmonar. D es incorrecta la gastrosquisis es la
extrusión del contenido abdominal a través de una abertura en la pared abdominal
anterior lateral al ombligo. E es incorrecta el riñón en herradura es una
malformación renal usualmente funcional, y no afecta el desarrollo pulmonar. (Sin
referenciar).
12)¿El primordio del sistema respiratorio que forma el epitelio de
revestimiento interno de la traquea, bronquio y pulmones es derivado
completamente de cuál de las siguientes entidades embriológicas?
A. Endodermo extraembrionario pared anterior.
B. Endodermo pared media.
C. Mesodermo esplácnico.
D. Estomodeo.
E. Ambos endodermo y mesodermo.

La respuesta es A. El epitelio que reviste internamente la tráquea, los bronquios


y los pulmones desde el divertículo hasta el la pared ventral anterior (endodermo)
es llamado divertículo respiratorio. B es incorrecto, el endodermo de la pared
medial proporciona el revestimiento interno de la mayoría del tracto
gastrointestinal, duodeno, intestino delgado, colon ascendente dos tercios
derechos y el colon transverso. C es incorrecto el mesodermo esplácnico
contribuye al músculo liso y el esqueleto cartilaginoso. D es incorrecto el
estomodeo es originado del ectodermo y origina la boca. El estomodeo
eventualmente se continúa con el endodermo de la pared la laringe (y tráquea) y el
esófago. E es incorrecto el epitelio que recubre internamente la tráquea, bronquios
y pulmones son derivados enteramente del endodermo de la pared anterior
ventral, no de ambos endodermo y mesodermo. (Sin referenciar).

13)El septum transverso en el feto da lugar a cuál de las siguientes


estructuras en el adulto:
a) Área desnuda del hígado.
b) Tendón central del diafragma.
c) Pericardio fibroso.
d) Omento mayor.
e) Ligamento venoso.

La respuesta es B. El diafragma torácico es derivado de cuatro estructuras


separadas: mesodermo parietal, dos membranas pleuroparietales, el mesenterio
esofágico y el septum transverso. La porción mesodérmica del septum transverso
da lugar a la porción tendinosa del diafragma. A, C, o E son incorrectas el área
libre del hígado es la porción del hígado que no está cubierta por peritoneo. Este
está en contacto con la porción central del diafragma derivada del septum
transverso. (Sin referenciar).
APARATO TEGUMENTARIO

2) De la formación de la glándula mamaria se puede decir que, excepto:


a) Se da por la interacción entre la hoja somática del mesodermo lateral
definitivo y el ectodermo epitelial
b) Se comienza a formar hacia la quinta semana de vida cuando se forma la
placa mamaria
c) La placa mamaria constituye un engrosamiento del mesodermo lateral
somático definitivo
d) Su desarrollo en hombres está inhibido por la acción de la testosterona
sobre las células mesodérmicas
e) Su desarrollo en mujeres se ve favorecido por la unión entre los lóbulos y la
superficie glandular

Respuesta: C. La placa mamaria constituye un engrosamiento del mesodermo


lateral somático definitivo. La glándula mamaria deriva del ectodermo epitelial a
partir de la línea o pliegue mamario. En el embrión de cinco a seis semanas la
línea se extiende a ambos lados y se introduce en el mesénquima subyacente,
que dan esbozos macizos a través de proliferaciones epidérmicas (embriología de
Langman, págs. 395-397).

ÓRGANOS DE LOS SENTIDOS

3) Los órganos de los sentidos se caracterizan porque se forman por la


interacción:
a) Ectodermo epitelial-mesodermo lateral esplácnico definitivo
b) Ectodermo epitelial-mesodermo lateral somático definitivo
c) Ectodermo epitelial-ectodermo neural
d) Ectodermo neural-mesodermo lateral esplácnico definitivo
e) Ectodermo neural-mesodermo lateral somático definitivo

Respuesta: C. Ectodermo epitelial-ectodermo neural. El periodo de


organogénesis transcurre entre la tercera y octava semana del desarrollo, y
durante esta las tres capas germinales, el ectodermo, mesodermo y endodermo,
originan diversos tejidos y órganos específicos. Referidos a los derivados del
ectodermo tenemos en primer lugar el neuroectodermo (tejido nervioso) por
inhibición del BMP-4 y la epidermis por presencia del BMP-4, a partir de estos de
manera bilateral se desarrollan las placodas auditivas y las del cristalino. Tiempo
después las placodas se invaginan y forman las vesículas auditivas; necesarias
para la audición y el equilibrio, y los cristalinos de los ojos. En términos generales,
la capa germinal ectodérmica origina los órganos y estructuras que están en
contacto con el mundo exterior: sistema nervioso, epitelio sensorial del oído, nariz,
ojo y epidermis de pelo y uñas (Embriología de Lagman, págs. 67, 71,72).

4) Los músculos intrínsecos estriados esqueléticos del ojo se derivan de:


a) Ectodermo epitelial
b) Ectodermo neural
c) Somitas occipitales
d) Crestas neurales cefálicas
e) Mesodermo lateral definitivo somático

Respuesta: C. Somitas occipitales. Los somitas se forman a partir del mesodermo


y al inicio de la cuarta semana tiene una disposición en forma de anillo con
paredes mediales mesenquimatosas y laterales epiteliales. El mesodermo paraxial
origina el mesénquima de la cabeza y se organizan en somitas de segmentos
caudales y occipitales. Las células de los márgenes dorsomedial y ventrolateral de
la región superior del somita forman los precursores de las células musculares,
entre los cuales se encuentran los músculos intrínsecos de los ojos a nivel de los
somitas occipitales (Embriología de Langman, págs. 76, 77).

5) La fisura coroidea en su región más anterior origina principalmente:


a) Retina
b) Córnea
c) Pupila
d) Coroides
e) Conjuntiva

Respuesta: C, Durante la séptima semana, los labios de la fisura coroidea se


fusionan y la boca de la cúpula óptica se transforma en un orificio redondo, la
futura pupila. (Sin referenciar).

6) De las vesículas ópticas se puede decir que, excepto:


a) Son evaginaciones del diencéfalo
b) Se ponen en contacto con el ectodermo superficial e inducen cambios para
la formación del cristalino
c) Su invaginación produce la cúpula óptica con una doble capa celular
d) Entre sus capas celulares se forma el cristalino principalmente
e) La superficie inferior de la cúpula óptica se establece la fisura coroidea
Respuesta: D. Entre sus capas celulares se forma el cristalino principalmente. Al
cerrarse el tubo neural, estos surcos producen evaginaciones del cerebro anterior,
las vesículas ópticas. En la etapa ulterior estas vesículas se ponen en contacto
con el ectodermo superficial e inducen en éste cambios necesarios para la
formación del cristalino. Poco después, la vesícula óptica comienza a invaginarse
y forma la cúpula óptica de pared doble, otra porción de la superficie inferior
también se invagina, y allí se forma la fisura coroidea. La placoda ulteriormente se
invagina y forma la vesícula del cristalino (Embriología de Langman, pág 381).

7) Los músculos del iris se desarrollan a partir de :


a) Ectodermo epitelial
b) Ectodermo neural
c) Ectodermo de somitas occipitales
d) Ectodermo de crestas neurales
e) Mesodermo lateral definitivo somático

Respuesta: C. Ectodermo de somitas occipitales. La región situada entre la


cúpula óptica y el epitelio superficial suprayacente es ocupada por mesénquima
laxo. En este tejido aparecen los músculos esfínter de la pupila y dilatador de la
pupila, desarrollados a partir del ectodermo subyacente de la cúpula ótica,
referente a los somitas occipitales (Embriologia de Langman, págs 281-283).

8) La coroides, esclerótica y córnea se originan de:


a) Ectodermo epitelial
b) Células mesenquimatosas
c) Ectodermo neural
d) Somitas occipitales
e) Crestas neurales

Respuesta: B. Células mesenquimatosas. Hacia el final de la quinta semana el


primórdio del ojo está rodeado completamente por mesénquima laxo, el cual se
diferencia en una capa interna y una externa. Respectivamente se forman la
coroides, capa pigmentada muy vascularizada, y la esclerótica. En resumen se da
por la diferenciación de las capas mesenquimáticas suprayacentes a la cara
anterior del ojo (Embriología de Langman, págs 284-285).

9) Una niña recién nacida se encuentra con una acomodación anormal de


los lentes. Una evaluación posterior revela una producción anormal del
humor acuoso. Una malformación de la estructura responsable de realizar
éstas funciones, puede deberse al desarrollo anormal de:
a) Axones de las células ganglionares de la retina.
b) Fisura coroidea del tallo óptico.
c) Mesodermo que rodea la copa óptica.
d) Mesodermo que rodea el tallo óptico.
e) Neuroectodermo de la copa óptica.

La respuesta es E. Los procesos ciliares que forman el humor acuoso derivan de


las copas ópticas que se derivan de las vesículas ópticas que son evaginaciones
del diencéfalo. Las dos capas de la copa óptica se fusionan y formaran la retina,
parte del cuerpo ciliar, parte interna del iris, músculos del iris, nervio óptico, en
asociación con el mesodermo coroideo. A es incorrecta los axones de las células
ganglionares de la retina que corren en el tallo óptico, se convierten en las fibras
nerviosas del nervio óptico. B es incorrecta por la fisura coroidea en el tallo óptico
pasan los vasos hialoideos que irrigaran el cristalino, el cierre de la fisura ocurre
durante la séptima semana de desarrollo. El tallo óptico es entonces llamado
nervio óptico. C es incorrecta el mesodermo que rodea la copa óptica se convierte
en la esclera y la coroides del ojo. D es incorrecta el mesodermo que rodea el tallo
óptico origina las meninges que rodean el nervio óptico. (Sin referenciar).

SISTEMA NERVIOSO

1) Un neurocientífico descubre la forma de etiquetar selectivamente las


células de la cresta neural en el desarrollo de un animal de laboratorio.
Luego del nacimiento, sacrifica al animal y examina el tejido en busca de
las células etiquetadas. ¿Cuál de los siguientes tipos celulares contendrá
la etiqueta?
a) Astrocitos.
b) Células ependimarias
c) Microglía.
d) Oligodendroglia.
e) Células Pseudounipolares.

La respuesta es E. Las células pseudounipolares de los ganglios de los nervios


craneales y espinales derivan de la cresta neural. Otros tipos celulares que
derivan de la cresta neural incluyen las células de Schwann, las células
multipolares ganglionares de los ganglios autónomos, células de cromafines de la
médula adrenal, odontoblastos, melanocitos, y las células que conforman las
leptomeninges (piamadre y aracnoides). A, B, D son incorrectas pues derivan del
tubo neural. C es incorrecta la microglia parece derivar de los monocitos
sanguíneos. (Sin referenciar).

2) La examinación microscópica del cerebro de un feto abortado reveló la


presencia de figuras mitóticas (cromosomas condensados visibles
durante la profase de la mitosis). Durante la histogénesis temprana en el
desarrollo del hemisferio cerebral, usted podría esperar que todas las
células exhibieran figuras mitóticas si están localizadas en:
a) En la zona ventricular.
b) En la capa del manto.
c) En la placa alar.
d) En la superficie pial.
e) Eventualmente distribuidas a través del espesor del manto cortical.

La respuesta es A. Durante la histogénesis en el hemisferio cerebral y en otros


sitios en el desarrollo del cerebro, las células germinales o matrices sufren la
división en la zona ventricular la cual es la región que se muestra al espacio
luminal (futuro espacio ventricular) del tubo neural. En un tiempo predeterminado
esas células pierden su habilidad para dividirse y migrar lateralmente hacia el
perímetro del tubo neural donde se acumulan como neuroblastos y forman la capa
del manto. Gradualmente el sulcus limitants forma un canal en la pared del tubo
neural y las células en la porción dorsal en la capa del manto alcanzan la placa
alar y se asocian con funciones sensoriales. Las células ventrales que este canal
forman la placa basal y eventualmente llegan a alcanzar neuronas motoras. Las
células que sufren mitosis normalmente no se encuentran en la superficie pial, y
no se distribuyen de manera uniforme a través del espesor del desarrollo del tubo
neural. (Sin referenciar).

3) Durante una visita a su ginecólogo, una paciente informa que recibió


vitamina A para el tratamiento de su acné, sin saberlo, durante los 2
primeros meses de un embarazo no detectado. ¿Cuál de los siguientes
sistemas de órganos en el feto en desarrollo es más propensos a ser
afectados?
a) El sistema digestivo
b) Los órganos endocrinos
c) El sistema respiratorio
d) Los sistemas urinario y reproductivo
e) Los sistemas nerviosos centrales y esqueléticos

La respuesta es E. La vitamina A es un miembro de la familia del ácido retinoico.


El ácido retinoico dirige la polaridad de desarrollo en el sistema nervioso central, el
esqueleto axial (columna vertebral), y probablemente el esqueleto apendicular. El
ácido retinoico induce la transcripción de diversas combinaciones de genes,
dependiendo del tipo de tejido y la ubicación (distancia y dirección desde la fuente
de ácido retinoico). Las fuentes exógenas de ácido retinoico pueden inducir la
secuencia errónea o combinación de genes, que conduce a anormalidades
estructurales en el sistema nervioso y esquelético. Los otros sistemas de órganos
que figuran no son tan susceptibles a la vitamina A (respuestas a, b, c, y d). (Sin
referenciar).

4) La corteza cerebral se forma de ¿cuál de las siguientes?


a) Telencéfalo
b) Mielencéfalo
c) Metencéfalo
d) Mesencéfalo
e) Diencéfalo

La respuesta es A. La corteza cerebral se forma del telencéfalo. La corteza se


desarrolla por ondas de proliferación, formando las capas I a VI, las capas más
internas se forman primero y las capas más superficiales más tarde. La pared del
SNC en desarrollo contiene tres capas: ventriculares, manto (intermedio), y zonas
marginales. La corteza, las áreas periféricas de la materia gris, se forma a través
de la migración de las células de la zona del manto a la zona marginal. La
segmentación del tubo neural craneal forma las vesículas cerebrales. (Sin
referenciar).

5) Una mujer de 32 años primigestante en el 1er trimestre se expone a una


persona con parotiditis. La paciente manifiesta que ella nunca ha sido
inmunizada, ha tenido paperas previamente. Siete meses después, ella da
luz a un recién nacido con macrocefalia. Un TC de cráneo muestra
hidrocefalia con alargamiento simétrico de los ventrículos tercero y
lateral. El cuarto ventrículo es de tamaño normal. ¿Cuál de las siguientes
estructuras embrionarias contribuyó al desarrollo de la estructura
obstruida en este paciente?
a) Diencéfalo.
b) Mesencéfalo.
c) Metencéfalo.
d) Mielencéfalo.
e) Telencéfalo.

La respuesta es B. Esta mujer adquirió una infección de paperas en el primer


trimestre como consecuencia de no ser inmune. Un resultado teratogénico
conocido de esto es la estenosis acueductal, resultando en agrandamiento de los
ventrículos proximales y por lo tanto en hidrocefalia no comunicante. El acueducto
cerebral (de Silvio) está en el cerebro medio o mesencéfalo. (Sin referenciar).

6) Una mujer de 32 años primigrávida de 20 semanas de gestación va por


una cita prenatal de rutina. La alfafetoproteina sérica materna esta
elevada. Un ultrasonido de alta resolución mostró una estructura similar a
un saco de 3cm por 5cm que contiene elementos neuronales que recubre
el área lumbo sacra. ¿Cuál de los siguientes explica mejor este hallazgo?
a) Falla de absorción de fluido cerebro espinal por las vellosidades
aracnoideas.
b) Falla de cierre del neuroporo caudal.
c) Falla de cierre del neuroporo rostral
d) Sobre secreción de fluido cerebroespinal.
e) Persistencia del remanente de la bolsa de Rathke´s.
f) Progresiva cavitación del canal central del cordón espinal.

La respuesta es la B. La lesión es probablemente un mielomeningocele, una


herniación de las meninges y cordón espinal a través de un defecto de la piel y las
vértebras posteriores. El mielomeningocele es asociado con el incremento de la
fetoproteina alfa (AFP), el cual puede ser detectado en el suero materno o en el
fluido amniótico. Desde el día 18 al 22, la placa neural se pliega y el tubo neural se
está formando. El neuroporo rostral (opción c) es la abertura en el extremo de la
cabeza, y se cierra en el día 25. La falla del cierre resulta en anencefalia. El
neuroporo caudal (opción B) es en el extremo distal, normalmente cierra el día 28).
La falla del cierre resulta en varias formas de espina bífida: ya sea espina bífida
oculta, o espina bífida con meningocele (herniación de las meninges),
mielomeningocele (herniación de las meninges y cordón espinal) o mielosquisis
(tubo neural abierto que se encuentran en la superficie de la espalda). La falla de
absorción del fluido cerebroespinal (CSF) por las vellosidades aracnoideas
(opción A) resulta en hidrocefalia de presión normal. Los ventrículos son
ensanchados debido a la deficiencia del paso final de absorción en la circulación
del LCR, pero en este no hay obstrucción presente. La sobresecreción de LCR
(opción D) resulta en hidrocefalia comunicante sin obstrucción. Una posible
etiología de este podría ser un papiloma de plexo coroideo. La bolsa de Rathke´s
es un sobrecrecimiento de ectodermo oral en el que la pituitaria anterior se
desarrolla. La persistencia del remanente de la bolsa de Rathke´s (opción E)
puede producir un craniofaringioma, un tumor calcificado usualmente en la silla
turca que frecuentemente exhibe algunos síntomas a través de la compresión del
quiasma óptico. La cavitación progresiva del canal central del cordón espinal
(opción F) causa siringomielia. Este más comúnmente ocurre en el canal cervical,
pero puede ocurrir en otras partes del canal, o a nivel de la médula. Resulta en la
destrucción de las fibras espinotalámicas que cruzan en la comisura blanca
anterior. La presentación temprana es con la perdida bilateral de dolor y
temperatura en manos y antebrazo. Con progresión, las neuronas motoras
inferiores en los cuernos ventrales se comprimen, resultando en parálisis flácida
de los músculos del miembro superior. (Sin referenciar).

7) Un hombre de 47 años se presenta a su neurólogo con una marcha


inestable, de base amplia y lenta y dificultad para hablar. El examen
neurológico revela disdiadococinecia, temblor de intención, hipotonía y
nistagmus. La lesión del paciente está en una estructura cerebral que
deriva de ¿Cuál de las siguientes estructuras embrionarias?
a) Diencéfalo.
b) Mesencéfalo.
c) Metencéfalo.
d) Mielencéfalo.
e) Telencéfalo.

La respuesta es C. El paciente tiene una lesión cerebelar. La disfunción del


cerebelo puede llevar a una variedad de disfunciones motoras, incluyendo ataxia
truncal (que aparece similar a la marcha de un individuo intoxicado), temblor de
intención (temblor incontrolado de la extremidad afectada que se presenta solo
con el movimiento intencionado y cuando se va a alcanzar el objetivo de la
movilidad) disdiadococinecia (la incapacidad de llevar a cabo movimientos
alternantes rápidos y regulares), dismetría (incapacidad de detener movimientos
en un punto deseado), disartria (habla atáxica), hipotonía y nistagmus. Durante la
cuarta semana de desarrollo embrionario, el extremo anterior del tubo neural
desarrolla tres vesículas: el prosencéfalo (cerebro anterior), mesencéfalo (cerebro
medio), y romboencéfalo. Para la quinta semana, se han desarrollado los
siguientes componentes: El prosencéfalo se divide en telencéfalo y diencéfalo, el
mesencéfalo no se divide y el romboencéfalo se ha desarrollado ahora en el
metencéfalo y el mielencéfalo. El cerebelo y el puente se derivan del metencéfalo.
A es incorrecta el diencéfalo que es derivado del prosencéfalo, se convierte en
tálamo, hipotálamo, epitálamo, subtálamo, el lóbulo posterior de la hipófisis y la
retina neural. B es incorrecta el mesencéfalo o cerebro medio es la única vesícula
cerebral que no produce una segunda vesícula. D es incorrecta el mielencéfalo
que es derivado del romboencéfalo, se convierte en la medula oblonga. E es
incorrecta el telencéfalo que es derivado del prosencéfalo, se desarrolla hacia los
hemisferios cerebrales (corteza cerebral, ganglios basales, amígdala y materia
blanca profunda). (Sin referenciar).
8) Un bebé recién nacido tiene una gran masa quística en la región lumbar.
La masa consistía de tejidos meníngeos y líquido cefalorraquídeo que se
ha herniado a través de un defecto de las vértebras. El niño mueve todas
sus extremidades y responde a los estímulos externos de manera normal.
¿El término más utilizado para describir la lesión del niño es cuál de los
siguientes?
a) Encefalocele.
b) Meningocele.
c) Mielomeningocele.
d) Espina bífida abierta.
e) Espina bífida oculta.

La respuesta es B. El meningocele ocurre debido a un fallo en el cierre del tubo


neural en la porción caudal. En contraste con el caso más serio de
mielomeningocele, en el cual el cordón espinal de hecho se hernia a través del
defecto vertebral, el meningocele está típicamente asociado con solo un daño
neurológico mínimo y es relativamente fácil de tratar por medios quirúrgicos. A es
incorrecta el encefalocele implica una hernia del cerebro a través de un defecto
óseo en el cráneo causado por el cierre defectuoso del extremo craneal y el tubo
neural. C es incorrecta el mielomeningocele implica una hernia del cordón espinal,
que incluye las meninges a través de un defecto vertebral. Está asociado
usualmente a déficits neurológicos severos. D es incorrecta en la espina bífida
abierta, hay un fallo completo en la fusión de los arcos vertebrales con cierre
incompleto del tubo neural de tal forma que el componente espinal esté cubierto
únicamente por piel. Además se relaciona con protrusión quística del contenido
del canal raquídeo, y el paciente usualmente tiene déficits neurológicos mayores.
E es incorrecta la espina bífida oculta es el más leve de los defectos del tubo
neural, típicamente consiste en un pequeño defecto vertebral con un cordón
espinal normal y meninges. El defecto es usualmente asociado con un hoyuelo en
la piel al que se superpone tejido adiposo, folículos pilosos o sinusoides. (Sin
referenciar).

9) Un neurocientífico de desarrollo descubre un marcador que es exclusivo


de las células neuroepiteliales. El marcador se puede detectar usando
métodos de inmunofluorescencia. ¿Cuál de los siguientes tipos de
células podría mostrar positividad para este marcador?
a) Astrocitos.
b) Células enterocromafines.
c) Melanocitos.
d) Odontoblastos.
e) Células de Schwann.
La respuesta es A. Los astrocitos y los oligodendrocitos son ambos derivados de
los glioblastos, que, a su vez, son derivados de las células neuroepiteliales. Los
otros derivados de células neuroepiteliales incluyen neuroblastos y células
ependimarias. B, C, D, E son incorrectas estas células derivan de las células de
la cresta neural. Otros derivados de las células de la cresta neural incluyen las
neuronas de los ganglios simpáticos y parasimpáticos (incluyendo la médula
adrenal), los ganglios de la raíz dorsal del sistema nervioso periférico, los ganglios
sensitivos de los nervios craneales V, VII, IX, X, y las leptomeninges (piamadre y
aracnoides). (Sin referenciar).

10)El no cierre del neuroporo craneal del tubo neural causa:


a) Anencefalia
b) Mielomeningocele
c) Espina bífida
d) Meningocele
e) Malformación de chiari

Respuesta: A. Anencefalia. Los DTN son un grupo de defectos del tubo neural,
que aparecen cuando el tubo neural no consigue cerrarse de manera efectiva. Si
el tubo neural no se cierra en la región craneal, la mayor parte del cerebro no se
forma y se conoce como anencefalia, defecto mortal. Si el cierre no se da por
cualquier otra parte de la región cervical a la caudal, el defecto se llama espina
bífida (se da con mayor frecuencia en la zona lumbosacra) (Embriología de
Lagman, págs.. 73,74).

11)Son derivados de las crestas neurales, excepto:


a) Melanocitos de la piel
b) Células de schwann
c) Ganglio de la raíz dorsal
d) Piamadre
e) Epidermis de la piel

Respuesta: E. Epidermis de la piel. Dentro de los derivados de la cresta neural


encontramos el tejido conjuntivo y huesos de la cara y el cráneo, ganglios del
nervio craneal, células de la glándula tiroidea, tabique del corazón, odontoblastos,
dermis de la cara y el cuello ganglios espinales, ganglio pre aórticos y de la
cadena simpática, ganglios parasimpáticos, medula suprarrenal, células de
Schwann, células gliales, meninges, melanocitos y células de músculo liso de los
vasos sanguíneos de la cara y el prosencéfalo. A pesar de que la epidermis de la
piel si deriva de la capa germinativa ectodérmica, no lo hace exactamente de las
crestas neurales (Embriología de Langman, págs. 72,73).

12)En el desarrollo embrionario, el SNC aparece hacia la semana:


a) Semana 1
b) Semana 2
c) Semana 3
d) Semana 4
e) Semana 5

Respuesta: C. Semana 3. Al inicio de la tercera semana del desarrollo, la capa


germinal ectodérmica tiene forma de disco, siendo más ancho en la región cefálica
que en la región caudal. La aparición de la notocorda y el mesodermo procordal
induce el ectodermo suprayacente a engrosarse y formar la placa neural. Las
células de esta placa forman el neuroectodermo (Embriología de Langman págs.
67 y 89).

13) El pliegue cervical del tubo neural lo encontramos entre:


a) Rombencéfalo y mesencéfalo
b) Mesencéfalo y telencéfalo
c) Rombencéfalo y médula espinal
d) Prosencéfalo y mesencéfalo
e) Mesencéfalo y médula espinal

Respuesta: C. Romencéfalo y médula espinal. En el extremo cefálico del tubo


neural se encuentran tres dilataciones llamadas vesículas cerebrales primarias:
prosencéfalo, mesencéfalo y rombencéfalo. Al mismo tiempo este forma dos
pliegues: cervical; en la unión del cerebro posterior y la médula espinal, y el
cefálico; en la región del cerebro medio (Embriología de Langman, págs. 67, 293).

14) Son vesículas secundarias, excepto:


a) Mesencéfalo
b) Rombencéfalo
c) Telencéfalo
d) Mielencéfalo
e) Metencéfalo

Respuesta: B. Rombencéfalo. Las vesículas cerebrales primarias son


prosencéfalo, mesencéfalo y rombencefalo y las secundarias, telencéfalo,
diencéfalo, mesencéfalo, metencéfalo y mielencéfalo (Embriología de Langman,
págs. 293-295).
15) Las células neuroepiteliales en orden de producción forman las
siguientes células:
a) Neuroblastos-neuroglia-microglia
b) Neuroblastos-neuroglia-ependimarias
c) Neuroblastos-microglia-neuroglia
d) Neuroblastos-microglia-ependimarias
e) Neuroblastos-ependimarias-neuroglia

Respuesta: B. Neuroblastos-neuroglia-ependimarias. Cuando el tubo neural se


cierra, las células neuroepiteliales empiezan a originar otro tipo de células, en
primer lugar los neuroblastos o células nerviosas primitivas, luego los glioblastos o
neuroglia como células de sostén primitivas, y finalmente se diferencian en células
ependimarias que revisten el canal central de la médula espinal (Embriología de
Langman, págs. 294-299).

16) El proceso de mielinización se da a partir de :


a) 1 mes de vida intrauterina
b) 2 mes de vida intrauterina
c) 3 mes de vida intrauterina
d) 4 mes de vida intrauterina
e) 5 mes de vida intrauterina

Respuesta: D. 4 mes de vida intrauterina. Las células de Schwann efectúan


mielinización de los nervios periféricos. Estas células se originan en la cresta
neural, emigran a la periferia y se disponen alrededor de los axones formando el
neurilema. Así a partir del cuarto mes de vida intrauterina, muchas fibras nerviosas
adquieren ese color blanquecino consecuencia del depósito de mielina
(Embriología de Langman, págs. 408).

17) El plegamiento cefalo-caudal y lateral del embrión tienen como objetivo:


a) Lograr la tridimensionalidad del cuerpo embrionario
b) Regular la diferenciación del sistema nervioso central
c) Formar el intestino primitivo a partir del saco vitelinico definitivo
d) a y c son ciertas
e) a, b y c son ciertas

Respuesta: E. Lograr la tridimensionalidad del cuerpo embrionario, regular la


diferenciación del sistema nervioso central y Formar el intestino primitivo a partir
del saco vitelinico definitivo. El tracto gatrointestinal es el principal sistema
derivado de la hoja germinativo endodérmica y du formación depende en gran
medida del plegamiento cefalocaudal y lateral del embrión. El plegamiento
cefalocaudal es causado principalmente por el crecimiento longitudinal del sistema
nervioso central y por la necesidad de otorgarle al mismo una estructura
tridimensional (Embriología de Langman, pág 95).

18)Del cierre del tubo neural se puede decir que, excepto:


a) Es un proceso mediado por las cadherinas.
b) Se da hacia el día 21 del desarrollo embrionario
c) Se da a partir de cuatro oleadas proliferativas de células del neuro-
ectodermo
d) Se da en sentido caudal a cefálico
e) La espina bífida es causada por el no cierre de la región caudal o cefálico
del tubo neural

Respuesta: D. Se da en sentido caudal a cefálico. Los pliegues neurales se


aproximan uno a otro en la línea media, donde se fusionan. Dicha fusión comienza
en la región del futuro cuello (quinto somita) y avanza hacia la cabeza y la cola, el
resultado es la formación del tubo neural antes del cierre del neuroporo craneal en
día 25 (18 a 20 somitas) y el neuroporo caudal en el día 27 (estado de 25 somitas)
(Embriología de Langman, pág 85).

19)La neurulación es un proceso que compromete:


a) Formación de la placa neural
b) Doblamiento de la placa neural
c) Fusión de los pliegues neurales
d) Oclusión del tubo neural
e) Todas las anteriores son ciertas

Respuesta: E. Formación de la placa neural, doblamiento de la placa neural,


fusión de los pliegues neurales y oclusión del tubo neural. Una vez que se produce
la inducción, la lámina alargada llamada placa neural se extiende gradualmente
hacia la línea primitiva. Al finalizar la tercera semana, los bordes laterales de la
placa neural se elevan y forman los pliegues neurales, y la porción media
deprimida constituye el surco neural. Dichos pliegues se aproximan uno a otro en
la línea media donde se fusionan, dando como resultado el tubo neural
(Embriología de Langman, págs 84-85).

20)Un recién nacido nace con múltiples deformidades físicas. Tiene labio
hendido, paladar hendido y ojos que son muy pequeños, con fisuras en el
iris, arcos superciliares planos y fisuras palpebrales inclinadas. Sus
orejas son de implantación baja y malformadas. Cada mano tiene seis
dedos y un pliegue simiano. ¿Cuál de los siguientes es la complicación
neurológica más probable asociada con este trastorno congénito?
a) Anencefalia.
b) Encefalocele.
c) Hidranencefalia.
d) Holoprosencefalia.
e) Porencefalia.

La respuesta correcta es la D. Esta es la trisomía 13, conocida también como


síndrome de Patau. La representación descrita aquí es típica. Los niños afectados
generalmente mueren antes de un año de edad, usualmente secundario a
múltiples anomalías, particularmente varios defectos congénitos del corazón.
Otras características incluyen holoprosencefalia, microftalmia, microcefalia,
labio/paladar hendido, polidactilia, cardiopatía congénita y retraso mental severo.
La anencefalia (respuesta A) es un defecto en la fusión de varios sitios de cierre
del tubo neural en el proceso de neurulación durante la embriogénesis. Esta
condición es uno de los trastornos menos comunes del sistema nervioso central
fetal, y los afectados no suelen vivir más allá de unas horas luego del nacimiento.
La encefalocele (respuesta B) es una enfermedad congénita rara, en la cual, un
divertículo del tejido cerebral y de las meninges protruyen a través de defectos en
la bóveda craneana, principalmente tiene lugar durante la cuarta semana de
gestación. Aunque su origen no se conoce con precisión, ha sido asociado a una
deficiencia materna en los niveles de ácido fólico durante el embarazo, así como
consecutiva a un traumatismo y por lo tanto producirse como las hernias. La
hidranencefalia (respuesta C) es una de las malformaciones cerebrales que se
presenta desde el final del tercer trimestre de gestación hasta los 2 años de edad,
caracterizada por compromiso isquémico, especialmente en la circulación cerebral
anterior, que induce a destrucción del parénquima cerebral y cavitación. Su
etiología ha sido atribuida a múltiples causas que conducen a la lesión isquémica,
las infecciones virales son las principales implicadas. La porencefalia (respuesta
E) es un trastorno extremadamente poco común del sistema nervioso central, que
involucra un quiste o una cavidad en un hemisferio cerebral y se ha relacionado
con traumas del parto. Por esto se considera una lesión necrótica, por interrupción
del flujo arterial durante la vida fetal. (Sin referenciar).

21)Un infante nace con hidrocefalia. El TAC demuestra una hernia del
cerebelo en el canal espinal. Éste es un ejemplo de cuál de las siguientes
opciones:
a) Malformación de Arnold-Chiari.
b) Malformación de Dandy-Walker.
c) Holoprosencefalia.
d) Lisencefalia.
e) Porencefalia.

La respuesta es A. Las malformaciones de Arnold-Chiari son hernias congénitas


en las estructuras de la parte posterior del cerebro en el canal espinal. En la
malformación Arnold-Chiari tipo I, las amígdalas cerebelosas se hernian en el
foramen magno; en el tipo II, porciones de la parte posterior del cerebro, el vermis
cerebeloso, y el cuarto ventrículo se hernian en el foramen. Este segundo tipo es
comúnmente asociado con meningoceles. Otros síntomas neonatales asociados
con la malformación de Arnold-Chiari incluyen hidrocefalia (por la obstrucción del
flujo del líquido cerebroespinal) y disfunción del tronco cerebral (causando estridor
y deglución pobre). Se pueden presentar casos más leves luego en la vida con
síntomas del cordón espinal o del cerebelo. B es incorrecto la malformación de
Dandy-Walker está asociada con hipoplasia del cerebelo pero principalmente una
imperforación de los agujeros de Magendie y Luschka agujeros que son
importantes en el drenaje del LCR del 4 ventrículo hacia el espacio subaracnoideo
lo cual redunda en un cuarto ventrículo agrandado por la obstrucción. C es
incorrecto la holoprosencefalia es una malformación compleja del cerebro, que
consiste principalmente en la no formación de los lóbulos frontales con
alteraciones en la formación de la cara de tal forma que solamente existe un solo
ventrículo alargado entre los hemisferios que no están separados. D es incorrecto
la lisencefalia o agiria, significa “cerebro liso” y es una condición caracterizada por
una falta de surcos hemisféricos y discapacidad neurológica severa. E es
incorrecto la porencefalia es una cavidad intracraneal que en su centro tiene
contenido líquido y en su exterior masa cerebral desorganizada, en la
esquicencefalia la cavidad comunica el ventrículo con la piamadre. (Sin
referenciar).

22)Un hombre de 32 años acude a su médico debido a palpitaciones y


dolores de cabeza severos, sufridos durante los últimos meses. En el
consultorio, su presión arterial es de 190/110 mmHg, pero en una revisión
de su historia clínica anterior se mostró que su presión arterial se
encontraba en un rango de 120/70 mmHg. El TC (tomografía
computarizada) abdominal mostró una masa en su glándula suprarrenal
derecha. ¿La parte de la glándula suprarrenal que contiene la masa se
deriva directamente de cuál de los siguientes tejidos embrionarios?
a) Ectodermo.
b) Endodermo.
c) Mesodermo.
d) Cresta neural.
e) Notocorda.
La respuesta es D. El paciente presenta un cuadro clínico compatible con
feocromocitoma, teniendo en cuenta su hipertensión paroxística. El
feocromocitoma es un tumor de la medula adrenal que se deriva de las células de
la cresta neural. La cresta neural también da origen a las células de los nervios
espinales y craneales, ganglios autónomos (la medula adrenal es como un ganglio
autónomo), melanocitos, leptomeninges y tejido conectivo y óseo del origen del
arco branquial. A es incorrecta, el ectodermo da lugar al sistema nervioso central,
sistema nervioso periférico, epidermis y sus apéndices, glándulas mamarias,
glándula pituitaria, esmalte dental, y la cresta neural. B es incorrecto el endodermo
da lugar al parénquima de las glándulas tiroides, paratiroides, amígdalas, timo,
hígado páncreas, y el revestimiento epitelial del tracto gastrointestinal y
respiratorio, la vejiga urinaria, uretra y tuba o trompa auditiva. C es incorrecto el
mesodermo da lugar a la corteza adrenal antes que a la medula. También da lugar
a tejido conectivo, cartílago, hueso, vasos sanguíneos y linfáticos, riñones,
gónadas, revestimiento de membranas serosas en las cavidades corporales y el
bazo. E es incorrecta la notocorda forma el núcleo pulposo de los discos
intervertebrales en la columna espinal. (Sin referenciar).

23)Un investigador está estudiando los genes envueltos en el desarrollo del


diencéfalo del cerebro. Él produce mutaciones en estos genes que
selectivamente perjudican las estructuras diencefálicas en ratones. La
secreción de qué hormonas se verá más probablemente afectada en
estas mutaciones:
a) Hormona Adrenocorticotrópica (ACTH).
b) Epinefrina.
c) Oxitocina.
d) Prolactina.
e) Hormona estimulante de la tiroides (TSH).

La respuesta es C. La neurohipófisis (pituitaria posterior) es derivada de una


evaginación de neuroectodermo diencefálico. Esta estructura es responsable de la
liberación de oxitocina y vasopresina a la circulación general. Ambas hormonas
son sintetizadas en los cuerpos celulares dentro del hipotálamo de los núcleos
paraventricular y supraóptico. A, D, E son incorrectas ACTH, Prolactina, y TSH
son todas sintetizadas y liberadas en la pituitaria anterior, o adenohipófisis, que es
derivada de una evaginación del ectodermo de la bolsa de Rathke, un divertículo
de la boca primitiva. Remanentes de ésta bolsa pueden dar a lugar a un
craneofaringioma en el transcurso de la vida. B es incorrecta la epinefrina es
sintetizada por feocromocitos y liberada en circulación por la médula adrenal, un
derivado de la cresta neural. (Sin referenciar).
24)A un neonato se le encuentra que tiene una masa en el área
sacrococcígea. El examen patológico revela que la masa es un teratoma
que contiene varios tipos de tejidos diferentes que resultan de una
persistencia de la línea primitiva. ¿La línea primitiva da lugar
normalmente a cuál de las siguientes estructuras?
a) Ganglios de la raíz dorsal.
b) Revestimiento del tracto gastrointestinal.
c) Notocorda.
d) Médula espinal.
e) Glándula tiroides.

La respuesta es C. La línea primitiva es la región del epiblasto a partir de la cual


las células migran desde el nódulo primitivo que da origen a la notocorda. El único
derivado adulto de la notocorda es el núcleo pulposo del disco intervertebral. La
explicación de los múltiples tipos de tejidos en los teratomas radica en la presencia
de células pluripotenciales que se pueden dividir y formar cualquier tipo de una
capa embrionaria por lo tanto al haber varias células pluripotenciales se forman
varios tejidos que conforman el tumor. Los demás distractores pertenecen a
estructuras que no están presentes en la organogénesis puesto que son las
estructuras definitivas. A es incorrecta los ganglios de la raíz dorsal se derivan de
las células de la cresta neural. La cresta neural se desarrolla al momento de la
neurulación, que es el proceso de invaginación del ectodermo neural que es
inducido por la notocorda. B es incorrecta el revestimiento del tracto
gastrointestinal se deriva del endodermo. El endodermo da también origen a
evaginaciones del tracto gastrointestinal, tales como el hígado y el páncreas. El
músculo liso y el tejido conectivo del tracto gastrointestinal se derivan del
mesodermo. D es incorrecta el cordón espinal se deriva del ectodermo neural, que
se invagina durante la neurulación para formar el tubo neural. La parte caudal del
tubo neural forma el cordón espinal, y la parte rostral del tubo neural forma el
cerebro. E es incorrecta la glándula tiroides se deriva del endodermo. El
endodermo del piso de la faringe se evagina para formar el ducto tirogloso, el cual
desciende hasta la glándula tiroidea. La localización adulta de la evaginación del
ducto tirogloso está marcada por el agujero ciego en la lengua. (Sin referenciar).

25)Un biólogo identifica un marcador que se expresa selectivamente en la


notocorda de un embrión en desarrollo. A continuación, se desarrolla una
técnica que consiste en la identificación del marcador por medio de
imágenes en este caso una resonancia magnética (RMI). ¿Cuál de los
siguientes tiempos representa el día antes posible que el marcador
teóricamente podría ser visualizado?
a) Día 2.
b) Día 7.
c) Día 17.
d) Día 28.
e) Día 60.

La respuesta es C. La formación de la notocorda es un evento propio de la


tercera semana, ocurriendo como una parte relativamente temprana del desarrollo
del disco trilaminar. Más específicamente, la formación de la notocorda comienza
alrededor del día 16 y el canal neuroentérico se forma alrededor del día 18. A es
incorrecta 2 días luego de la concepción la implantación no ha ocurrido aún. B es
incorrecta a los siete días de la concepción usualmente el blastocisto ha sido
implantado en el endometrio. D es incorrecto para el día 28 la gastrulación ha
tenido lugar, y la organogénesis está en marcha. E es incorrecto para el día 60 la
organogénesis se ha completado. (Sin referenciar).

26)Un neurocientífico está investigando el desarrollo del sistema nervioso.


En sus experimentos, el inyecta una tinción dentro del embrión de un
animal in vivo. Luego del nacimiento, realiza estudios histológicos para
determinar el destino de la tinción. En uno de los animales, localiza la
tinción en el asta dorsal del cordón espinal. ¿Cuál de las siguientes
localizaciones en el embrión fue el sitio más probable de inyección?
a) La placa alar.
b) La placa basal.
c) La cresta neural.
d) El extremo rostral del tubo neural.
e) Surco limitante.

La respuesta es A. El cordón espinal surge del engrosamiento ventral y dorsal a


cada lado del tubo neural. E es incorrecta durante el desarrollo, se forman una
placa alar y una basal, separadas por una ranura longitudinal llamada surco
limitante. La placa alar forma la parte dorsal (posterior) del cordón espinal y se
convierte en la porción aferente o sensorial del cordón. B es incorrecta la placa
basal está en la parte ventral (anterior) del cordón, y se convierte en la porción
motora o eferente del cordón espinal y por lo tanto contendrá las células del asta
anterior. C es incorrecta la cresta neural se desarrolla en células ganglionares
multipolares de los ganglios autónomos, células pseudounipolares de los ganglios
craneales y espinales, células leptomeningeas, células de Schwann, células
cromafines de la médula adrenal, melanocitos, mesénquima de arcos faríngeos y
odontoblastos. D es incorrecta el encéfalo se forma del extremo rostral del tubo
neural es decir de las vesículas cerebrales.
27)Son estructuras derivadas de las crestas neurales, excepto.
a) Ganglios espinales
b) Células de schwann
c) Leptomeninges
d) Oligondendrocitos
e) Ganglios craneales

Respuesta D, (Langman 11° edición capítulo 6) Una vez cerrado el tubo neural,
las células de la cresta procedentes de la región del tronco abandonan el
neuroectodermo y migran por unas de las siguientes vías: 1) una vía dorsal a
través de la dermis y 2) una ventral a través de la mitad anterior de cada somita.
Las células que siguen la primera vía entrarán en el ectodermo a través de unos
orificios de la lámina basal y formarán los melanocitos y los folículos pilosos de la
piel. Las que siguen la segunda vía se convertirán en ganglios sensitivos,
neuronas simpáticas y entéricas, células de Schwann y células de la medula
suprarrenal. Las células de la cresta neural también forman pliegues neurales
craneales, a partir de los cuales migran para abandonar el tubo neural antes de
que se cierre en esta región. Estas células ayudan a la formación del esqueleto
craneofacial, las células gliales y los melanocitos.

28)Son estructuras derivadas del telencéfalo, excepto.


a) Corteza cerebral
b) Cuerpo estriado
c) Tálamo
d) Ventrículos laterales
e) Rinencéfalo

Respuesta C, (Langman 11° edición capítulo 17) El tálamo es una estructura neuronal
que se origina en el diencéfalo (división del prosencéfalo en el embrión), es la estructura
más voluminosa de esta zona. Se halla en el centro del cerebro, encima del hipotálamo y
separado de éste por el surco hipotalámico de Monroe. Su localización es muy importante
ya que si ésta sufriera algún daño no podríamos recibir ciertos estímulos, por este motivo
está en el centro de nuestro encéfalo.

También podría gustarte